Sie sind auf Seite 1von 284

Dr Bhatia Medical Coaching Institute:Online Test Platform

test.dbmci.com /Result/ShowAllQuestionInHtml.aspx

GRAND (CLT) QUESTIONS (Formatted Paper)

SECTION

(Q.1) Levosimendan is used in the treatment of?

(a) Hypotension

(b) CHF

(c) CAD

(d) None of the above

Your Response : b

Correct Answer : B

Exp: CHF

It is used for treatment of acute & decompensated CHF, exerts potent positive inotropic
action and peripheral vasodilatory effects,

(Q.2) Slow flow Phenomena occurs in which of the following condition:

(a) Berger disease

(b) Waldenstrom Hypergamaglobinemia

(c) Ischemic heart disease

(d) Coarctation of aorta

Your Response : c

Correct Answer : C

Exp: Ischemic heart disease

Slow progression of angiographic contrast in the coronary arteries in the absence of


stenosis in the epicardial vessels in some patients presenting with chest pain.

1/284
(Q.3) Mipomersen is used in which of the following condition:

(a) Hyperlipidemia

(b) Nephrotic syndrome

(c) Atrial fibrillation

(d) Congestive heart failure

Your Response : a

Correct Answer : A

Exp: Hyperlipidemia

It is used to treat homozygous familial hypercholesterolemia and is administered by


subcutaneous injection. It builds to mRNA coding for apo B-100 which is the main
component of LDL & VLDL

(Q.4) Manifestations shown in Photograph are seen in:

(a) Beri beri

(b) Scurvy

(c) Rickets

(d) Pernicious anemia

Your Response : b

Correct Answer : B

Exp: Scurvy

Easy bruisability,
hyperaemia, swelling,
sponginess and bleeding of
gums is seen in scurvy.

2/284
(Q.5) Thiamine deficiency results in decrease energy production, because TPP:

(a) Interferes with alcohol metabolism

(b) Interferes with Transketolase activity

(c) Is a cofactor for pyruvate & alpha keto glutamate dehydrogenase activity

(d) Interferes with energy production from amino acids

Your Response : c

Correct Answer : C

Exp: Is a cofactor for pyruvate & alpha keto glutarate dehydrogenase activity

Thiamine B1 is required for Link reaction ( pyruvate dehydrogenase) & TCA cycle ( isocitrate
dehydrogenase & alpha keto glutarate dehydrogenase).

Link reaction gives 5 ATP from one glucose & TCA gives 20 ATPs from one glucose. So if
B1 deficiency, then these 25 ATPs cannot be released from one glucose. Glucose on
complete oxidation gives 32 ATPs. Therefore Thiamine deficiency result in decreased
energy production. Also note: Link reaction & TCA also required B2, B3, B5 & Lipoic acid.

3/284
(Q.6) Nanotechnology has found tremendous application in the diagnosis of cancers because
of all of following advantages, except:

(a) Nanocrystals exhibit bright, photostable fluorescence

(b) Nonocrystals have a narrow spectrum wavelength

(c) Peak spectrum wavelength is tunable

(d) Nanocrystals exhibit a narrow difference between their excitation and emission peak
spectra

Your Response : b

Correct Answer : D

Exp: Nanocrystals exhibit a narrow difference between their excitation and emission peak
spectra

● Nanocrystals, also known as Quantum Dots, are inorganic crystals. They are better
than

other fluorophores.

● They have broad , continuous excitation & emission spectrum.

● They have narrow , tunable spectrum wavelength.

● They have bright , photostable fluorescence

4/284
(Q.7) The sequence of compartment for gluconeogenesis is:

(a) Cytoplasm → Mitochondria → Endoplasmic reticulum

(b) Mitochondria→ Cytoplasm→ Endoplasmic reticulum

(c) Cytoplasm→ Mitochondria→ Golgi apparatus

(d) Endoplasmic reticulum→ Cytoplasm →Mitochondria

Your Response :

Correct Answer : B

Exp: Mitochondria→ Cytoplasm→ Endoplasmic reticulum

The first step of gluconeogenesis (Pyruvate Carboxylase) occurs in mitochondria. Then


next all steps occurs in cytoplasm, But the last step (glucose-6-phosphatase) occurs in
Endoplasmic reticulum.

(Q.8) A woman came with bleeding PR and on examination found to have fungating mass in
rectum 8 cm from the anal verge, which was biopsied and revealed adenocarcinoma. She
wishes to preserve her sphincters. What would be best treatment option

(a) Abdominoperineal resection only

(b) Neoadjuvant therapy followed by abdominoperineal resection

(c) Neoadjuvant therapy followed by low anterior resection

(d) Only Neo adjuvant therapy

5/284
Your Response : c

Correct Answer : C

Exp: Neoadjuvant therapy followed by low anterior resection

● Surgery is the only potential cure for carcinoma rectum. Because of the proximity to the
vital structure in the narrow space of pelvis, surgery for carcinoma rectum is associated
with various complications. Options of surgery include local resection or complete
resection.

● The most important decision to make is to whether it is possible to save the anal
sphincters or not. This decision is influenced by location of tumor from the anal canal. For
sphincter saving surgery to be possible tumor should be more than 2 cm above the anal
canal. Average length of anal canal is 3-4 cm. so if the tumor is less than 5 cm from anal
verge then its not possible to save the sphincters and in that case APR (abdomino-
perineal resection) is appropriate.

Terminology-

● APR (abdomino-perineal resection)- means that tumor has been resected completely
with sigmoid colon till anal verge and permanent colostomydone. This is done using both
abdominal and perineal approach usually simultaneously.

● Anterior resection- tumor has been resected with adequate margins on both sides of
bowel ends and colorectal anastomosis made. Term anterior means surgery has been done
above the reflection of peritoneum on the bowel.

● Low anterior resection - if the resection require dissection below the reflection of
peritoneum its called Low. Rest is same as anterior resection with resection of tumor and
colorectal anastomosis done.

● So, both LAR and anterior resection preserves sphincters while APR sacrifices
sphincter and end up with a permanent colostomy.

Role of neoadjuvant therapy-

● Role of neoadjuvant therapy in rectal cancer is now well established. It has some
distinct advantages -

● Reduces local recurrence rate

● Help reduce tumor size making a previously inoperable cancer operable.

● Neoadjuvant therapy has become standard as initial treatment of locally advanced


rectal cancer.

● Therapy can use radiotherapy, chemotherapy or a regimen combining both.

● Despite rectal cancer responding dramatically to neoadjuvant therapy, they are not the
replacement for surgery. If a tumor is operable, surgery is the best chance for cure.

6/284
(Q.9) A 65-year-old female on her routine examination was noted to have a pulsatile abdominal
mass. She has been otherwise healthy with history of hypertension with no other history,
except family history of father dying of ruptured abdominal aortic aneurysm. What are the
acceptable reasons to operate on abdominal aortic aneurysms in 65-year-old female with 5-
cm infrarenal aneurysm?

(a) Presence of aneurysm

(b) Aneurysm with intramural thrombus

(c) Asymptomatic aneurysm 5.5 cm

(d) Associated 2-cm iliac aneurysm

Your Response : b

Correct Answer : C

Exp: Asymptomatic aneurysm 5.5 cm

● The current indication for repair of abdominal aortic aneurysm in female includes
aneurysm size 5 cm in acceptable risk patient.

● A small aneurysm study has increased the size that could be observed to 5.5 cm in
male while in female it is acceptable to treat aneurysm at 5 cm size for acceptable risk. Any
aneurysm with associated complication should be treated; just the presence of intramural
thrombus does not justify repair.

● Asymptomatic 5.5-cm aneurysm should be treated in all patients, male or female, at


acceptable cardiac risk.

● Patients with 2-cm aneurysm of iliac artery without any symptoms and complications
should be observed; as the risk of surgery is higher than risk of observation till they reach to
4 cm.

● In patients, not in child-bearing age, 1.5-cm splenic aneurysm could be observed

7/284
(Q.10) Which of the following vascular beds experiences the most vasoconstriction in a person
near the end of a 10-km run?

(a) Cerebral

(b) Exercising muscle

(c) Intestinal

(d) Skin

Your Response : c

Correct Answer : C

Exp: Intestinal

Distribution of CO during heavy exercise situation

● Working muscle receives 71% of cardiac output.

● Skin blood flow is also increased to meet the thermoregulatory demands of exercise.

● The absolute amount of coronary muscle blood flow muscle increase although the
%CO remains relatively constant.

● The absolute amount of cerebral blood flow remains constant, which means that the
%CO distributed to brain decreases.

● Both renal and splanchnic blood flow decreased as exercise intensity increases.

8/284
(Q.11) Effective renal plasma flow (ERPF) is calculated by:

(a) DTPA

(b) MAG 3

(c) Inulin e

(d) Hippurate I 131

Your Response : b

Correct Answer : D

Exp: Hippurate I 131

Effective Renal plasma flow (ERPF) = 625 mL/min . Renal plasma flow (RPF) = 700
mL/min. eRPF measure by Hippurate.

(Q.12) Which of the following is an Atypical vitamin?

(a) Vitamin B3 (Niacin)

(b) Vitamin B12 (Cobalamine)

(c) Vitamin C (Ascorbate)

(d) Vitamin B9 (Folate)

Your Response : b

Correct Answer : A

Exp: Vitamin B3 (Niacin)

Vitamin B3 (Niacin) & Vit D are considered atypical Vitamins as they can be synthesized
in body. Do not consider those Vitamins which are synthesized by intestinal flora. (Vitamin
B3 is synthesized from amino acid-Tryptophan).

9/284
(Q.13) A 3 month old infant present with hepatosplenomegaly and failure to thrive. A liver
biopsy reveals glycogen with an abnormal, amylopectin like structure with long outer chains
and missing branches. Which of the following enzymes would most likely be deficient?

(a) Alpha Amylase

(b) Branching enzyme

(c) Debranching enzyme

(d) Glycogen phosphorylase

Your Response : c

Correct Answer : B

Exp: Branching enzyme

During Glycogenesis, branching enzyme creates branch points and further elongation is
carried out by Glycogen synthase. In the deficiency of Branching enzyme stored glycogen
is abnormal, in the form of long polysaccharide chains with few branch points, resembling
the structure of Amylopectin, thus this defect is also called Amylopectinosis. Alpha
Amylase is an enzyme for digestion of starch and glycogen. Debranching enzyme
deficiency results in the accumulation of abnormal glycogen, there is inability to remove
the branch points, the resultant structure resembles Limit dextrin, and thus it is also called
Limit dextrinosis. Glucose 6 phosphatase deficiency Is observed in Von-Gierke’s
diseases, a type 1 glycogen storage disease, the stored glycogen is always normal in
chemistry.

10/284
(Q.14) Which of the following is correct about RBCs?

(a) Deficiency of Pyruvate Kinase is not common in RBCs

(b) Glucose enters RBCs by sodium dependent glucose transport (SGLT)

(c) Deficiency of HMP pathway enzyme in RBCs will lead to anemia

(d) RBCs does not contains enzymes of nucleotide metabolism (like Adenosine deaminase)

Your Response :

Correct Answer : C

Exp: Deficiency of HMP pathway enzyme in RBCs will lead anemia

Deficiency of Pyruvate Kinase (enzyme of glycolysis) is common. Pyruvate kinase


deficiency is the second most common human enzyme deficiency. & Glucose-6-P
dehydrogenase deficiency is the first most common human enzyme deficiency.

● Glucose enters RBCs by facilitative transport (GLUT -1)

● Deficiency of HMP pathway enzyme (Glucose-6-P dehydrogenase) & glycolysis


enzymes in RBCs leads to haemolytic anemia

● Biosynthesis of glycogen, fatty acids, protein & nucleic acid does not occurs in RBCs.

● RBCs contain some enzymes of nucleotide metabolism (like Adenosine deaminase)

11/284
(Q.15) If Aerobic glycolysis uses glycerol-3-phosphate shuttle, How many ATPs are produced?

(a) 2 ATP

(b) 5 ATP

(c) 7 ATP

(d) 3 ATP

Your Response :

Correct Answer : B

Exp: 5 ATP

In glycerol-3-phosphate shuttle, NADH taken form cytoplasm but FADH2 delivered into the
mitochondria. So, now 2 FADH2 gives 3 ATPs. 2 ATPs used at Hexokinase & PFK -1
(Phosphofructo kinase-I). 4 ATPs are produced at substrate level Phosphorylation steps.
So, 3+4-2 = 5 ATPs.

12/284
(Q.16) Identify the sign and the related pathology with it:

(a) Tear drop sign, Fracture


orbital floor

(b) Bare orbit sign: greater


Sphenoidal wing hypoplasia

(c) Bare orbit sign: Lesser


Sphenoidal wing hypoplasia

(d) Bare orbit sign: NF2

Your Response :

Correct Answer : B

Exp: Bare orbit sign: greater


Sphenoidal wing hypoplasia

Bare orbit sign is characteristic


appearance of orbit where
innominate line is absent,
Innominate line is a projection
of greater wing of sphenoid.

13/284
(Q.17) Which one of the following radioisotopes is commonly used as external beam
radiotherapy in treatment of cancer patients?

(a) Strontium 89

(b) Radium 226

(c) Cobalt 59

(d) Cobalt 60

Your Response : a

Correct Answer : D

Exp: Cobalt 60

● A commonly used radioisotope for external beam irradiation in treating cancer


patients is cobalt 60.

● Cobalt emits two gamma rays per disintegration. The dose rate at any practical
distance from a gamma ray emitting point source in air can be obtained by means of
inverse square law.

● Besides radium, at present, radioactive cobalt, gold and iridium are being employed in
small sources used in body cavities or implanted directly into tissues (brachytherapy).

14/284
(Q.18) A 13 year old present with a 3 month history of a painful left knee. There is no preceding
history of trauma. An AP left knee x-ray demonstrates a well defined lytic lesion with a thin
sclerotic rim in the periphery of the tibial epiphysis. A subsequent CT of the region
demonstrates that the cortex is intact and the lesion is confined to the epiphysis. MRI shows
that the lesion is high signal on STIR and T2 weighted images with surrounding marrow
oedema and fluid- fluid levels. What is the most likely diagnosis?

(a) Primary aneurismal bone cyst

(b) Simple bone cyst

(c) Chondroblastoma

(d) Chondromyxoid fibroma

Your Response : a

Correct Answer : C

Exp: Chondroblastoma

Epiphyseal lesion, age less than 20, chondroblastoma.

15/284
(Q.19) Following appearance is seen in :

(a) CML

(b) Osteoporosis

(c) Osteopetrosis

(d) Bone infarct

Your Response : c

Correct Answer : C

Exp: Osteopetrosis

In osteopetrosis there is reduced


osteoclastic bone resorption resulting
in diffuse symmetrical skeleton
sclerosis. Also k/a marble bone
disease d/t its stone like quality of
bones; however the bones are
abnormally brittle and fracture like a
piece of chalk. It can present
radiologically as

a. Sclerosis of all bones more


prominent at base of skull

b. Sclerosis of vertebral end plate l/t


characteristic sandwich or broad
stripped (rugby jersey spine).

c. Bone in bone appearance d/t


sclerotic foci with in the bone.

Bone within bone sign:

Inner layer of cortical new bone


within an existing bone seen with
osteopetrosis, sickle cell disease.

16/284
(Q.20) On reaching the crime scene, it is found that body has been shifted to some other room
along with his wallet & identity card. This comes under which IPC?

(a) 375

(b) 201

(c) 313

(d) 84

Your Response : b

Correct Answer : B

Exp: 201

● The given question says about disappearance of evidence.

● This comes under IPC 201.

● Option A-IPC 375 defines rape.

● Option C-IPC 313- Provides punishment for voluntarily causing miscarriage without
the consent of woman with imprisonment of either description for life/ upto 10 years with
fine.

● Option D-IPC 84 defines about criminal responsibility of insane.

17/284
(Q.21) In Aluminium phosphide poisoning, patient feels the taste of?

(a) Garlic

(b) Rotten egg

(c) Fishy

(d) Bitter

Your Response : a

Correct Answer : C

Exp: Fishy

18/284
(Q.22) Identify the syndrome:

(a) Prader Willi Syndrome

(b) Turner Syndrome

(c) Down Syndrome

(d) Marfan Syndrome

Your Response : a

Correct Answer : D

Exp: Marfan Syndrome

Elongated facies, droopy lids, apparent dolichostenomelia, and mild scoliosis.

19/284
(Q.23) Phenylbutyrate is used in urea cycle disorder because it:

(a) Scavenges nitrogen

(b) Activates enzymes

(c) Maintains renal output

(d) Maintains energy production

Your Response :

Correct Answer : A

Exp: Scavenges nitrogen

Sodium phenylbutyrate metabolites allows the kidney’s to excrete excess nitrogen in place
of urea. Phenylbutyrate conjugates with glutamate and forms a complex, which is
excreted in urine. It contains the same amount of nitrogen as urea, which makes it all
alternative to urea for excreting nitrogen.

20/284
(Q.24) A child with pellagra like symptoms, amino acids in urine, family history of two siblings
affected & two normal. Parents are normal. What is the diagnosis?

(a) Phenylketonuria

(b) Alkaptonuria

(c) Maple syrup urine disease

(d) Hartnup’s disease

Your Response :

Correct Answer : D

Exp: Hartnup’s disease

Hartnup’s disease is Autosomal Recessive. There is failure to reabsorb tryptophan from


urine. So tryptophan comes in urine (Aminoaciduria). Patient has Pellagra like symptoms
(as Tryptophan forms Niacin – Vitamin B3 in body).

(Q.25) Which of the following therapies has been found to be effective in long-term follow-up for
some patients with the X-linked neurodegenerative disease adrenoleukodystrophy?

(a) Vegetable oil as a dietary supplement

(b) Hyperbaric oxygen therapy

(c) Bone marrow transplantation

(d) Total volume exchange blood transfusion

Your Response : d

Correct Answer : C

Exp: Bone marrow transplantation

Performed prior to the onset of severe symptoms, bone marrow transplantation has had
some value in improving the clinical course in some patients with X-linked
adrenoleukodystrophy.

(Q.26) In case of cause of death in sea water due to method shown in the picture below which
among the following occurs?

21/284
(a) Hemoglobin increases.

(b) Hemoglobin decreases.

(c) No change.

(d) Either may occur.

Your Response : a

Correct Answer : A

Exp: Hemoglobin increases.

In the image, the mode of death of the person is due to drowning in sea water.

In case of drowning in sea water hemoglobin increases.

22/284
(Q.27) The age of criminal liability is:

(a) 12 years

(b) 14 years

(c) 16 years

(d) 17 years

Your Response : b

Correct Answer : A

Exp: 12 years

● The age of criminal liability is 12 years.

● In the words of section 3 of the Juveniles Act, "It shall be conclusively presumed that
no child under the age of 12 years can be guilty of any offence."

(Q.28) The Court of a Chief Judicial Magistrate may pass any imprisonment for a term less than

(a) 5 yrs.

(b) 7 yrs.

(c) 9 yrs.

(d) 10 yrs.

Your Response : a

Correct Answer : B

Exp: 7 yrs.

The Court of a Chief Judicial Magistrate may pass any sentence authorised by law except
a sentence of death or of imprisonment for life or of imprisonment for a term exceeding
seven years

23/284
(Q.29) What is the normal time of disappearance of this neonatal reflex?

(a) Birth

(b) 2-3 months

(c) 5-6 months

(d) Persists throughout life

Your Response : c

Correct Answer : C

Exp: 5-6 months

This is the asymmetrical tonic neck reflex (ATNR). It appears at about 35 weeks of IU life,
and disappears by 5-6 months of age.

24/284
(Q.30) A 4-year-old child is brought in by her mother because of a painful, honey colored,
crusted lesion on her face. The most likely diagnosis is:

(a) Malaria

(b) Impetigo

(c) Cellulites

(d) Seborrheic dermatitis

Your Response : b

Correct Answer : B

Exp: Impetigo

● Impetigo is a contagious staphylococcal or streptococcal infection of the epidermis.

● It presents initially as a vesicle or bulla, which ruptures to generate a honey –


colored, crusted exudates.

● Cellulitis may also be caused by staphylococcus or streptococcus, but entails a


dermal or subcutaneous infection that presents as a hot, tender, and erythematous region
of skin without an exudates.

● Seborrheic dermatitis, or “cradle cap” is a scaly, erythematous lesion that may also be
crusted but is generally localized to the scalp and intertriginous areas.

25/284
(Q.31) Which non- governmental organisation does the given logo represent?

(a) Oxfam

(b) Cochrane

(c) Save the children

(d) Amnesty International

Your Response : b

Correct Answer : B

Exp: Cochrane

Cochrane is a British non- profit,


non- governmental organization
formed to organize medical research
findings. It helps policy makers,
health professionals and patients to
make evidence- based choices about
health interventions. The Cochrane
logo illustrates a meta- analysis of
data from seven randomized
controlled trials (RCTs), comparing
one health care treatment with a
placebo in a forest plot.

26/284
(Q.32) Mission Indradhanush provides vaccination against all of the following diseases except:

(a) Polio

(b) Tuberculosis

(c) Hepatitis- B

(d) Typhoid

Your Response : b

Correct Answer : D

Exp: Typhoid

Mission Indradhanush was launched on 25 December, 2014. It aims to immunize all


children under the age of 2 years, as well as all pregnant women, against seven vaccine
preventable diseases. The diseases targeted are diphtheria, whooping cough, tetanus,
poliomyelitis, tuberculosis, measles and Hepatitis- B. In addition, vaccines for Japanese
Encephalitis and Haemophilus influenza type B are also being provided in selected states.

27/284
(Q.33) The second phase of Measles- Rubella vaccination campaign in India will cover all of the
following states except:

(a) Andhra Pradesh

(b) Madhya Pradesh

(c) Uttarakhand

(d) Telangana

Your Response : d

Correct Answer : B

Exp: Madhya Pradesh

The first phase of measles- rubella vaccination campaign was launched across five states
in India- Tamil Nadu, Karnataka, Goa, Lakshadweep and Puducherry- in February 2017.
The drive was carried out at schools, community centres and medical institutes, covering
children between ages of 9 months and 15 years. The Centre has rolled out the second
phase of measles- rubella vaccination campaign across 8 states and Union territories-
Andhra Pradesh, Chandigarh, Himachal Pradesh, Kerala, Telangana, Uttarakhand, Dadra
and Nagar Haveli and Daman and Diu. After the special drive is completed, the measles-
rubella vaccine will be made part of routine immunisation process replacing the measles
vaccine that is currently being given between 9-12 months and 16-24 months after birth.

(Q.34) A poison which is taken from the seeds shown below and that produces petechial
hemorrhages under the skin and peritoneum is?

28/284
(a) Abrusprecatorius

(b) Castor

(c) Cannabis

(d) Dhatura

Your Response : a

Correct Answer : A

Exp: Abrusprecatorius:

● Also known as Jequirity/lndianliquorice, Ratti.

● Active principle - Abrin - found in seeds, Abralin

● All the parts are poisonous.

● Seeds are tasteless and odourless.

● Fatal dose: 90-120 mg of abrin.

● Produces tetanic convulsions.

● At postmortem- Petechial hemorrhages are seen under the skin, pleura, pericardium,
and peritoneum

● Used as - Cattle poison and abortifacient.

● Suis are prepared mixing it with onion, water and Dhatura, which are 15 mm long.

● Two suis are slapped over the skin so that entry site looks like snakebite as
symptoms resemble viper snakebite.

29/284
(Q.35) Which of the following strain is not present in Bivalent Oral Polio Vaccine ?

(a) Type 1

(b) Type 2

(c) Type 3

(d) None of the above

Your Response : b

Correct Answer : B

Exp: Type 2

Polio has been eradicated from India but neighbouring countries still have active
circulation of polio virus. This poses a threat of importation of virus to India. Therefore, it is
important to maintain high population immunity through continued polio vaccination. Wild
poliovirus type 2 has been eradicated from the world, so there is no need to continue
giving tOPV containing polioviruses type 1, 2 and 3. Bivalent Oral Polio Vaccine (bOPV)
which contains only type 1 and 3 has completely replaced tOPV throughout the world. The
shift from tOPV to bOPV is called the SWITCH. In India, the National Switch Day was 25
April, 2016.

(Q.36) Which route can be used for the administration of Dengvaxia- the recently developed
vaccine against dengue ?

(a) Subcutaneous

(b) Intradermal

(c) Intramuscular

(d) None of the above

Your Response : a

Correct Answer : A

Exp: Subcutaneous

Dengvaxia is used to protect adults or children against dengue disease caused by


dengue virus serotypes 1, 2, 3 and 4. It is given to adults, adolescents and children 9 to 45
years of age living in endemic areas. 3 doses of 0.5 ml each are given at 6 month
intervals. It is given subcutaneously in the upper arm.

30/284
(Q.37) Gantt chart is used to illustrate:

(a) Project schedule

(b) Geographical distribution of a disease

(c) Both of the above

(d) None of the above

Your Response : a

Correct Answer : A

Exp: Project schedule

A Gantt chart, commonly used in project management, is a useful way of showing


activities (tasks or events) displayed against time. On the left of the chart is a list of the
activities and along the top is a suitable time scale. Each activity is represented by a bar.
The position and length of the bar reflects the start date, duration and end date of the
activity.

(Q.38) The targets of National Health Policy 2017 include all of the following except:

(a) Reduce Total Fertility Rate (TFR) to 2.1 at sub- national and national level by 2025

(b) Reduce Maternal Mortality Rate (MMR) to 100 by 2020

(c) Reduce Infant Mortality Rate to 28 by 2019

(d) Reduce mortality rate of children under 5 years to 23 per 1000 by 2020

Your Response :

Correct Answer : D

31/284
Exp: Reduce mortality rate of children under 5 years to 23 per 1000 by 2020

The Union Cabinet has approved the National Health Policy 2017. It will replace the
previous policy which was framed 15 years ago in 2002. It aims at providing healthcare in
an “assured manner” to all by addressing current and emerging challenges arising from
the changing socio-economic, epidemiological and technological scenarios.

Highlights of National Health Policy 2017

(i) It aims to raise public health care expenditure to 2.5% of GDP from current 1.4%,
with more than two-thirds of resources going towards primary healthcare.

(ii) It aims at providing assured comprehensive primary healthcare through the “Health
and Wellness Centres”.

(iii) It includes care for major non-communicable diseases (NCDs), geriatric healthcare,
mental health, palliative care and rehabilitative care services.

(iv) It proposes free diagnostics, free drugs and free emergency and essential
healthcare services in all public hospitals.

(v) It advocates development of mid-level service providers, public health cadre, nurse
practitioners to improve availability of appropriate health human resource.

(vi) It looks at reforms in the existing regulatory systems both for easing drugs and
devices manufacturing to promote Make in India and also reforming medical education.

Targets of National Health Policy 2017

(i) Ensure availability of 2 beds per 1000 population to enable access within golden
hour.

(ii) Increase life expectancy from 67.5 to 70 years by 2025.

(iii) Reduce total fertility rate (TFR) to 2.1 at sub-national and national level by 2025.

(iv) Reduce mortality rate of children under 5 years of age to 23 per 1000 by 2025 and
maternal mortality rate to 100 by 2020.

(v) Reduce infant mortality rate to 28 by 2019 and reduce neonatal mortality rate to 16
and still birth rate to “single digit” by 2025.

(Q.39) Tattooing in an old body as shown in the picture below can be visualized by use of all
except ?

32/284
(a) H2O2 3%.

(b) Gamma rays.

(c) Infrared photography.

(d) Examination with magnifying glass.

Your Response : d

Correct Answer : B

Exp: Gamma rays.

Tattoo marks

● Tattoo marks are designs made by multiple small puncture wounds made through the
skin with needles or similar penetrating tools dipped in colouring agents (dye).

● Tattoos are distinctive, potentially important acquired identifying features that last for
the life of the bearer, and persist into the postmortem period.

Latent tattoo marks are faded marks and they can be visualized by

● Use of ultraviolet light

● Infrared photography

● Rubbing the part and examining under magnifying glass

● If tattoo marks are obscured by decomposition they can be visualized by treating with
3 % H2O2.

● Tattoo marks can be developed by treating the skin by 0.5% caustic potash.

● Histopathology of local lymph nodes for pigment.

33/284
(Q.40) For the monitoring and containment of which of the following disease has the
Munialappa Committee been constituted ?

(a) Swine flu

(b) Bird flu

(c) Malaria

(d) Chikungunya

Your Response : b

Correct Answer : B

Exp: Bird flu

The Union Agricultural Ministry has constituted a High-Level Committee to monitor the bird
flu situation which will be headed by Joint Commissioner in the Department of Animal
Husbandry, Dairy and Fisheries (DADF) Munialappa. The Committee will also assist the
state governments in taking steps to contain the spread of avian influenza. The committee
was constituted after mortality among birds in National Zoological Park, Delhi NCR and
other parts of the country due to Avian Influenza Virus (H5N1) was found.

34/284
(Q.41) According to the latest data released by the Registrar General of India in October 2016,
which of the following states has surpassed Kerala as the state with the highest life
expectancy in India for all ages, barring life expectancy at birth?

(a) Himachal Pradesh

(b) Maharashtra

(c) Jammu and Kashmir

(d) Rajasthan

Your Response : b

Correct Answer : C

Exp: Jammu and Kashmir

J&K has surpassed kerala as the state with the highest life expectancy in India for all
ages, except life expectancy at birth, according to the latest data released on October 19,
2016, by the Registrar General of India (RGI). Not too bad poverty levels, more equitable
land ownership and temperate climate in Kashmir valley that is less conducive for tropical
diseases and pollution may have created conditions for this higher life span.

(Q.42) Diagram used in:

(a) FGD

(b) Panel discussion

(c) Symposium

(d) Lecture

Your Response : b

Correct Answer : A

Exp: FGD

This is a balanced sociogram used


in focused group discussion which
indicates participation of the
people.

35/284
(Q.43) “Project Sunrise” has been launched by the Union Government to combat the increasing
prevalence of which of the following disease in North- Eastern States ?

(a) Tuberculosis

(b) HIV

(c) Malaria

(d) Dengue

Your Response : b

Correct Answer : B

Exp: HIV

North-eastern states like Manipur, Nagaland and Mizoram account for highest adult (15-49
years) HIV prevalence in the country. Project Sunrise has been launched for the
prevention of AIDS especially among people injecting drugs in the 8 North- eastern states.
It aims to diagnose 90% of such drug addicts with HIV and put them under treatment by
2020. The project will bring people living with HIV/AIDS into the national mainstream and
create more awareness about the disease.

(Q.44) As per WHO, which of the following is the first Asian country to eliminate mother- to-
child HIV transmission ?

(a) Indonesia

(b) Sri Lanka

(c) Thailand

(d) India

Your Response : c

Correct Answer : C

Exp: Thailand

Thailand is the first country in Asia and the Pacific region to have eliminated
mother-to-child transmission of HIV and syphilis.

36/284
(Q.45) According to the Biomedical Waste Management Rules 2016, which container is used for
the collection of contaminated recyclable waste like catheters and gloves ?

(a) Cardboard boxes with blue coloured marking

(b) Red coloured non- chlorinated plastic bags or containers

(c) Yellow coloured non- chlorinated plastic bags

(d) Black coloured non- chlorinated plastic bags

Your Response : c

Correct Answer : B

Exp: Red coloured non- chlorinated plastic bags or containers

Red coloured non-chlorinated plastic bags or containers are used for collection of
recyclable contaminated waste generated from disposable items like tubing, bottles,
intravenous tubes and sets, catheters, urine bags, syringes and gloves.

37/284
(Q.46) What is the sign ? What is the diagnosis?

(a) Hilar lymphadenopathy sarcoidosis

(b) Vanishing tumour, TTN

(c) Hampton’s sign: Pulm. Embolism

(d) Thymic Sail sign Pneumomediastinum

Your Response : d

Correct Answer : D

Exp: Thymic Sail sign Pneumomediastinum

Its thymic sail sign seen in pneumomediastinum also known a s spinnaker sign.

38/284
(Q.47) “Undertaker’s fracture” is commonly seen in:

(a) Skull

(b) Cervical spine

(c) Lumbar spine

(d) Pelvis

Your Response : b

Correct Answer : B

Exp: Cervical spine

● Undertaker’s fracture is basically a postmortem fracture, and occurs due to careless


handling of the dead body by undertakers; in fact this fracture can be produced by anyone
who has to do anything with the dead body.

● Pathologists are known to extend the head to make the removal of the neck structures
easier. To do this, they usually insert a block about 10-15 cm high under the neck may fall
forcibly backwards, producing this fracture.

● It involves subluxation of the lower cervical spine due to tearing of the intervertebral
disc at about C6-C7

39/284
(Q.48) Telemedicine Consultation Centre (TCC) is the site where:

(a) The patient is present

(b) The specialist is present

(c) Both of the above

(d) None of the above

Your Response : b

Correct Answer : A

Exp: The patient is present

Telemedicine is the use of electronic information to communicate technologies to provide


and support healthcare when distance separates the participants. It facilitates healthcare
in remote, rural areas. Telemedicine Consulting Centre (TCC) is the site where the
patient is present. It has equipment for scanning/ converting, transformation and
communicating the patient’s medical information. Telemedicine Speciality Centre (TSC)
is the site where the specialist is present.

40/284
(Q.49) Concept of “Hospice” is related to:

(a) Euthanasia for terminally ill patients

(b) An association running orphanages for abandoned children

(c) Special group of people helping the old and terminally ill patients

(d) None of the above

Your Response : a

Correct Answer : C

Exp: Special group of people helping the old and terminally ill patients

Hospice care is a type of care that focuses on the palliation of chronically ill, terminally ill
or seriously ill patients’ pain and symptoms, and attending to their emotional and spiritual
needs. A team of health care professionals and volunteers provides it. It focuses on quality
rather than length of life.

(Q.50) “Medusa Lock” appearance in X-ray abdomen is seen in:

(a) Ascariasis

(b) Tapeworm

(c) Hookworm

(d) Ascaris and tapeworm

Your Response : a

Correct Answer : A

Exp: Ascariasis

Larger collection of coiled worms of Ascaris lumbricoides giving the “Medusa lock”
appearance may be identified on plain abdominal radiographs itself.

The worms can be identified in barium studies as single or multiple smooth longitudinal or
coiled filling defects, sometimes with a thin central track of barium outlining the worm’s
intestinal tract.

41/284
(Q.51) Radio graphical findings in Freiberg disease is :

(a) Depending on the stage of the disease, radiographs may show only sclerosis and
widening of the joint space (early), with complete collapse of the metatarsal head and
fragmentation later.

(b) Oblique views may be especially useful to fully appreciate subtle changes early in the
disease.

(c) Osteochondral loose may be seen late in the disease as well.

(d) All are true.

Your Response : d

Correct Answer : D

Exp: All are true.

● It is osteochondroses of head of 2nd of 2nd (3rd/4th) metatarsal.

● The diagnosis of Freiberg disease is relatively straightforward when patients present


with the typical complaints of activity-related forefoot pain with passive motion of the MTP
joint and pain with palpation over the metatarsal head.

● The differential diagnosis may include metatarsalgia, Morton neuroma, stress fracture
of the metatarsal, or synovitis. Physical examination typically reveals a limited range of
motion (ROM), swelling, and tenderness with direct palpation of the MTP joint. A small
effusion may be present.

● A callus may be seen underneath the affected metatarsal head. Occasionally,


patients are completely asymptomatic with changes noted only on radiographs taken for
other purposes.

● X rays in Early stage shows flattening increased density cystic lesions lesions of
metatarsal head, widening of matatarsophangeal joint.

● X rays in late stage shows osteochondral fragment, sclerosis, flattening of metatarsal


head with increased cortical thickening.

42/284
(Q.52) Under which programme has the SAATHIYA SALAH mobile app been launched ?

(a) Janani Suraksha Yojana

(b) Rashtriya Kishore Swasthya Karyakram

(c) Janani Shishu Suraksha Karyakram

(d) NavjatShishu Suraksha Karyakram

Your Response : b

Correct Answer : B

Exp: Rashtriya Kishore SwasthyaKaryakram (RKSK)

The Union Ministry of Health and Family Welfare has launched SAATHIYA Resource Kit
and SAATHIYA SALAH mobile app for adolescents as part of the Rashtriya Kishor
Swasthya Karyakram (RKSK) programme. The kit and app will help Peer Educators
(Saathiyas) who are introduced under RKSK programme. The Saathiyas act as catalyst
for generating demand for adolescent health services and also impart age appropriate
knowledge on key adolescent health issues to their peer groups.

43/284
(Q.53) The patient is HIV positive , and LP is india ink positive ? What is the MRI finding ?

(a) Leptomeningeal enhancement

(b) Ventricular dilatation

(c) Virchow – Robin space


dilatation

(d) All of the above

Your Response : d

Correct Answer : D

Exp: All of the above

It is prominent virchow robbin


space in cryptococcal meningitis.

Rest 2 findings are also seen in


this.

44/284
(Q.54) A 33 year old gentlemen presents with headaches which are frequently worse on leaning
forward. He undergoes a non-contrast CT scan of his brain which shows a dense round
mass in the midline in the region of the Foramen of Munro. On MRI, the lesion is high signal
on T1 and on T2w images. What is the likely diagnosis?

(a) Colloid cyst

(b) Epidermoid cyst

(c) Dermoid cyst

(d) Choroid plexus carcinoma

Your Response : a

Correct Answer : A

Exp: Colloid cyst

Findings on MRI are typical of colloid cyst.

It is a cyst containing gelatinous material in the brain. It is almost always found just
posterior to the foramen of monro in the anterior aspect of the 3rd ventricle originating from
the roof of the ventricle.

45/284
(Q.55) Which of the following is estimated by the Sisterhood method?

(a) Infant mortality

(b) Maternal deaths

(c) Genetic diseases among twins

(d) All of the above

Your Response : b

Correct Answer : B

Exp: Maternal deaths

The Sisterhood method is the most common household survey for estimating
maternal deaths. It is time- and cost-effective, and reduces sample size
requirements. This method includes four questions about the sisters of the woman
in question:

(i) How many sisters have you ever had, born to the same mother, who ever
reached the age 15 (or who were ever married), including those who are now dead?

(ii) How many of your sisters who reached the age 15 are alive now?

(iii) How many of these sisters are dead?

(iv) How many of your sisters who are dead died during a pregnancy or during
childbirth, or during the six weeks after the end of a pregnancy?

46/284
(Q.56) Which of the following represents the target of The END TB Strategy?

(a) 95% reduction by 2035 in number of TB deaths compared with 2015

(b) 90% reduction by 2035 in TB incidence rate compared with 2015

(c) Zero TB- affected families facing catastrophic costs due to TB by 2035

(d) All of the above

Your Response :

Correct Answer : D

Exp: All of the above

THE END TB STRATEGY

Vision : A world free of TB. Zero deaths, disease and suffering due to TB.

Goal : End the global TB epidemic.

Indicators :

(i) 95% reduction by 2035 in number of TB deaths compared with 2015

(ii) 90% reduction by 2035 in TB incidence rate compared with 2015

(iii) Zero TB-affected families facing catastrophic costs due to TB by 2035.

(Q.57) What does the given logo represent?

47/284
(a) Millennium Development
Goals

(b) Sustainable Development


Goals

(c) Biomedical Waste


Management Rules

(d) None of the above

Your Response : b

Correct Answer : B

Exp: Sustainable Development


Goals

The Sustainable
Development Goals
(SDGs), officially known
as Transforming our
world: the 2030 Agenda
for Sustainable
Development is a set of
17 “Global Goals” with
169 targets among them.
It acts as the Post-2015
Development Agenda
(successor to the
Millennium Development
Goals).

The 17 SDGs are as


follows:

Goal 1: No poverty

Goal 2: Zero hunger

Goal 3: Good health and


well-being

Goal 4: Quality education

Goal 5: Gender equality

Goal 6: Clean water and


sanitation

Goal 7: Affordable and


clean energy

Goal 8: Decent work and


economic growth

Goal 9: Industry,
Innovation and
Infrastructure

Goal 10: Reduced


inequalities

48/284
Goal 11: Sustainable
cities and communities

Goal 12: Responsible


consumption and
production

Goal 13: Climate action

Goal 14: Life below water

Goal 15: Life on land

Goal 16: Peace, justice


and strong institutions

Goal 17: Partnerships for


the goals

49/284
(Q.58) The following are the characteristics of Mediterranean diet except:

(a) Abundance of plant foods

(b) Wine in low to moderate amounts with meals

(c) Olive oil as the principal source of fat

(d) Avoidance of dairy products

Your Response :

Correct Answer : D

Exp: Avoidance of dairy products

The Mediterranean Diet is characterized by:

(i) An abundance of plant food (fruits, vegetables, bread, cereals, potatoes,


beans, nuta, seeds)

(ii) Minimally processed, seasonally fresh, locally grown foods

(iii) Desserts comprised typically of fresh fruit daily and occasional sweets
containing refined sugars or honey

(iv) Olive oil (high in polyunsaturated fat) as the principal source of fat

(v) Daily dairy products (mainly cheese and yogurt) in low to moderate amounts

(vi) Fish and poultry in low to moderate amounts

(vii) Upto 4 eggs weekly

(viii) Red meat rarely

(ix) Wine in low to moderate amounts with meals.

(Q.59) Which programme implements the 5 x 5 matrix for high impact interventions ?

(a) Rashtriya Kishore SwasthyaKaryakram

(b) NavjatShishu Suraksha Karyakram

(c) RMNCH + A

(d) Janani Shishu Suraksha Karyakram

Your Response :

Correct Answer : C

Exp: RMNCH + A

The 5x5 matrix under RMNCH+A programme includes 5 high impact interventions
50/284
under 5 thematic areas as follows:

A. Reproductive health

(i) Focus on spacing methods, particularly PPIUCD at high case load facilities

(ii) Focus on interval IUCD at all facilities including subcentres on fixed days

(iii) Home delivery of contraceptives (HDC) and Ensuring Spacing at Birth (ESB)
through ASHAs

(iv) Ensuring access to Pregnancy Testing Kits (Nischay Kits) and strengthening
comprehensive abortion care services

(v) Maintaining quality sterilization services

B. Maternal Health

(i) Use MCTS to ensure early registration of pregnancy and full ANC

(ii) Detect high risk pregnancies and line list

(iii) Equip delivery points with highly trained human resource and ensure equitable
access to EmOC services through FRUs

(iv) Review maternal, infant and child deaths for corrective actions

(v) Identify villages with low institutional deliveries and distribute misoprostol to selected
women during pregnancy

C. Newborn health

(i) Early initiation and exclusive breastfeeding

(ii) Home based newborn care through ASHA

(iii) Essential newborn care and resuscitation services at all delivery points

(iv) Special newborn care units with highly trained human resource

(v) Community level use of Gentamicin by ANM

D. Child Health

(i) Complementary feeding, IFA supplementation and focus on nutrition

(ii) Diarrhoea management using ORS and Zinc

(iii) Management of pneumonia

(iv) Full immunisation coverage

(v) RBSK- screening of children for 4 Ds (explained earlier)

E. Adolescent health

(i) Address teenage pregnancy and increase contraceptive prevalence among


adolescents

(ii) Peer educators

(iii) ARSH clinics

(iv) National Iron Plus Initiative including weekly IFA supplementation

(v) Promote menstrual hygiene

51/284
(Q.60) Sullivan’s Index is:

(a) Expectation of life free of disability

(b) Expectation of life at birth

(c) Expectation of life at 1 year of age

(d) Average life expectancy

Your Response : a

Correct Answer : A

Exp: Expectation of life free of disability

Sullivan’s Index is the number of remaining years, at a particular age, that an individual
can expect to live in a healthy state. It is calculated by the formula:

Life expectancy minus duration of disability.

(Q.61) Increase in Pauwels angle indicate

(a) Good prognosis

(b) Impaction

(c) More chances of displacement

(d) Trabecular alignment disrupted

Your Response :

Correct Answer : C

Exp: More chances of displacement

More chances of displacement are due to increased Pauwels angle.

52/284
(Q.62) A 5-year-old short child presented with normal intelligenc e. His hands showed typical
appearance. His X-ray pelvis showed the following picture. What is your diagnosis?

(a) Achondroplasia

(b) Osteogenesis imperfect

(c) Hypothyroidism

(d) Osteopetrosis

Your Response : c

Correct Answer : A

Exp: Achondroplasia.

Showing trident hand & B showing Champagne glass pelvis, are seen in Achondroplasia.

It is an autosomal dominant disease due to mutation involving FGFR3 gene

53/284
(Q.63) Patient Complaints of hoarseness, Vocal fatigue and pain in neck on prolonged
phonation. Diagnosis is:

(a) Reinke’s oedema

(b) Contact Ulcer

(c) Vocal Polyp

(d) Vocal Nodules

Your Response : d

Correct Answer : D

Exp: Vocal Nodules

They appear symmetrical on the free edge at the vocal cord, at the junction of anterior 1/3
rd and posterior 2/3 rd as this area is of maximum vibration of the cord and thus subject to
maximum trauma.

54/284
(Q.64) What is the name of this clinical finding that can be seen in Rickets?

(a) Cubitus varus

(b) Double malleoli

(c) Windswept deformity

(d) Plagiocephaly

Your Response : c

Correct Answer : C

Exp: Windswept deformity

Genu Valgum of 1 leg & genu varum of


another leg, are together
called windswept deformity: as if the legs
are sweeped off to one side because of the
speedy wind; It is one of the signs of
Rickets, involving the lower limbs of
children

55/284
(Q.65) Which of the following was the World Health Day Theme for the year 2017?

(a) Halt the rise: beat diabetes

(b) Depression: Let’s talk

(c) Food safety

(d) Good health adds life to years

Your Response : b

Correct Answer : B

Exp: Depression: Let’s talk

World Health Day Themes:

2017: Depression- Let’s talk

2016: Beat diabetes

2015: How safe is your food?

2014: Small bite. Big threat

2013: Control your blood pressure

2012: Good health adds life to years

56/284
(Q.66) The beneficiaries of Weekly Iron and Folic acid supplementation programme include:

(a) Adolescent girls and boys

(b) Women in the age group of 15 to 45 years

(c) Pregnant and lactating women

(d) All of the above

Your Response : c

Correct Answer : A

Exp: Adolescent girls and boys

The Ministry of Health and Family Welfare has launched the Weekly Iron and Folic
Acid Supplementation (WIFS) programme to meet the challenge of high prevalence
and incidence of anemia among adolescent girls and boys. It includes supervised
weekly ingestion of IFA supplementation and biannual helminthic control.

Target groups:

(i) School going adolescent girls and boys in 6 th to 12 th class enrolled in


government/ government aided/ municipal schools.

(ii) Out of school adolescent girls.

57/284
(Q.67) Which of the following states has approved the APJ Abdul Kalama Amrut Yojna scheme
to provide one full hot cooked nutritious food to pregnant, lactating women in the tribal
areas?

(a) Odisha

(b) Uttar Pradesh

(c) Tamil Nadu

(d) Maharashtra

Your Response : d

Correct Answer : D

Exp: Maharashtra

Maharashtra government has approved APJ Abdul Kalam Amrut Yojna scheme to provide
one full hot cooked nutritious food to pregnant, lactating women in the tribal areas.

Objective: To curb malnutrition among tribal children by providing nutrition to pregnant


women for 6 months when the child’s growth is at its peak.

Implementation: Anganwadis falling under Women and Child Welfare Department in 16


districts of state having tribal population will implement it.

Scheme Plan: To provide one full hot cooked nutritious food to every pregnant woman (in
third trimester) and lactating mother (first three months post- delivery) in tribal areas.

Meal: It will include pulses, fruits, vegetables, rice and boiled eggs and milk on few
occasions.

58/284
(Q.68) Most common site of osteo chondritis desiccans:

(a) Lateral part of the medial femoral condyle

(b) Medial part of the medical femoral condyle

(c) Lateral part of the lateral femoral condyle

(d) Medial part of the lateral femoral condyle

Your Response : a

Correct Answer : A

Exp: Lateral part of the medial femoral condyle

OD is a joint disorder which affects articular cartilage and underlying subchondral


bone. It usually causes pain & swelling of the affected joint. Physical examination
typically reveals an effusion, tenderness & cracking sound with joint movement.

(Q.69) A 13 year old boy presents with severe anemia and organomegaly. Peripheral smear was
performed. The X-ray of hand shows:

59/284
(a) Squaring of metacarpals

(b) Hemarthrosis

(c) Swan neck deformity

(d) Subperiosteal resorbtion

Your Response : b

Correct Answer : A

Exp: Squaring of metacarpals

X- ray of the hand in thalassemia exhibits the squaring of metacarpals due to bone marrow
expansion.

60/284
(Q.70) Staging of bone tumours is done by:

(a) Enneking

(b) Manchester

(c) Edmonton

(d) TNM

Your Response : a

Correct Answer : A

Exp: Enneking .

● The Enneking system, which addresses the unique problems related to sarcomas of
the extremities and applies to tumors of the bone as well as those of soft tissue, is
generally preferred by orthopedic oncologists.

● The Enneking system has a three-point scale for classifying tumors as stage I, II, or III
on the basis of their histologic and biologic appearance and their likelihood of
metastasizing to regional lymph nodes or distant sites such as the lung.

61/284
(Q.71) Denis model for spine stability divides the spine into how many columns :

(a) 2

(b) 3

(c) 4

(d) 5

Your Response :

Correct Answer : B

Exp: 3

62/284
(Q.72) The accompanying x-ray shows the shoulder of an 11-year-old female who fell off the
monkey bars, extending her arm in an attempt to break her fall. The small arrows indicate a
fracture in the area of the surgical neck of the humerus. The large arrows in image indicate
which of the following?

(a) A fracture at the anatomic neck of


the humerus

(b) The glenohumeral joint

(c) The joint space between the


proximal humerus and the
acromion of the scapula

(d) The proximal humeral epiphyseal


plate

Your Response :

Correct Answer : D

Exp: The proximal humeral


epiphyseal plate

● The large arrows indicate the


proximal humeral epiphyseal plate.

● The young girl was only 11


and still growing. The epiphyseal
plates show up on x-rays as radio-
lucent cartilage and should not be
confused with a fracture.

● The epiphysis is located at the


anatomic neck of the humerus but
is not discoid-shaped like many
epiphyseal plates in long bones.

● This plate is tent-shaped, that


is why it is not clearly visible all the
way across the proximal humerus.

63/284
(Q.73) 2 year old kid, suffering from neonatal hepatitis. Liver biopsy shows the below, which
was negative for Prussian blue. The most probable cause ?

(a) Hemochromatosis

(b) Dubin johnson syndrome

(c) Autophagy

(d) Necrosed cell

Your Response :

Correct Answer : C

Exp: Autophagy

● Lipofuscin. It's prussian blue negative can't be iron. In Dublin, johnson syndrome,
black pigments seen.

● Lipofuscin is a sign of cell injury seen in old cells and not old age.

64/284
(Q.74) Molecular groups of Medulloblastoma are classified on the basis of all of the following
except:

(a) SHH pathway

(b) Wnt pathway

(c) KRAS mutations

(d) N-MYC mutations

Your Response : d

Correct Answer : C

Exp: KRAS mutations

● The WNT type, characterized by mutations in the WNT signaling pathway, tends to
occur in older children, has a classic medulloblastoma histology, and shows monosomy of
chromosome 6 and nuclear expression of β-catenin. The prognosis is best in this subtype
with 90% 5-year survival.

● The SHH type, characterized by mutations involving the sonic hedgehog signaling
pathway, tends to occur in infants or young adults, tends to have a nodular desmoplastic
histology and may have MYCN amplification. The prognosis is intermediate between the
WNT subtype and groups 3 and 4.

● Group 3 medulloblastoma, often with MYC amplification and isochromosome 17


(i17q), tends to occur in infants and children, with a classic or large cell histology and the
worst prognosis.

● Group 4 is characterized by an i17q cytogenetic alteration, classic or large cell


histology, without MYC amplification, but sometimes with MYCN amplification. The
prognosis in group 4 is intermediate. In general, isochromosome 17q signals a poor
prognosis, and is restricted to groups 3 and 4.

65/284
(Q.75) Where do u see the below smear finding :

(a) G6PD Deficiency

(b) Sickle cell anemia

(c) Asplenia

(d) Thalassemia

Your Response : d

Correct Answer : A

Exp: G6PD Deficiency

Heinz bodies are seen in the


smear, These are inclusions
within RBCs composed of
denatured Hemoglobin.

66/284
(Q.76) This deformity of tibia is seen in:

(a) Congenital syphilis

(b) Rickets

(c) Scurvy

(d) Hyperparathyroidism

Your Response : b

Correct Answer : A

Exp: Congenital syphilis

Saber shin, displaying arching of the anterior edge of the tibia.

67/284
(Q.77) Identify the below neoplasm:

(a) Squamous cell carcinoma

(b) Adenocarcinoma

(c) Basal cell carcinoma

(d) Pleomorphic adenoma

Your Response : a

Correct Answer : C

Exp: Basal cell carcinoma

Basal cell carcinoma there is peripheral palisade arrangement and retraction clefts.

68/284
(Q.78) Which of the following is true about DICER enzyme?

(a) It is an exoribonuclease

(b) Cleaves ssRNA

(c) Mutations seen in Sertoli Leydig Cell Tumor

(d) Produces miRNA of 50-65 nucleotides long

Your Response :

Correct Answer : C

Exp: Mutations seen in Sertoli Leydig Cell Tumor

● Dicer, also known as endoribonuclease Dicer or helicase with RNase motif, is an


enzyme that in humans is encoded by the DICER1 gene.

● Being part of the RNase III family, Dicer cleaves double-stranded RNA (dsRNA) and
pre-microRNA (pre-miRNA) into short doublestranded.

● RNA fragments called small interfering RNA and microRNA, respectively. These
fragments are approximately 20-25 base pairs long with a two-base overhang on the 3'
end. Dicer facilitates the activation of the RNA-induced silencing complex (RISC), which is
essential for RNA interference. RISC has a catalytic component argonaute, which is an
endonuclease capable of degrading messenger.

● RNA (mRNA). SLCT is the most frequent gonadal tumour associated with DICER1
mutation, but juvenile granulosa cell tumour and gynandroblastoma can also show similar
mutations.

69/284
(Q.79) Correlate the two images and give your diagnosis ?

(a) Ollier’s disease

(b) Maffuci syndrome

(c) Hyperparathyroidism

(d) Rheumatoid arthritis

Your Response : a

Correct Answer : B

Exp: Maffuci syndrome

Multiple enchondromas in the X-ray as well as multiple cavernous hemangiomas on the


foot suggest the diagnosis of Maffuci syndrome.

70/284
(Q.80) Wilson fracture is seen in :

(a) Metatarsals

(b) Calcanium

(c) Scaphoid

(d) Volar plate fracture

Your Response : d

Correct Answer : D

Exp: Volar plate fracture

● Volar plate fractures are the result of hyperextension. This injury typically involve the
proximal interphalangeal joint of the fingers.

● At this location, the volar plate (a dense fibrous band) forms a portion of the capsule.
Typically, there is a small fragment of bone avulsed from the volar aspect of the base of
the proximal phalanx.

● If not repaired, this can lead to instability.

● A Wilson fracture refers to a volar plate injury to the middle phalanx of a finger.

71/284
(Q.81) Transition from G2 to M phase of the cell cycle is controlled by?

(a) Retinoblastoma gene product

(b) Cyclin A

(c) Cyclin E

(d) Cyclin B

Your Response : c

Correct Answer : D

Exp: Cyclin B

The lines from Robbins "Cyclin B-CDK1 is essential for the G2-to-M transition."

72/284
(Q.82) An electron microscopic (EM) picture taken of the irreversibly injured myocardium
reveals the presence of large, dark, irregular amorphic densities within mitochondria, which
are referred to as ?

(a) Apoptotic bodies

(b) Flocculent densities

(c) Myelin figures

(d) Psammoma bodies

Your Response : c

Correct Answer : B

Exp: Flocculent densities

● With prolonged ischemia, certain cellular events occur that are not reversible, even
with restoration of oxygen supply.

● These cellular changes are referred to as irreversible cellular injury.

● This type of injury is characterized by severe damage to mitochondria (vacuole


formation), extensive damage to plasma membranes and nuclei, and rupture of
lysosomes.

● Severe damage to mitochondria is characterized by the influx of calcium ions into the
mitochondria and the subsequent formation of large, flocculent densities within the
mitochondria.

● These flocculent densities are characteristically seen in irreversibly injured


myocardial cells that undergo reperfusion soon after injury.

(Q.83) Below is a histopathology section of left main coronary artery of a person who presented
with acute chest pain. Your diagnosis is?

73/284
(a) Ehlers-Danlos syndrome

(b) Familial hypercholesterolemia

(c) Kawasaki disease

(d) Marfan syndrome

Your Response : a

Correct Answer : B

Exp: Familial hypercholesterolemia

● The section shows a blood vessel.

● A round structure will help on identifying it.

● In the wall of the blood vessel there are empty spaces. (anything clear in pathology
for a UG is fat unless proved otherwise.)

● So fat in the wall of blood vessel.

● The finding is called cholesterol cleft.

● It's seen in familial hypercholesterolemia.

74/284
(Q.84) Antigen presenting cells are all of the following except:

(a) Astrocytes

(b) Endothelial cells

(c) Epithelial cells

(d) Langerhans cells

Your Response : b

Correct Answer : A

Exp: Astrocytes

An antigen-presenting cell (APC) or accessory cell is a cell that displays foreign antigens
complexed with major histocompatibility complexes (MHCs) on their surfaces; this
process is known as antigen presentation. APCs fall into two categories: professional and
nonprofessional.

Professional APCs There are three main types of professional antigen-presenting cells:

● Dendritic cells (DCs), which have the broadest range of antigen presentation, and are
probably the most important APC. Activated DCs are especially potent TH cell activators
because, as part of their composition, they express co-stimulatory molecules such as B7.
This B7 co-stimulator of mature interdigitating dendritic cell (IDC) interacts with surface
CD28 of naïve T-cell.

● Macrophages.

● Certain B-cells, which express (as B cell receptor) and secrete a specific antibody,
can internalize the antigen, which bind to its BCR and present it incorporated to MHC II
molecule, but are inefficient APC for most other antigens.

● Certain activated epithelial cells

Non-professional

A non-professional APC does not constitutively express the Major Histocompatibility


Complex class II (MHC class II) proteins required for interaction with naive T cells; these
are expressed only upon stimulation of the non-professional APC by certain cytokines
such as IFN-γ.

Non-professional APCs include:

● Fibroblasts (skin)

● Thymic epithelial cells

● Thyroid epithelial cells

● Glial cells (brain)

● Pancreatic beta cells

● Vascular endothelial cells

75/284
(Q.85) Patient came to emergency and X- ray abdomen done is shown below. Name the finding
with pathology:

(a) Bird peak sign – Achalasia

(b) Coffee Bean sign – Caecum volvulus

(c) Coffee Bean sign – Sigmoid volvulus

(d) Bird of Prey Sign - Sigmoid volvulus

Your Response : c

Correct Answer : C

Exp: Coffee bean sign - sigmoid volvulus.

Bird of Prey sign is also seen in sigmoid Volvulus but on barium study not on plain
X – ray abdomen.

76/284
(Q.86) In diffuse calcification of pancreas calcium lies in:

(a) Acinar tissues

(b) Interstitium

(c) Pancreatic ducts

(d) Any where

Your Response : b

Correct Answer : D

Exp: Any where

Pancreatic calcifications can arise form many aetiologies

Punctate intraductal calcifications

● acute alcoholic pancreatitis (20-40%) :

o intraductal, numerous, small, irregular

o preponderant cause of diffuse pancreatic intraductal calcification

● idiopathic : no underlying cause can be determined

● hyperparathyroidism (10%) : look for nephrocalcinosis or urolithiasis (70%)

● Kwashiorkor :

o calcifications are common. Seen in pediatric population

● Smaller intraductal calcifications

o senile : over 70 y/o. Scattere d. Calcifications increase with age

o cystic fibrosis : finely granular calcifications in smallest ducts in end stage disease
with pancreatic failure

● atherosclerotic calcifications

● Larger intraductal calcifications

● hereditary pancreatitis :

o autosomal dominant 9% penetrance

o large rounded shape

o peaks at 5-17 yrs

● gallstone migration

● tropical pancreatitis : young persons in tropical countries

● Dystrophic calcification

77/284
(Q.87) A young male with complaints of severe headache was brought unconscious. Declared
dead and on autopsy the following is seen. The probable cause is:

(a) Arterial wall defect due to diabetes

(b) Atherosclerotic plaque deposition

(c) Muscle weakness of the arterial wall (elastic lamina)

(d) Cystic medial necrosis

Your Response : b

Correct Answer : C

Exp: Muscle weakness of the arterial wall (elastic lamina)

Image shows berry Aneursym

The lesion results from a congenital defect in smooth muscle distribution at

a branch point of the arterial wall, defect lies in Internal elastic lamina

78/284
(Q.88) Common sites of metastases from primary lung cancer inclu de all of the following except

(a) Adrenal glands

(b) Spleen

(c) Bones

(d) Brain

Your Response : b

Correct Answer : B

Exp: Spleen

● Metastatic neoplasms of the lungs and pleura are more common than primary
neoplasms.

● Such metastases occur in about 25 percent of all fatal malignant neoplasms.

● The most common neoplasms giving rise to pulmonary or pleural metastases are
carcinoma of the breast, gastrointestinal tract, kidneys, and malignant melanoma.

● Neoplastic cells may reach the lung by direct extension, pulmonary arterial embolism,
or retrograde lymphatic extension from the mediastinal lymph nodes.

79/284
(Q.89) “Rod bodies” seen in Rod body myopathy are believed to be derived from:

(a) Motor end plate

(b) Z lines of myofibrils

(c) Sarcodermal sheath

(d) Not known

Your Response :

Correct Answer : B

Exp: Z lines of myofibrils

● Nemaline, or rod, myopathies are a group of conditions which fall under the umbrella
of congenital myopathies.

● They are characterised by rod-like structures in the muscle cells, and clinical features
such as muscle weakness, breathing problems, and feeding problems.

● There are 6 sub-groups which are defined according to age of onset and severity.
Around 1 in 50,000 individuals are estimated to be affected, and these include both males
and females.

● There is currently no effective treatment or cure to halt the progression, but


management of the condition is very important and includes physiotherapy, and where
necessary the use of ventilation and/or a feeding tube.

● Rod bodies” seen in Rod body myopathy are beloved to be derived from Z lines of
myofibrils

80/284
(Q.90) A 20 year old with history of bleeding from gums. On routine peripheral smear
examination the below finding was observed. Your diagnosis ?

(a) ALL

(b) AML

(c) CML

(d) APML

Your Response : b

Correct Answer : D

Exp: APML

● The picture shows Auer rods perfect. But the Auer rods is in bundles. The cells are
called

(Faggot cells)

● The diagnosis is APML - AML M3 - Acute promyelocytic leukaemia.

81/284
(Q.91) Name the following instrument shown below?

(a) Jobson horne probe

(b) Ear vectis and curette

(c) Periosteal elevator

(d) Jansen elevator

Your Response : a

Correct Answer : A

Exp: Jobson horne probe

Image shows a jobson horne probe with ring curette. The ring curette is used to remove
ear wax and foreign body while the other round is used to apply cotton to clean ear
discharge.

82/284
(Q.92) The prognosis of rapidly proliferating glomerulonephritis (Crescentric GN) depends
upon:

(a) Number of crescents

(b) Size of crescents

(c) Shape of crescents

(d) Cellularity of crescents

Your Response :

Correct Answer : A

Exp: Number of crescents

● Prognosis can be roughly related to the number of crescents; patients with crescents
in less than 80% of the glomeruli have a slightly better prognosis than those with higher
percentage of crescents.

83/284
(Q.93) A full-term baby boy was noted in the immediate neonatal period to fail to pass
meconium. Progressive abdominal distention was noted. Multiple laboratory and clinical
tests lead to a decision to perform a rectal biopsy. What is the most important histologic
finding that you expect to see in the rectal biopsy?

(a) Ischemic necrosis of the bowel mucosa

(b) Acute ulcerative colitis

(c) Granulomatous inflammation

(d) Absence of ganglion cells in the rectal mucosa and submucosa

Your Response : d

Correct Answer : D

Exp: Absence of ganglion cells in the rectal mucosa and submucosa

● Hirschsprung disease usually manifests in the immediate neonatal period by failure to


pass meconium, followed by obstructive constipation.Abdominal distention develops and,
in general, a large segment of the colon is involved and distended.

● The incidence of Hirschsprung disease is 1 in 5000 live births, with an 80% male
predominance in nonfamilial cases. There is no apparent difference in occurrence among
races.

● A number of abnormalities have been associated with Hirschsprung disease,


including Down syndrome (2–3% of the cases), congenital heart disease, colonic atresia,
and malrotation.

● The tissue diagnosis is made on the basis of an absence of ganglion cells in the
submucosa and the myenteric plexus on a full-thickness rectal biopsy. Some surgeons
prefer suction biopsy to full-thickness biopsy because it is easy to obtain the specimen and
they can avoid scarring and fibrosis in the area.

● The other choices are not applicable and can be ruled out on the basis of clinical
history and an extremely low incidence of other pathologic conditions at the

84/284
(Q.94) A 30 year old woman has coldness and numbness in her arms and decreased vision in
the right eye for past 5 month . On physical examination, she is afebrile. Her blood pressure
is 100/70 mm Hg . Radial pulses are not palpable but femoral pulses are . She has
decreased sensation and cyanosis in her arms but no warmth of swelling. Which of the
following is the most likely diagnosis ?

(a) Aortic dissection

(b) Kawasaki disease

(c) Microscopic polyangiitis

(d) Takayasu arteritis

Your Response : c

Correct Answer : D

Exp: Takayasu arteritis

● Takayasu arteritis leads to “pulseless disease”because of involvement of the aorta


(particularly the arct) and branches such as coronary, carotid, and renal arteries, with
granulomatous inflammation, aneurysm formation, and dissection.

● Fibrosis is a late finding, and the pulmonary arteries can also be involve d.

85/284
(Q.95) What percent of giant cell tumor of bone are malignant?

(a) 5 – 10%

(b) 15 – 20%

(c) 25 – 30%

(d) 50 – 60%

Your Response : b

Correct Answer : A

Exp: 5 – 10%

● Giant cell tumors are commonly benign.

● The tumors are malignant in 5-10% of patients

● Malignant giant cell tumors of bone usually result from secondary malignant
transformation after radiation treatment

86/284
(Q.96) A 31-year-old man is hospitalized after sustaining multiple rib fractures from a motor
vehicle accident. Forty-eight hours later, she complains to the hospital's patient relations
department that the nurses are constantly hiding in the bathroom of hospital room and
making fun of him. After extensively speaking with the nurses, the hospital administration
finds no evidence that any such behavior has occurred. His vital signs are stable. The
patient has no psychiatric history, denies smoking or recreational drugs, and has four
standard drinks daily. On the sixth day of hospitalization, the patient is no longer agitated
and does not have the same complaints. What is the most likely diagnosis?

(a) Paranoid personality disorder

(b) Brief psychotic disorder

(c) Adjustment disorder

(d) Alcoholic hallucinosis

Your Response : d

Correct Answer : D

Exp: Alcoholic hallucinosis

Auditory, tactile, and visual hallucinations are typical components of alcohol hallucinosis.
This is characterized by symptoms that frequently resolve by 24 to48 hours and does not
typically include autonomic dysregulation. Because the patient's auditory hallucinations
and agitation appeared 48 hours after admission, hallucinosis caused by alcohol
consumption is a likely explanation.

Paranoid personality disorder consists of a long-standing history of suspiciousness.


Personality disorders are unlikely to disappear within days of hospitalization. Brief
psychotic disorder is a psychosis that lasts no more than 1 month but atleast 1 day. It
consists of disorganized speech and behavior, hallucinations, and delusions and is an
unlikely diagnosis because the patient has a history of alcoholism. An adjustment disorder
typically occurs within 3 months of aspecific stressor and results in emotional turmoil that
does not include hallucinations. Posttraumatic stress disorder is frequently triggered by
natural disasters, military combat, sexual abuse, and even motor vehicle accidents. It
includes flashbacks, nightmares, and recurrent thoughts of the event but does not involve
hallucinations.

(Q.97) Identify the procedure in image:

87/284
(a) Repetitive Transcranial Magnetic Stimulation

(b) Vagal Nerve Stimulation

(c) Deep Brain Stimulation

(d) Electro Convulsive Therapy

Your Response : a

Correct Answer : A

Exp: Repetitive Transcranial Magnetic Stimulation

● Repeated transcranial magnetic stimulation (rTMS) is a noninvasive technique for


stimulating cells of the cerebral cortex.

● It creates a time-varying magnetic field in which a localized pulse magnetic field over
the surface of the head depolarizes the superficial neurons.

● TMS uses a hand-held magnet to allow focused electrical stimulation across the scalp
and cranium without the pain associated with percutaneous electrical stimulation.

● If TMS pulses are delivered repetitively and rhythmically, the technique is called
rTMS. rTMS was originally used to map cortical motor control and hemisphere
dominance.

88/284
(Q.98) A 50 year old came with complaints of weakness, routine CBC showed pancytopenia.
Bone marrow biopsy is as below. Your diagnosis:

(a) AML

(b) ALL

(c) Hairy cell leukemia

(d) Aplastic Anaemia

Your Response : d

Correct Answer : D

Exp: Aplastic Anaemia

Here you see bone marrow biopsy with fat (clear spaces) which means the marrow has
more fat instead of normal growing cells.

So it's Aplasia of marrow.

89/284
(Q.99) Late-onset schizophrenia:

(a) Is more common in men

(b) Is associated with a preponderance of paranoid symptoms

(c) Is clinically distinguishable from early-onset schizophrenia

(d) Results in poorer response to antipsychotic medications

Your Response : a

Correct Answer : B

Exp: Is associated with a preponderance of paranoid symptoms

Late-onset schizophrenia is clinically indistinguishable from early-onset schizophrenia but


has an onset after age 45 years. This condition tends to appear more frequently in women
and tends to be characterized by a predominance of paranoid symptoms. The prognosis
is favorable, and these patients usually develop on antipsychotic medication.

(Q.100) Features weighing toward a good prognosis in schizophrenia include all of the following
except:

(a) depression

(b) a family history of mood disorders

(c) paranoid features

(d) undifferentiated or disorganized features

Your Response : d

Correct Answer : D

Exp: undifferentiated or disorganized features

Poor prognostic features in schizophrenia include a family history of schizophrenia; poor


premorbid social, sexual, and work histories; and undifferentiated or disorganized
features. Features weighting toward a good prognosis in schizophrenia include mood
symptoms (especially depression), a family history of mood disorders, paranoid features,
and an undulating course.

90/284
(Q.101) Which of the following statements regarding rapid cycling bipolar disorder is true?

(a) Alcohol, stimulants, and caffeine use are risk factors.

(b) It is defined as at least four episodes per month.

(c) Hospitalization of these patients is rare.

(d) It is more common in men than women.

Your Response : c

Correct Answer : A

Exp: Alcohol, stimulants, and caffeine use are risk factors.

Among the factors favoring the occurrence of rapid cycling bipolar disorder are alcohol,
stimulant, and caffeine use. Other factors include female gender because this subtype is
much more common in women than men. Most anti depressants readily induce excited
episodes and thus aggravate the rapid cycling pattern. Rapid cycling is defined as the
occurrence of at least four episodes of depression and hypomania or mania per year (not
per month). Hospitalization of these patients is often frequent to stabilize medication and
achieve compliance.

91/284
(Q.102) Which of the following stressors most often leads to psychological impairment that
could be diagnosed as an adjustment disorder?

(a) Loss of a job

(b) A plane crash

(c) Rape

(d) All of the above

Your Response : c

Correct Answer : A

Exp: Loss of a job

By definition, an adjustment disorder is precipitated by one or more stressors. The


severity of the stressor or stressors does not always predict the severity of the disorder;
the stressor severity is a complex function of degree, quantity, duration, reversibility,
environment, and personal context. Stressors may be single, such as a divorce or the
loss of a job, or multiple, such as the death of a person important to a patient, which
coincides with the patient's own physical illness and loss of a job. Specific developmental
stages, such as beginning school, leaving home, getting married, becoming a parent,
failing to achieve occupational goals, having the last child leave home, and retiring, are
often associated with adjustment disorders.

92/284
(Q.103) Damage to which part of the brain leads to Marchiafava Bignami syndrome?

(a) A

(b) B

(c) C

(d) D

Your Response : c

Correct Answer : A

Exp: A

Marchiafava Bignami syndrome occurs due to degeneration of corpus callosum marked


as A in the image provided. It is seen in male Italian red wine drinkers and presents with
seizures, tremors, ataxia, excitement and apathy.

● B points at Basal ganglia

● C Points at Lateral ventricles

● D points at temporal lobe.

93/284
(Q.104) A Known case of HIV positive patient came to emergency with chest Infection. BAL
taken and on Methenamine silver stain following picture is seen. Identify the cause of
Infection:

(a) Mycoplasma pneumoniae

(b) Aspergillosis

(c) Pneumocystis carinii

(d) Mucor

Your Response : c

Correct Answer : C

Exp: Pneumocystis carinii

● It's ping pong


appearance.

● Other name is cup and


saucer appearance.

● The stain is GMS

(Q.105) A 10 year old male presented with swelling localised to diaphysial region tibia on Xray.
No history of trauma or enlarged lymph nodes. FNAC showed the below picture. Which of
the stains given below would help in diagnosis:

94/284
(a) PAS

(b) Leder stain

(c) Oil red O

(d) None of the above

Your Response : b

Correct Answer : A

Exp: PAS

Ewings Sarcoma. Stain - PAS.

It is a malignant small round blue cell tumour that typically has clear cytoplasm on H/E
stain due to glycogen. P/O glycogen can be demonstrated with PAS staining and
negative PAS diastase staining.

95/284
(Q.106) MC Antipsychotic drugs leading to side effect as shown in Photograph:

(a) Haloperidol

(b) Chlorpromazine

(c) Thioridazine

(d) Clozapine

Your Response : c

Correct Answer : C

Exp: Thioridazine

● Impaired vision
can occur with the
high does (over 800
mg per day)

● Can also cause


pigmentary
retinopathy, with
choriocapillary
atrophy.

96/284
(Q.107) What is the probable diagnosis, below peripheral smear:

(a) Malaria

(b) Filaria

(c) Babesia

(d) Trypnasoma

Your Response : c

Correct Answer : C

Exp: Babesia

It is malaria – like parasitic disease caused by injection with Babesia(B. microti). Babesia
parasites reproduce in RBCs, where they can be seen as cross – shaped inclusions (4
merozoite sexually budding but attached together forming a structure looking like a
Maltese cross) and cause hemolytic anemia quite similar to malaria. Treatment is
combination if Atovaquone and Azithromycin for 7 – 10 days.

(Q.108) Identify the stain used in the below pic ( and attempt the organ and probable diagnosis:

97/284
(a) PAS

(b) Alcian Blue

(c) VVG

(d) Reticulin

Your Response : d

Correct Answer : D

Exp: Reticulin.

● Reticulin is a silver stain.

● Any silver stain will highlight thin fibres or structures

● Any silver stain the end product will be BLACK (background colour varies on the
counter stain used)

● Examples of silver stain. (below going through the below list, re-collect which all
stains will be black)

● GMS (green background, but fungus appears black)

● Warthin starry (spirochetes)

● Reticulin (as seen above)

● Masson fontana (stain for melanin pigments, also a silver stain)

● Von-Kossa (silver stain, silver ions reacts with phosphates of calcium)

98/284
(Q.109) Which of the following drug is used for Uveitis in Rheumatologic diseases?

(a) Adalimumab

(b) Infliximab

(c) Etanercept

(d) Golimumab

Your Response :

Correct Answer : A

Exp: Adalimumab

Adalimumab has been successfully used for the treatment of rheumatoid arthritis,
ankylosing spondylitis, and psoriasis arthritis.

Adalimumab is a promising drug for the treatment of uveitis, even though further studies
are needed on its application as a primary therapy in uveitis

(Q.110) Cebranopadol is a novel first-in-class drug for:

(a) Alcohol withdrawal

(b) Trigeminal neuralgia

(c) Chronic back pain

(d) Opioid withdrawal

Your Response : c

Correct Answer : C

99/284
Exp: Chronic back pain

Cebranopadol

● Cebranopadol is a novel first in class compound belonging to benzenoid class


having binding affinity for all three opioid receptors as well as nociceptin receptor or
orphanin receptor-1(OPLR-1).

● Nociceptin receptor, µ receptor, δreceptor –Agonist

● К receptor – partial agonist

● The drug has strong anti-nociceptive and anti-hypertensive properties.

Uses:

● The drug has cleared phase 3 clinical trial for chronic neuropathic pain.

● It has also cleared phase 3 trials for chronic cancer and non-cancer pain.

Advantages over morphine

● It has higher efficacy than morphine.

● Tolerance is delayed (26 days vs 11days) as compared to morphine.

● It does not affect motor function and respiratory depression is minimal in therapeutic
doses.

Disadvantages

Being a kappa agonist, it has potential for abuse and can lead to dysphoria and
psychotomimesis.

(Q.111) All of the following can be examples of an orphan drug-disease in India except:

(a) Neostigmine-Myasthenia gravis

(b) Levodopa-Parkinson’s disease

(c) Itraconazole-Sporotrichosis

(d) Rivastigmine-Alzheimer’s disease

Your Response : d

Correct Answer : D

100/284
Exp: Rivastigmine-Alzheimer’s disease

● Orphan diseases are those which have a prevalence of less than 1 in 5000 in India
as per ORDI (organization for rare diseases in India).

● Drugs used for treatment of orphan diseases are called orphan drugs.

● Alzheimer’s disease, being the most common neurodegenerative disorder, is not an


orphan disease and hence the answer.

Some other definitions of orphan disease

i. As per WHO, orphan disease is defined as one with frequency less than 6.5 – 10 per
10,000 people.

ii. In the US, it is defined as any disease or condition that affects less than 200,000
persons.

Some important orphan diseases in India

● Acromegaly

● Amyotrophic lateral sclerosis

● Alpha1-antitrypsin deficiency

● Coarctation of aorta

● Ebstein’s anomaly

● Gaucher’s disease, Graves’ disease

● Myasthenia gravis

● Parkinson’s disease

● Rabies

● Sporotrichosis

● Tetralogy of Fallot, etc.

101/284
(Q.112) All of the following are used for treatment of multiple sclerosis except :

(a) Basiliximab

(b) Daclizumab

(c) Alemtuzumab

(d) Ocrelizumab

Your Response : c

Correct Answer : A

Exp: Basiliximab

Daclizumab, Alemtuzumab and Ocrelizumab are approved for patients in multiple


sclerosis.

Basiliximab is used for prophylaxis of organ transplant rejection.

Treatment of multiple sclerosis

Non-responders: Fingolimod,

Daclizumab, Alemtuzumab, Natalizumab

102/284
(Q.113) All of the following drugs inhibit the activity of IL-17 except:

(a) Ustekinumab

(b) Brodalumab

(c) Secukinumab

(d) Ixekizumab

Your Response :

Correct Answer : A

Exp: Ustekinumab

● Brodalumab, Ixekizumab and secukinumab all inhibit the activity of IL-17.

● Brodalumab is a IL-17 receptor antagonist while Ixekizumab and secukinumab bind


to IL-17 itself and inhibit its binding to receptor.

Uses

All three monoclonal antibodies are approved for treatment of moderate to severe plaque
psoriasis.

Adverse effects

i. Secondary infections like URI, nasopharyngitis

ii. Reactivation of TB

iii. PML

Ustekinumab is a monoclonal antibody against IL-12 and IL-23. It binds specifically to


p40 subunit of IL-12 and IL-23, thereby inhibiting their binding to respective receptors on
T-cells. This leads to inhibition of inflammatory response.

Uses

i. Moderate to severe plaque psoriasis

ii. Psoriatic arthritis

iii. Crohns disease

Adverse effects

i. URI, headache and tiredness

ii. Reactivation of tuberculosis

iii. Secondary cancers

iv. PML (posterior myelo- leukoencephalopathy)

103/284
(Q.114) Pethidine blocks the shivering induced by halothane general anaesthesia through:

(a) µ receptors

(b) B2 receptors

(c) α2 receptors

(d) δ receptors

Your Response :

Correct Answer : C

Exp: α2 receptors

Pethidine is the drug of choice for treatment of post-anaesthetic chills and shivering
associated with Halothane. It’s main mechanism here is exerted through α2 receptors or
к receptors, or direct action on thermoregulatory center in brain.

(Q.115) A 60 yrs old hypertensive female on amlodipine 10 mg/day presents to you with
gradually increasing breathlessness for last 1 month with difficulty to carry out her routine
daily activities. On examination, her JVP was raised, she had mild hepatomegaly and there
were bilateral basal crepts in lungs. Her Echocardiography revealed moderate distension
of ventricles with normal ejection fraction. Her LVEDP was 12 mmHg. The chest X-ray is
shown below. Drugs treatment would include all except

(a) Lisinopril

(b) Carvedilol

(c) Furosemide

(d) Nitroprusside

Your Response :

Correct Answer : D

104/284
Exp: Nitroprusside

a. This is a typical case


of Diastolic heart failure
(DHF).

b. DHF is HF with normal


ejection fraction. These
patients have abnormalities
in diastolic filling which
leads to low end diastolic
volumes and hence low
cardiac output. Hence as a
compensatory response,
ventricle undergoes
concentric hypertrophy
which leads to elevated left
ventricular end diastolic
pressure which in turn leads
to maintenance of cardiac
output but back flow into
lungs and increased
pulmonary artery occlusion
pressure and pulmonary
edema along with systemic
congestion leading to raised
JVP, hepatomegaly and
peripheral edema.

c. Therefore these
patients are critically
dependent on cardiac
preload for maintaining the
cardiac output.

d. I.v. nitroprusside being


an arterio-venous dilator will
lead to decreased cardiac
preload as a result of
venodilation which can be
detrimental in such patients.

e. Note: patients with


DHF need to maintain
LVEDP 12-15mmHg.

f. Digoxin is not indicated


for DHF patients as the
systolic contractility is
normal in such patients.

105/284
(Q.116) A pregnant female patient of HIV infection, diagnosed as WHO clinical stage 4, was
started on regimen of tenofovir, lamivudine and efavirenz. She was also diagnosed with
mycobacterium avium complex pulmonary tuberculosis. She was given a combination of
azithromycin, rifampin and ethambutol. Following were her liver enzymes after2 weeks.

(a) Patient should have been prescribed rifabutin in place of rifampin

(b) Patient should have been given ART only

(c) Patient should have been given MAC treatment only

(d) Patient should have been started on ART only and if stabilized then prescribed MAC
treatment with azithromycin and ethambutol.

Your Response : a

Correct Answer : D

Exp: Patient should have been started on ART only and if stabilized then prescribed
MAC treatment with azithromycin and ethambutol.

The management of suspected MAC infection in HIV patients involves the administration
of ART only as per NACO guidelines 2015. In most of these patients, MAC infection
resolves with commencement of ART.

Some other conditions in which ART should be commenced first are:

i. P-Papulo-pruritic eruption and seborrhoiec dermatitis

ii. P-Psoriasis

iii. E- HIV related exfoliative dermatitis

iv. S-Suspected MAC

v. C-Cryptosporidiosis

vi. M-Microsporidiosis

In case the infection does not resolve, patient should be started on MAC regimen of
clarithromycin or azithromycin and ethambutol. Rifabutin should be added as third line
agent in case of non-responders.

106/284
(Q.117) Guillian-Barre' is not linked to which of the following statements?

(a) Infections often present

(b) Greater initial upper extremity deficits

(c) Facial weakness

(d) Orthostatic Hypotension

Your Response : d

Correct Answer : B

Exp: Greater initial upper extremity deficits

● The disorder is characterized by symmetrical weakness which usually affects the


lower limbs first, and rapidly progresses in an ascending fashion. Patients generally
notice weakness in their legs, manifesting as "rubbery legs" or legs that tend to buckle,
with or without dysesthesias (numbness or tingling).

● As the weakness progresses upward, usually over periods of hours to days, the
arms and facial muscles also become affected.

● Frequently, the lower cranial nerves may be affected, leading to bulbar weakness,
(oropharyngeal dysphagia, that is difficulty with swallowing, drooling, and/or maintaining
an open airway) and respiratory difficulties.

● Facial weakness is also commonly a feature, but eye movement abnormalities are
not commonly seen in ascending GBS, but are a prominent feature in the Miller-Fisher
variant (see below.) Fever should not be present, and if it is, another cause should be
suspected.

● In severe cases of GBS, loss of autonomic function is common, manifesting as wide


fluctuations in blood pressure, orthostatic hypotension, and cardiac arrhythmias.

● Acute paralysis in Guillain-Barré syndrome may be related to sodium channel


blocking factor in the cerebrospinal fluid (CSF).

● Significant issues involving intravenous salt and water administration may occur
unpredictably in this patient group, resulting in SIADH.

● The symptoms are similar to those for progressive inflammatory neuropathy

(Q.118) Patient came to emergency with complaints of bleeding PR on colonoscopy a polyp is


seen and biopsy was taken which showed the following Pathological findings. What advice
will you give to the patient about the findings?

107/284
(a) Lesion is benign, no need to worry

(b) Lesion is Pre Malignant 40% risk of carcinoma is present

(c) Lesion is malignant, colectomy is to be done

(d) No advice is given.

Your Response :

Correct Answer : B

Exp: Lesion is Pre Malignant 40% risk of carcinoma is present

Figure is showing villous adenoma and risk of Ca in various polyps are:

1. Tubular Adenoma – 5%

2. Tubulo villous Adenoma – 20 %

3. Villous Adenoma – 40 %

108/284
(Q.119) All of the following drugs are consumed in gram quantities except:

(a) Icosapent

(b) Colesevelam

(c) Alcohol

(d) Selexipag

Your Response : c

Correct Answer : D

Exp: Selexipag

Selexipag is prostacyclin receptor agonist indicated for pulmonary arterial hypertension.


It is given in a dose of 200 µg twice daily at weekly intervals up to maximum tolerated
dose of 1600 µg twice daily.

Icosapent is EPA (eicosapentaenoic acid, isolated from fish oil). It is used to decrease
triglyceride levels in hyperlipidemic patients. It is given in dose of 0.5 g to 1.0 g daily.

Colesevelam is a bile acid sequesterant given for reducing LDL cholesterol


inhyperlipidemic patients. It is given in daily dose of 3.5-4.0 g.

Alcohol 1-2 pegs (containing 14 g per 30 ml) is often consumed for prevention of
cardiovascular disease.

109/284
(Q.120) Following are the adverse effects of Lisdexamphetamine except:

(a) Insomnia

(b) Weight gain

(c) Dry mouth

(d) Irritability

Your Response : b

Correct Answer : B

Exp: Weight gain

● Lisdexamphetamine is a prodrug of dextroamphetamine, made by conjugating


dextroamphetamine with amino acid lysine.

● It acts like amphetamine, being an indirect acting sympathomimetic, increases the


release of norepinephrine, dopamine and serotonin into the synapse by displacing these
biogenic amines from vesicles.

● The effects produced by Lisdexamphetamine are an extension of the effects of


increased NE, dopamine and serotonin in CNS. Some of these effects are restlessness,
dizziness, tremor, hyperactive reflexes, talkativeness, tenseness, irritability, weakness,
insomnia, decreased appetite, fever, and sometimes euphoria.

● Therefore amphetamine analogs lead to decreased appetite by stimulating lateral


hypothalamic center and hence weight loss.

Approved uses of Lisdexamphetamine

i. ADHD disorder in children

ii. Binge eating disorder in adults

Advantages: It is longer acting and has better oral absorption than dextroamphetamine
and the risk of abuse is also less.

110/284
(Q.121) One of the following anti-psychotic drugs is associated with least chances of cognitive
dysfunction:

(a) Olanzapine

(b) Quetiapine

(c) Thioridazine

(d) Aripiprazole

Your Response : a

Correct Answer : D

Exp: Aripiprazole

● Aripiprazole being a partial agonist of D2 and D3 receptors is the only atypical


antipsychotic that is being explored for its anti-anxiety and pro-cognitive effects.

● It’s D2 partial agonism is sometimes used in treatment of hyperprolactenemia in


psychotic patients on typical anti-psychotics.

● However the incidence of nausea is greatest with Aripiprazole due to it’s D2 partial
agonistic activity.

111/284
(Q.122) Droxidopa is:

(a) Prodrug of dopamine

(b) Prodrug of epinephrine

(c) Prodrug of serotonin

(d) Prodrug of norepinephrine

Your Response : d

Correct Answer : D

Exp: Prodrug of norepinephrine

Droxidopa is L-dihydroxyphenyserine, that is converted by dopa decarboxylase enzyme


into nor-epinephrine directly. Therefore this drug can bypass the effects of tyrosine
hydroxylase (rate limiting enzyme in nor-epinephrine synthesis). Increased
norepinephrine will activate the adrenergic receptors alpha and beta.

Uses

The drug is being approved for patients with neurogenic hypotension leading to
orthostatic hypotension due to following causes:

i. Pure autonomic failure

ii. Non-diabetic autonomic neuropathy

iii. Multiple system atrophy, familial amyloid polyneuropathy

Side effect

● Only side effect of concern is supine hypertension. Others are headache and
fatigue.

● Note: Midodrine is another drug approved for such patients.

112/284
(Q.123) Following is the order of blockade caused by local anaesthetic. The anaesthetic most likely
can be: Inhibition of urination and sweating → Loss of cold sensation → Loss of Fine
touch

→ Loss of hot sensation → Loss of deep touch → Loss of deep pressure

(a) Lignocaine 0.5% subcutaneous

(b) Lignocaine 0.25% intrathecal

(c) Bupivacaine 0.25%subcutaneous

(d) Bupivacaine 0.5% intrathecal

Your Response : c

Correct Answer : B

Exp: Lignocaine 0.25% intrathecal

Since there is inhibition of urination and sweating, therefore there is autonomic


ganglionic blockade, the chances of which are more if local anaesthetic is given
intrathecally. Now we are left with two options i.e. lignocaine 0.25% intrathecal (option B)
and bupivacaine (0.5%) intrathecal. Second higher the local anesthetic rises up the
spinal cord, more the chances of autonomic ganglionic blockade. Bupivacaine 0.5% is
more concentrated than lignocaine 0.25% and since concentrated anaesthetic rises less,
therefore lignocaine 0.25% intrathecal is answer.

Note: Order of blockade of nerve fibre is B>C>A.

113/284
(Q.124) The antidepressant associated with least chances of sexual dysfunction is :

(a) Mirtazapine

(b) Bupropion

(c) Vortioxetine

(d) Venlafaxine

Your Response : c

Correct Answer : C

Exp: Vortioxitene

Vortioxetine is the latest antidepressant approved for patients with major depressive
disorder. It is called serotonin stimulator and modulator.

Vilazodone is another new antidepressant approved for depression.

Mechanism of action of Vortioxetine:

i. Serotonin reuptake inhibitor

ii. Agonist- 5HT1A

iii. Partial agonist- 5HT1B

iv. Antagonist- 5HT1D, 5HT3 and 5HT7

Dose: 5-20 mg/day, t1/2: 66 hrs

Metabolism: mainly by CY2D6, should not be co-prescribed with CYP2D6 inhibitors

Adverse effects

i. Nausea, headache, dry mouth, dizziness and constipation.

ii. Serious adverse effects -hypertensive crisis, increased risk of suicide,


pancreatitis

iii. Serotonin syndrome with MAO inhibitors

iv. Hyponatremia due to SIADH

v. Mania/Hypomania in patients in patients of bipolar disorder

Advantage over SSRI

Lesser weight gain and sexual dysfunction

114/284
(Q.125) The antibiotic of choice for treatment of Enterobacter infection is:

(a) Ceftriaxone

(b) Meropenem

(c) Cefpodoxime

(d) Cefepime

Your Response : b

Correct Answer : B

Exp: Meropenem

Enterobacter is a gram negative facultative anaerobic organism belonging to


enterobacteriaceae family.

These are responsible for opportunistic infections like UTI and pneumonia in hospitalized
patients. These organisms are resistant to commonly used antibiotics such as penicillins
and cephalosporins.

The drug of choice for Enterobacter infection are Carbapenems such as Meropenem or
doripenem. Among cephalosporins, only fourth generation cephalosporins like Cefepime
are effective. Aminoglycosides such as amikacin can also be used.

115/284
(Q.126) All are true about starting of Beta-Blocker therapy in a case of CHF except

(a) They should be started with optimum doses

(b) They should be gradually increased over weeks.

(c) Special precautions should be taken in cases of NYHA class III and IV

(d) Carvedilol and Metoprolol are the preferred drugs.

Your Response : b

Correct Answer : A

Exp: They should be started with optimum doses

Beta blockers have traditionally been contraindicated in heart failure. Since the
publication of a hypothesis in 2000 that pulses of norepinephrine can lead to cardiac
arrhythmias leading to death, it has been realized that low dose of beta blockers should
be used.

In recent years, it has been shown that three beta blockers, e.g., metoprolol, bisoprolol
or carvedelol have reduced mortality in chronic heart failure.

However, American college of clinical cardiology and American heart association


guidelines:

● Low dose of beta blockers should be used

● They should be used only in NYHAI and NYHA II (i.e. in mild and moderate cases)

● Carvedilol or metoprolol can be used

● Carvedilol is preferred due to its antioxidant activity

● No beta blocker is used to treat severe CHF

116/284
(Q.127) This lesion is seen in?

(a) Primary

(b) Secondary

(c) Tertiary

(d) Congenital syphilis

Your Response : c

Correct Answer : D

Exp: Congenital syphilis

Hutchinson’s teeth, showing notched central incisors.

117/284
(Q.128) A 53 yr old lady on immunoblotting shows Ig G against desmoglein 3. She is likely to
present with?

(a) Oral erosions

(b) Tense bullae

(c) Grouped itchy excoriated papules on elbows and knees

(d) Periorificial erosions

Your Response : b

Correct Answer : A

Exp: Oral erosions

● IgG against desmoglein 3 is seen in pemphigus vulgaris.

● Almost 90-95% of patients have mucosal involvement while nearly 70% of them
present with oral erosions.

● Flaccid blisters on skin usually develop late or simultaneously with oral lesions.

(Q.129) A 70 yr old male patient was brought to the emergency department at 4.00 a.m. with
cardiac arrest. The patient was treated immediately as per CPR protocol and immediate
ECG was done. The findings of ECG are shown below. The appropriate anti-arrhythmic
drug for the patient would be

118/284
(a) Lignocaine

(b) Quinidine

(c) Propafenone

(d) Adenosine

Your Response : d

Correct Answer : A

119/284
Exp: Lignocaine

ECG is characteristic of ventricular fibrillation, the most common underlying ECG


abnormality inpatients with cardiac arrest.

Treatment of Cardiac Arrest

1. CPR protocol (5 cycles of chest compression and 2 breaths -2 minute (30 second
cycle)

2. Administration of defibrillating shock 200 J

3. Another cycle of CPR

4. Second Shock

5. Another cycle of CPR

6. Administration of epinephrine i.v. 1:1000 (0.5 mg)

7. Continuing CPR

8. i.v. amiodarone or i.v. lignocaine

Note: Results have shown higher chances of cardioversion with amiodarone than
lignocaine.

Atropine can be used in cases of cardiac arrest after epinephrine in cases with PEA
(Pulseless electrical activity) or asystole.

Lignocaine is a class Ib anti-arrythmic drug acting mainly by prolongation of inactivated


state of Na+ channels. This leads to inhibition of action potential propagation and hence
decreased conduction.

Lignocaine is given i.v.. Other drugs in same class are mexilitine and tocainide (both
given orally)

Main adverse effect of lignocaine is Neurotoxicity (tremors, convulsions, psychosis)

(Q.130) Based on the following histogram of a automated counter, tell the most probable type of
anaemia.

120/284
(a) Microcytic anaemia

(b) Aplastic anaemia

(c) Megaloblastic anaemia

(d) Normocytic anaemia

Your Response :

Correct Answer : A

Exp: Microcytic anaemia.

There is a hint of 250fl on one corner. Even if it's not there each bar is for 50 fl. as
correctly pointed out the peak is close to 50fl. The diagnosis is microcytic anaemia

121/284
(Q.131) Heck's disease is caused by ?

(a) HPV 6,11

(b) HPV 3,10

(c) HPV 16,18

(d) HPV 13,32

Your Response : d

Correct Answer : D

Exp: HPV 13,32

Heck's disease (also known as focal or multifocal epithelial hyperplasia) is an


asymptomatic, benign neoplastic condition characterized by multiple white to pinkish
papules that occur diffusely in the oral cavity.Can present with slightly pale, smooth or
roughened surface morphology. It is caused by the human papilloma virus types 13 and
32. It exhibits surface cells with vacuolated cytoplasm around irregular, pyknotic nuclei
and occasional cells with mitosis-like changes within otherwise mature and well-
differentiated epithelium. A distinguishing histologic feature is elongated rete ridges
resembling Bronze Age axe with mitosoid bodies present.

122/284
(Q.132) A 28 year old male has multiple grouped papulovesicles on both elbows, knees,
buttocks, upper back associate with severe itching. The most likely diagnosis is:

(a) Pemphigus vulgaris

(b) Dermatitis Herpetiformis

(c) Bullous pemphigoid

(d) Herpes zoster

Your Response : b

Correct Answer : B

Exp: Dermatitis Herpetiformis

This is classical clinical picture of Dermatitis herpetiformis. An uncommon skin condition


with the peak incidence around 30-40 years of age (M>F). Again, DH has nothing to do
with Herpetic infection. Pathogenesis: disease usually associates with celiac disease
(Gluten sensitive enteropathy). And many other antibodies are found for eg. Anti-
endomysial,anti transglutaminase (? Pathogenic) etc. and they deposit at or around BMZ
leading to defects at these sites with resulting clefting.

Clinical Features

● Intense itching

● Grouped paulovesicles which are usually excoriated and scabbed and needs to be
differentiated from Scabies and acute atopic dermatitis.

● The sites of predilection are elbows and knees, buttocks, upper back and face.

● Intact vesicles seldom seen since they are ruptured by self scratching.

● Lesions heal with no scarring.

Investigations

● Histology: Neutrophilic Microabscess in dermal papillary tip is characteristic.

● DIF: It shows granular IgA deposition in BMZ.

● Screening for the presence of celiac disease.

● Provocation test: iodides in diet or patch.

Treatment

● DAPSONE: It is the DOC. After starting the treatment, symptoms should subside
within 48 hours and rash should be cleared up within days.

● Sulfapyridine/sulphamethoxypyridazine: when dapsone is contraindicated.

● Gluten Free diet: It is useful but Dietary restriction is not easy to follow.

123/284
(Q.133) Melanin can be stained by?

(a) Levaditi stains

(b) Giemsa stain

(c) Masson-Fontana stain

(d) haematoxylin & eosin stain

Your Response : c

Correct Answer : C

Exp: Masson-Fontana stain

Levaditi is for spirochetes (treponematoses,borreliosis) colour black.

124/284
(Q.134) A 15-year-old boy presents with an unexplained, painless but dramatic swelling of the
upper lip. The likely diagnosis is:

(a) Cellulitis

(b) Hereditary angioedema (HAE)

(c) Contact dermatitis

(d) Angioedema

Your Response : d

Correct Answer : D

Exp: Angioedema

● The lip is one of the more common areas involved in angioedema, but many others
have been reported, including palms and soles, where it tends to be slightly tender to
touch. Whatever the location, angioedema usually resolves within 48 h, if not much
sooner.

● Cellulitis is commonly misdiagnosed, and is incorrect in this case, since it, unlike
angioedema, would be red, hot, and tender, and would probably not resolve within 48 h,
even with treatment.

● Hereditary angioedema (HAE) does present with angioedema, but is rare, so is not
the likely diagnosis.

● However, possible family history of unexplained respiratory distress needs to be


investigated to help rule out HAE.

125/284
(Q.135) A newborn child born of a non-consanguineous marriage presents with history of
recurrent blistering of hands and feet. His father also gives history of recurrent blisters of
hands, feet and trauma prone sites. The diagnosis is:

(a) Epidemolysis bullosa acquisita

(b) Epidermolysis bullosa congenita

(c) Porphyria cutanea tarda

(d) Bullous lupus erythematosus

Your Response : b

Correct Answer : B

Exp: Epidermolysis bullosa congenita

● This child has a genetically inherited bullous disease primarily affecting the trauma
prone sites like hands and feet.

● It is therefore a mechanobullous disease suggestive of defective structural proteins


from birth, unlike the EB acquisita which is immune mediated damage to type-7 collagen.

● The condition is calle epidermolysis bullosa congenital which is of further 3 types:


simplex

● The most superficial due to defective keratin 5 &14, junctionalis:due to defective


proteins like laminin, plectin, integrins etc. and dystrophic: defective defective collagen 7.

● The level of split in these three are through the basal layer, through the lamina lucida
and below the lamina densa

126/284
(Q.136) A 19-year-old white male presents with complaints of itching and burning of the feet and
maceration of the skin between the toes. Which of the following historical statements
would be most helpful to you in developing your differential diagnosis ?

(a) History of exposure to Rhus plant species

(b) A family member with the same symptoms

(c) Daily use of dark-colored socks

(d) Participation in school athletics with daily practice sessions

Your Response : c

Correct Answer : D

Exp: Participation in school athletics with daily practice sessions

● One of the most common sources of tinea pedis infection is the floors of athletic
locker rooms and showers. Patients in this age group frequently get this infection from
that source.

● The warm, moist environment promotes tinea growth. Dark socks tend to inhibit
resolution of tinea infections, but do not cause them.

● Rhus contact causes a localized reaction of the skin known as rhus dermatitis or
poison ivy, oak, and sumac.

● Symptoms include red skin, itching, and vesicle development.

127/284
(Q.137) Patient with pre-existing acne lesions was treated with long term
antibiotics(tetracyclines). Developed papulo-pustules or nodules around nose and lips.
Diagnosis?

(a) Morbihan disease

(b) Chloracne

(c) Gram negative folliculitis

(d) Rosacea

Your Response :

Correct Answer : C

Exp: Gram negative folliculitis.

It occurs in pts with pre-existing acne who are treated with long term antibiotics esp.
tetracyclines. There are papulo-pustules or nodules around nose. Culture of these
reveals Enterobacter, Klebsiella or Escherichia in pustules or Proteus in nodules.

(Q.138) A patient of moderate hypertension was started on an anti-hypertensive ‘X’ with


hydroclorothiazide12.5 mg twice a day and propranolol 100 mg twice a day. Following was
the chest X-ray of patient after 1 month of drug ‘X’. The drug ‘X’ could have been

128/284
(a) Sodium Nitroprusside

(b) Clonidine

(c) Hydralazine

(d) Minoxidil

Your Response :

Correct Answer : D

129/284
Exp: Minoxidil

The x-ray shows massive pericardial effusion which is caused by minoxidil.

Minoxidil is a directly acting vasodilator which acts by increasing the levels of cAMP in
blood vessels (arteries only). It is indicated for treatment of hypertensive patients with
decreased renal blood flow who do not respond to hydralazine. It is also indicated for
hypertensive pregnant females.

Minoxidil is associated with maximum incidence of sodium and water retention and reflex
tachycardia among all directly acting vasodilators. Hence it is always given in
combination with diuretic (hydrochlorithiazide) and beta blockers (to control reflex
tachycardia).

Adverse effects:

i. Sodium and water retention

ii. Reflex tachycardia

iii. Peripheral edema

iv. Increased chances of angina, CHF and MI.

v. T –wave abnormalities

vi. Rare adverse effect: Pericarditis and pericardial effusion with or without cardiac
tamponade. Chances of pericardial effusion are considerably increased in patients with
renal failure and CHF.

Mechanism of pericardial effusion: Pericardial effusion in minoxidil-treated patients


has been attributed to uremia, as well as the salt and water retention, a consequence of
the vasodilatory effect of drug.

130/284
(Q.139) A benign tumor in the pterygoid canal would spare which of the following nerve fibres:

(a) Postganglionic parasympathetic fibres

(b) Taste fibres from palate

(c) Post ganglionic sympathetic fibres

(d) General visceral afferent (GVA) fibres

Your Response : c

Correct Answer : D

Exp: General visceral afferent (GVA) fibres

The greater petrosal nerve passes anteriorly, receives a branch from the tympanic
plexus and traverses a hiatus on the anterior surface of the petrous part of the temporal
bone. It enters the middle cranial fossa, runs forwards in a groove on the bone above the
lesser petrosal nerve and then passes beneath the trigeminal ganglion to reach the
foramen lacerum. Here it is joined by the deep petrosal nerve from the internal carotid
sympathetic plexus, to become the nerve of the pterygoid canal (Vidian nerve). The
greater petrosal nerve contains parasympathetic fibres destined for the pterygopalatine
ganglion, and taste fibres from the palate

(Q.140) A 5 year old girl with a lesion for 3 months. Histopathology as shown in image given.
Diagnosis?

131/284
(a) Squamous cell carcinoma with a central crater

(b) Molluscum contagiosum

(c) Polyp

(d) Keratoacanthoma

Your Response : a

Correct Answer : B

Exp: Molluscum contagiosum

Keratoacanthoma (KA) is a relatively common low-grade tumor that originates in the


pilosebaceous glands and closely resembles squamous cell carcinoma.

Keratoacanthomas (KAs) are composed of singularly well-differentiated squamous


epithelium that form masses of keratin that constitute the central core of
keratoacanthoma.

Keratoacanthoma

132/284
H&E Section showing
exophytic of hyperplastic
stratified squamous
epithelium with central
crater containing keratin
plug & CT, showing
Infiltration of chronic
inflammatory cells.

● HOLMES- STD
(Layer wise histological
changes seen in
MOLLUSCUM
CONTAGIOSUM )

● In the basal layer, the


nuclei and cytoplasm of
the keratinocytes are
enlarged and there is an
increase in the mitotic
rate.

● In the spindle layer, as a result of viral replication in the cytoplasm, cells begin to
display cytoplasmic vacuolization, enlargement, and then replacement by eosinophilic
compartmentalized globules, the molluscum bodies, which are contained in well-defined
sacs and which compress the nuclei to the cell periphery.

● In the granular layer, the molluscum bodies become more homogeneous with loss of
their internal structural markings and are finally desquamated into the cystic lobules.

● Dermal changes are usually limited to stromal proliferation.

● These inclusions, the Henderson-Paterson bodies( can measure 35µm in diameter)


are membrane-bound sacs that contain numerous molluscum contagiosum virions. The
viral particles increase in size as they progress up toward the granular layer, causing
compression of the nucleus to the periphery of the infected keratinocytes. The
surrounding dermis is relatively unremarkable.

● HP Bodies are intracytoplasmic, eosinophilic, granular inclusions within the


keratinocytes of the basal, spinous, and granular layers of the epidermis.

(Q.141) In a Subclavian artery block at the outer border of 1 st rib, all of the following arteries
help in maintaining the circulation to upper limb except-

(a) Subscapular artery

(b) Superior thoracic

(c) Thyrocervical trunk

(d) Suprascapular artery

Your Response :

Correct Answer : B

133/284
Exp: Superior thoracic

Anastomoses around the body of the scapula

a. The suprascapular artery, a branch of the thyrocervical trunk

b. The deep branch of the transverse cervical artery, another branch of the thyrocervical
trunk

c. The circumflex scapular artery, a branch of the subscapular artery which arises from
the third part of the axillary artery.

Anastomoses over the acromion process

a. The acromial branch of the thoracoacromial artery

b. The acromial branch of the suprascapular artery

c. The acromial branch of the posterior circumflex humeral artery

Note: This is an anastomoses between the first part of the subclavian artery and the third
part of the axillary artery

134/284
(Q.142) True about ovary:

(a) Covered anteriorly by broad ligament

(b) Connected to uterus by round ligament

(c) Anterior border not covered by peritoneum

(d) Lies above psoas major

Your Response :

Correct Answer : C

Exp: Anterior border not covered with peritoneum.

The anterior border faces the posterior leaf of the broad ligament and contains the
mesovarium. Each ovary is almost entirely covered by the peritoneum except along the
mesovarian border where two layers of the covering peritoneum are reflected on the
posterior layer of the broad ligament.

135/284
(Q.143) An injection to anesthetize pain from a frature of the seventh rib should be given in
what area?

(a) Seventh intercostals space immediately below the seventh rib in the midclavicular line

(b) Seventh intercostals space immediately below the seventh rib just lateral to the angle of
the rib

(c) Seventh intercostals space immediately below the seventh rib just medial to the angel of
the rib

(d) Sixth intercostals space immediately above the seventh rib in the midclavicular line

Your Response :

Correct Answer : B

Exp: Seventh intercostals space immediately below the seventh rib just lateral to the
angle of the rib

At the angle of the rib, where it is the most superficial and easiest. The patient is placed
in the lateral position with the side up if performing a unilateral block or in prone position
inserted through a skin wheal at the lower edge of the posterior angle of the rib.

(Q.144) Double inferior vena cava is formed due to-

(a) Persistence of sacrocardinal veins

(b) Persistance of supracardinal veins

(c) Persistance of subcardinal veins

(d) Persistance of both supracardinal and subcardianl veins

Your Response :

Correct Answer : A

Exp: persistence of sacrocardinal vein

136/284
A. Double inferior vena cava at the lumbar level arising from the persistence of the left
sacrocardinal vein.

B. Absent inferior vena cava. The lower half of the body is drained by the azygos vein,
which enters the superior vena cava.

The hepatic vein enters the heart at the site of the inferior vena cava.

A. Left superior vena cava draining into the right atrium by way of the coronary sinus
(dorsal view).

B. Double superior vena cava.

The communicating (brachiocephalic) vein between the two anterior cardinals has failed
to develop (dorsal view).

137/284
(Q.145) False about clavicle-

(a) Ossifies into membrane

(b) Horizontal bone

(c) No medullary cavity

(d) Most common site of fracture is the junction of medial 1/3rd with lateral 2/3rd

Your Response : d

Correct Answer : D

Exp: Most common site of fracture is the junction of medial 1/3rd with lateral 2/3rd

The peculiar features of the clavicle are as follows:

1. It is the only long bone which lies horizontally.

2. It has no medullary cavity.

3. It is subcutaneous throughout its extent.

4. It is the first bone to start ossifying (between the fifth and sixth week of intrauterine life)
and last bone to complete its ossification (at 25 years).

5. It is the only long bone which ossifies by two primary centers.

6. It is the only long bone which ossifies in membrane except for its medial end (cf. long
bones ossify in cartilage).

7. It may be pierced through and through by cutaneous nerve (intermediate


supraclavicular nerve).

It commonly fractures at the junction of its lateral one – third and medial two –third.

(Q.146) A and B marked structure in the given picture are tendons of:

138/284
(a) A- Flexor pollicis longus, B- Abductor pollicis longus

(b) A- Flexor carpi radialis, B- Flexor Pollicis longus

(c) A- Flexor pollicis longus, B- Flexor carpi radialis

(d) A- Abductor pollicis longus, B- Extensor pollicis brevis

Your Response : c

Correct Answer : C

Exp: A- Flexor pollicis longus, B- Flexor carpi radialis

139/284
(Q.147) A 66- year-old man noticed progressive bluish skin discolouration, predominantly in
sun-exposed areas as shown in figure. The patient had a history of sustained ventricular
tachycardia and renal transplant. Which of the following could be responsible for such a
presentation?

(a) Cyclosporine therapy for


Immunosuppression

(b) development of SLE

(c) Amiodarone for Arrhythmia

(d) Age related photosensitivity

Your Response : a

Correct Answer : C

Exp: Amiodarone for Arrhythmia

● Although photosensitivity
reactions due to amiodarone are
common, bluish-grey skin
discolouration is an uncommon
adverse effect of amiodarone,
occurring in less than 3% of patients
undergoing chronic therapy.

● Amiodarone may accelerate


normal cellular phagocytosis, leading
to increased production of lipofuscin,
which accumulates in lysosomes.
Phototoxic damage explains the
distribution of the discolouration in
sun-exposed areas

● It appears to be dose related


response, and the effect is reversible
if the total daily dose is reduced to
200 mg per day or less, with
adequate sun protection.

140/284
(Q.148) A patient with epileptic disorder presented with concentric visual field defects. Which
drug was he on?

(a) Valproate

(b) Ethosuximide

(c) Phenobarbital

(d) Vigabatrin

Your Response : a

Correct Answer : D

Exp: Vigabatrin

a. GABA-Transaminase inhibitor

b. Used in infantile spasm (ONLY IN REFRACTORY CASES)

i. DOC-ACTH

ii. S/e- Spinal cord edema, visual hallucinations

1. Infantile spasm (DOC)

2. With tuberous sclerosis: Vigabatrin

3. Without tuberous sclerosis: ACTH

(Q.149) Which of the following is incorrect regarding marked areas in the given image?

141/284
(a) A provides insertion to muscles for external rotation at shoulder

(b) B gives passage to suprascapular artery

(c) C Provides passage to the branch of anterior circumflex humeral artery

(d) D gives origin to muscle supplied by radial nerve

Your Response : c

Correct Answer : B

Exp: B gives passage to suprascapular artery

Suprascapular foramen gives passage to suprascapular nerve, suprascapular artery


passes above the transverse scapular foremen.

142/284
(Q.150) Which of the following is not true?

(a) Superior thyroid artery is a branch of external carotid artery

(b) Inferior thyroid artery is a branch of thyrocervical trunk

(c) Parathyroid artery is a branch of posterior division of superior thyroid artery

(d) Thyreoidea ima artery is invariably a branch of aortic arch

Your Response : c

Correct Answer : C

Exp: Parathyroid artery is a branch of posterior division of superior thyroid artery

Both superior and interior parathyroid glands are usually supplied by the inferior thyroid
arteries; the superior thyroid may be supplied by the superior thyroid artery or from
anastomoses between the superior and inferior thyroid arteries in 10-15% cases.

(Q.151) In a young female of reproductive age with regular menstrual cycles of 28 days
ovulation occurs around 14 th day of periods. When is the first polar body extruded?

(a) 24 hrs prior to ovulation

(b) Accompanied by ovulation

(c) 48 hrs after the ovulation

(d) At the time of fertilization

Your Response : b

Correct Answer : A

Exp: 24 hrs prior to ovulation

It is prior to ovulation even if 24 hours is not the exact time but still it is prior to ovulation
only so from the option given below i definitely will go with A.

143/284
(Q.152) During the third week of human embryo development-

(a) Neural plate formed by stimulation from notochord

(b) Amniotic disc forms

(c) Body stalk moves ventrally to form umbilical cord

(d) Primitive streak formation

Your Response :

Correct Answer : D

Exp: Primitive streak formation

The most characteristic event occurring during the third week of gestation is gastrulation,
the process that establishes all three germ layers (ectoderm, mesoderm, and endoderm)
in the embryo. Gastrulation begins with formation of the primitive streak on the surface of
the epiblast

144/284
(Q.153) True about Pheochromocytoma:

(a) Paraxysmal HT

(b) Sutained HT

(c) Sustained HT with postural hypotention

(d) Paroxymal HT with postural hypotention

Your Response : c

Correct Answer : D

Exp: Paroxymal HT with postural hypotention

Pheochromocytoma

Clinical Features

1. Hypertension

a. M.C. manifestation of pheochromocytoma

b. 60% of the patients have sustained hypertension

c. 40% have paroxysmal hypertension

2. Arrhythmia, Sinus tachycardia, or Sinus bradycardia

3. Angina and myocardial infarction even in the absence of coronary artery disease.

4. Profuse sweating

5. Mild to moderate Weight loss

6. Impaired carbohydrate tolerance

7. Orthostatic (Postural)hypotension

8. Hypercalcemia, Abdominal pain

9. Increase hematocrit

(Q.154) A 62 year old female patient presents with progressive right-sided weakness and
145/284
(Q.154) A 62 year old female patient presents with progressive right-sided weakness and
speech difficulty since one-month. Fundus examination showed the condition as
represented in the picture below. Two months ago, she had a history of fall in her bathroom
and struck her head against a wall. The most likely clinical diagnosis is?

(a) Alzheimer’s disease.

(b) Left parietal glioma.

(c) Left MCA territory stroke.

(d) Left chronic subdural


haematoma.

Your Response : c

Correct Answer : D

Exp: Left chronic subdural


haematoma.

Fundus Examination shows


Papilloedema.

The presence of progressive


hemiparesis and aphasias
together with signs of raised
ICT (papilloedema) as shown
in the picture above in an
elderly patient, developing
weeks after sustaining a mild
head injury is characteristic
ofchronic subdural
hematoma.

Papilledema

● It is optic disc swelling


that is caused by increased
intracranial pressure.

● The swelling is usually


bilateral and can occur over a
period of hours to weeks.

● Unilateral papilledema
can suggest a disease in the
eye itself, such as an optic
nerve glioma.

Symptoms:

● It may be asymptomatic
or present with headache in
the early stages.

● It may progress to
enlargement of the blind spot,
blurring of vision, visual
obscurations (inability to see
in a particular part of the
visual field for a period of
time) and ultimately total loss
of vision may occur.
146/284
● Signs of papilledema that
are seen using an
ophthalmoscope include:

o venous engorgement
(usually the first signs)

o loss of venous pulsation

o hemorrhages over and /


or adjacent to the optic disc

o blurring of optic margins

o elevation of optic disc

o Paton’s lines or radial


retinal lines cascading from
the optic disc.

● When papilledema is
found on fundoscopy, further
evaluation with a CT or MRI
of the brain and/or spine is
warranted as vision loss can
result if the underlying
condition is not treated.

Subdural hematoma

● It is a collection of blood
below the inner layer of the
dura but external to the brain
and arachnoid membrane .

● Generally, acute
subdural hematomas are less
than 72 hours old and are
hyperdense compared with
the brain on computed
tomography scans.

● The subacute phase


begins 3-7 days after acute
injury.

● Chronic subdural
hematomas develop over the
course of weeks and are
hypodense compared with
the brain. It can occur in the
elderly after apparently
insignificant head trauma.

Presentation:

● Symptoms and signs of


raised ICP including
headache, vomiting without
nausea, ocular palsies,
altered level of
consciousness, back pain
and Papilledema.

● Other symptoms include


147/284
Aphasia, Hemiparesis,
Difficulty with gait or balance,
Cognitive dysfunction or
memory loss , Personality
change ,etc

(Q.155) A patient on an antiarrhythmic medication, developed rapid onset of bradycardia,


hypotension and seizures. ECG shows the following features. Which of the following
medication will be given in the treatment of this condition?

(a) Quinidine

(b) Sodium Bicarbonate

(c) Propranolol

(d) Magnesium Sulfate

Your Response : a

Correct Answer : B

Exp: Sodium bicarbonate.

ECG shows:

● Widening of QRS complex in Lead II,and Prominent R wave in lead aVR.

● It is suggestive of Toxicity of Sodium Channel Blocker.

Sodium channel blockers

● Inhibition of the fast Na+ channels, in the phase 0 of the action potential (AP),
decreases the rate of rise and amplitude of the AP in Purkinje fibers, and in atrial and
ventricular myocardial cells. As a result, the upslope of depolarization is slowed and the
QRS complex becomes wide.
148/284
Inhibitors of fast Na+ channels

● Tricyclic antidepressants (= most common)

● Type Ia antiarrhythmics (quinidine, procainamide)

● Type Ic antiarrhythmics (flecainide, encainide)

● Local anaesthetics (bupivacaine, ropivacaine)

● Antimalarials (chloroquine, hydroxychloroquine)

● Dextropropoxyphene

● Propranolol

● Carbamazepine

● Quinine

Clinical Features of Sodium channel blocker Overdose:

● Sedation and coma

● Seizures

● Hypotension

● Tachycardia

● Broad complex dysrhythmias

● Anticholinergic syndrome.

● Patients with tricyclic overdose will also usually demonstrate sinus tachycardia
secondary to muscarinic (M1) receptor blockade.

ECG Features consistent with sodium-channel blockade are:

● Interventricular conduction delay — QRS > 100 ms in lead II

● Right axis deviation of the terminal QRS:

● Terminal R wave > 3 mm in aVR

● R/S ratio > 0.7 in aVR

Management of overdose:

● Secure IV access, adminster high flow oxygen .

● Administer IV sodium bicarbonate 100 mEq (1-2 mEq / kg); repeat every few
minutes until BP improves and QRS complexes begin to narrow.

● Intubate as soon as possible.

● Hyperventilate .

● Once the airway is secure, place a nasogastric tube and give 50g (1g/kg) of
activated charcoal.

● Treat seizures with IV benzodiazepines

● Treat hypotension with a crystalloid bolus (10-20 mL/kg). If this is unsuccessful in


restoring BP then consider starting vasopressors (e.g. noradrenaline infusion)

149/284
(Q.156) A patient with a history of intravenous drug abuse presented with a high grade fever.
Chest Xray shows the following features. What can be the most probable diagnosis?

(a) Right sided Pneumonia.

(b) Bronchiectasis.

(c) Right sided Septic Embolus.

(d) Right sided Pleural Effusion.

Your Response : c

Correct Answer : C

Exp: Right sided Septic Embolus.

Chest X ray in the image


shows:

● There is a small irregular


opacity on the right

● This opacity contains a


dark area - cavity(marked by
an arrow).

There is a history of
intravenous drug abuse, fever
and lung showing cavity. This
leads to a probable diagnosis
of septic embolus.

Differential diagnosis of lung


cavities

● Lung abscess - TB,


Klebsiella or Staph aureus

● Lung cancer

● Septic embolus - infected


thrombus

● Fungal infection - if
immunocompromised

● Granulomatosis with
polyangiitis

150/284
(Q.157) Aliskiren is a:

(a) ACEI

(b) Renin antagonist

(c) Endothelin receptor antagonist

(d) Diuretic

Your Response : b

Correct Answer : B

Exp: Renin antagonist

It is direct Renin Inhibitor.

It is used for essential HTN.

Cis are – Pregnancy, breastfeeding, preexisting Diabetes, Kidney or heart disease.

(Q.158) Most accurate test to diagnose Cushing disease is:

(a) MRI of the Brain

(b) High Dose Dexamethasone Suppression Test

(c) Inferior Petrosal Sinus Sample

(d) 24 hour urinary cortisol

Your Response : d

Correct Answer : C

Exp: Inferior Petrosal Sinus Sample

Cushing disease is Cushing syndrome due to Pituitary adenoma which secretes ACTH .

Inferior Petrosal Sinus Sample is the most accurate method for diagnosing Cushing
Disease.

151/284
(Q.159) In Addison’s disease, most diagnostic test is:

(a) Serum Na+, K +, rennin

(b) Serum Na+, K +, saline suppression

(c) Serum creatinine/urea ratio

(d) ACTH stimulation test

Your Response : d

Correct Answer : D

Exp: ACTH stimulation test

Low plasma cortisol (<3 µg/dl) at 8:00 am is diagnostic especially if accompanied by


simultaneous elevation of plasma ACTH level (>200 pg/ml)

(Q.160) Shy Dragger syndrome is Park insonism + ____________?

(a) Motor Neuropathy

(b) Sensory Neuropathy

(c) Cranial Neuropathy

(d) Autonomic Neuropathy

Your Response :

Correct Answer : D

Exp: Autonomic Neuropathy

It is a rare neurodegenerative disorder characterized by tremors, slow movement,


muscle rigidity, & postural instability (Parkinsonism) due to dysfunction of the ANS &
ataxia. It is caused by degeneration of neurons in SN, Striatum, Inj. Olivary & cerebellum.

152/284
(Q.161) A head injury patient opens his eye after loud voice, he is confused disoriente d. He
withdraws his arms to pain, What is his Glasgow coma scale?

(a) 9

(b) 10

(c) 11

(d) 12

Your Response : d

Correct Answer : C

Exp: 11

153/284
(Q.162) Given is the horizontal section through the floor of the left orbit (view from above).
Which of the marked option is INCORRECT:

(a) A- orbital plate of frontal bone

(b) B- orbital plate of maxillary


bone

(c) C- orbital plate of ethmoid


bone

(d) D- orbital process of palatine


bone

Your Response :

Correct Answer : A

Exp: A- orbital plate of frontal


bone

A (its Lacrimal bone and not


frontal)

A horizontal section through


the floor of the left orbit and the
adjacent ethmoidal and
sphenoidal sinuses, viewed
from above.

154/284
(Q.163) Choose the incorrect option regarding marked structures in the given image :

(a) A- Origin of superior constrictor

(b) B- Origin of Tensor Veli Palatini

(c) C- Origin of medial pterygoid

(d) D- Origin of Tensor tympani

Your Response :

Correct Answer : D

Exp: D- Origin of Tensor tympani

155/284
(Q.164) Cilostazol is used for which condition?

(a) Periphereal vascular disease

(b) Coronary artery disease

(c) AF

(d) VT

Your Response : a

Correct Answer : A

Exp: Periphereal vascular disease

It is a quinolinone derivative used in symptoms of intermittent claudication in peripheral


vascular disease.

(Q.165) Which drug acts on PPAR gamma receptors?

(a) Losartan

(b) Telmisartan

(c) Perindopril

(d) None of the above

Your Response : b

Correct Answer : B

Exp: Telmisartan

Telmisartan is a selective modulator of PPAR –y a central regulator of Insulin & glucose


metabolism.

156/284
(Q.166) Earliest feature of diabetic nephropathy :

(a) Enlarge kidney size

(b) Raise GFR

(c) Raise serum K

(d) Microalbuminuria

Your Response :

Correct Answer : B

Exp: Raise GFR

Diabetes Nephropathy

a. ↑GFR is the 1 st manifestation of diabetic nephropathy.

b. Then is the stage of Microalbuminuria (30 to 300 mg/day of albumin in urine) .

c. It is the most reliable marker of diabetic nephropathy .

d. Macro albuminuria (> 300 mg/day of albumin in urine)

e. CRF (diabetes is the commonest cause of CRF).

157/284
(Q.167) What does [B] indicate in the following partograph indicators ?

(a) Mild uterine contractions

(b) Moderate uterine contractions

(c) Strong uterine contractions

(d) Arrest of labour

Your Response :

Correct Answer : B

Exp: Moderate uterine contractions

The partograph is a graphical presentation


of the progress of labour, and of fetal and
maternal condition during labour. It is the
best tool to help you detect whether labour
is progressing normally or abnormally, and
to warn you as soon as possible if there are
signs of fetal distress or if the mothers vital
signs deviate from the normal range.
Research studies have shown that maternal
and fetal complications due to prolonged
labour were less common when the
progress of labour was monitored by the
birth attendant using a partograph.

Different shadings on the squares drawn on


partograph indicate the strength & duration
of uterine contractions:

158/284
(Q.168) The prerequisites for external cephalic version rotation of the head are all expect:

(a) Well-flexed head

(b) Singleton pregnancy

(c) Favourable shape of the pelvis

(d) Intact membranes

Your Response : d

Correct Answer : C

Exp: Favourable shape of the pelvis

External cephalic version rotation is done for singleton pregnancy and these are the
requirements for doing ecv:

● Intact membranes

● Not in active labor

● Liquor adequate

● FHR is normal

● No contraindication for vaginal delivery

● Gestational age more than 36 weeks

● Previous CS, preeclampsia and iugr are relative contraindications

159/284
(Q.169) Which of the following is the capacity of bakri balloon catheter, used in cases of severe
PPH?

(a) 350 ml

(b) 500 ml

(c) 800 ml

(d) 1000 ml

Your Response : b

Correct Answer : B

Exp: 500 ml

● The Bakri Balloon

● It is a medical device invented by Dr.YounesBakri.

● The obstetrical balloon is a 24 French, 54 cm-long, silicone catheter with a filling


capacity of 500 mL.

● The device is used for the temporary control and reduction of postpartum
hemorrhage (PPH).

160/284
(Q.170) Which is not used in management of stage III ovarian cancer:

(a) Debulking

(b) Abdomino-pelvic radiotherapy is very effective

(c) Chemotherapy

(d) Cytoreduction

Your Response : c

Correct Answer : B

Exp: Abdomino-pelvic radiotherapy is very effective

Management of Advanced stage disease (stages III and IV) in ovarian cancer:

● Advanced stage disease: Exploratory Laparotomy —» Cytoreductive or debulking


surgery. This includes: Total abdominal hysterectomy bilateral salpingo-oophorectomy,
complete omentectomy, retroperitoneal lymph node sampling and resection of any
metastatic tumor.

● Optimum cytoreductive surgery is aimed to reduce the residual tumor load < 1-2 cm
in diameter. Lesser the residual tumor volume (< 1 cm), better is the survival.

● Chemotherapy: Chemotherapy is used widely following surgery to improve the result


in terms of survival. Drugs are given for five or six cycles at 3-4 weekly interval.

● Combination chemotherapy: Paclitaxel (175 mg/m2) and carboplatin (400 mg/m2)


are commonly used.

● Neoadjuvant chemotherapy and interval cytoreductive surgery: Few cycles of


chemotherapy followed by interval primary cytoreductive surgery may be done.
Indications are: (i) Advanced epithelial ovarian cancer, (ii) High risk for surgery, (iii)
Associated comorbid conditions (pleural effusion), (iv) Predicted to be suboptimally
resected. Patient should have histological diagnosis of the tumor (biopsy).

● Benefits of neoadjuvant chemotherapy are: (i) Rapid clinical improvement, (ii)


Subsequent surgery is easier and morbidity is reduced, (iii) Optimum cytoreduction with
minimal residual disease may be possible.

● Radiotherapy: There is very little scope of radiotherapy as an adjunct to surgery


because of the advent of chemotherapy.

161/284
(Q.171) Which of the following drug is used in the treatment of diabetic retinopathy?

(a) Eculizumab

(b) Adalimumab

(c) Etanercept

(d) Bevacizumab

Your Response :

Correct Answer : D

Exp: Bevacizumab

1. Proliferative RP can be treated with intraocular injection of a vascular endothelial


growth factor antagonist.

2. Bevacizumab or ranibizumab is administered by direct injection into the vitreous


cavity.

3. These antibodies cause the regression of neovascular membranes by blocking the


action of vascular endothelial growth factor, thereby improving visual acuity. (VGEF
Inhibitor)

162/284
(Q.172) Which of the following marked structure will continue as content of Posterior
Mediastinum?

(a) A only

(b) B only

(c) C only

(d) Both A & C

Your Response :

Correct Answer : D

Exp: Both A & C

A. is vagus (posterior
mediastinum)

B. is Phrenic (middle
mediastinum)

C. is sympathetic trunk
((posterior mediastinum)

D. is roots of brachial plexus


(none)

(Q.173) A patient of Parkinson disease presents with purplish, mottled changes to skin as
shown in the picture. Which drug is most likely responsible for this skin reaction:

163/284
(a) Amantadine

(b) Entecapone

(c) Benserzide

(d) Selegilline

Your Response : d

Correct Answer : A

Exp: Amantadine

• Amantidine was originally introduced as an antiviral agent, it also has


antiparkinsonian effect

• Mostly used as an antidyskinesis agent in patients with advanced PD.

• Livedo reticularis (mottled, lace-like purplish discoloration of the skin) and edema of
ankles are side effects.

Remember

● Amantadine : livedo reticularis

● Minoxidil : Hypertricosis

● Chloramphenicol: Grey baby syndrome.

● Vancomycin: red man syndrome

● Amiodarone: Blue man Syndrome

164/284
(Q.174) Most specific eye sign seen in Grave‘s disease?

(a) Wide staring appearance

(b) Upper Lid retraction

(c) Diplopia

(d) All of the above

Your Response : a

Correct Answer : C

Exp: Diplopia

Following Eye signs are seen only in Grave disease.

a. Chemosis'

b. Exophthalmos

c. Corneal ulceration

d. Ophthalmoplegia,

e. Papilledema

f. Diplopia (diplopia results, typically when patient looks up and laterally. inferior
rectus is most commonly involved)

(Proptosis occur due to Glycosaminoglycan deposit behind the eyes)

165/284
(Q.175) Most Confirmatory investigation for Acromegaly is:

(a) Insulin induced GH suppression

(b) Oral Glucose Tolerance Test

(c) Random GH assay

(d) IGF - I level

Your Response : b

Correct Answer : B

Exp: Oral Glucose Tolerance Test

Test

a. Increase heal pad thickness seen on X-ray lateral view of the heal

b. Elevated IGF-I. Usually > 5 times normal.

c. Serum GH not suppressed following oral glucose. OGTT test (75 gms of
glucose given . After 1 hour if GH>1 ng/ml- suggestive of GH tumor.) It is the most
confirmatory test.

d. Confirmation for tumor- MRI/CT neoplasm

166/284
(Q.176) Which is the following is the JAK 1 Receptor antagonist?

(a) Anakinra

(b) Rituximab

(c) Tofacitinib

(d) Abatacept

Your Response : a

Correct Answer : C

Exp: Tofacitinib

Inhibitor of JAK1 & JAK3, used in treatment of Rheumatoid arthritis.

(Q.177) Tocilizumab is:

(a) TNF - α antagonist

(b) IL -6 antagonist

(c) Fusion protein

(d) Immuno supresive

Your Response : b

Correct Answer : B

Exp: IL -6 antagonist

It is IL -6R antagonist used in RA, systemic JIA, castle man’s disease,


Neuromyelitis optica, Giant cell arteritis, cytokine release syndrome.

167/284
(Q.178) MCHC more than normal is seen in which condition?

(a) Thalessmia

(b) SSA

(c) HS

(d) PNH

Your Response :

Correct Answer : C

Exp: HS

Investigations

a. Presence of hemolytic state MCV ↓, MCHC ↑

b. Increased osmotic fragility of RBCs:

c. Presence of spherocytes in the blood film

(Q.179) A patient suffers from TIA & suffers from unilateral hemiparesis lasting for 12 hrs. His
age is 62 yrs. His BP is 142/100. He is non diabetic. What is his ABCD2 score?

(a) 4

(b) 5

(c) 6

(d) 7

Your Response : a

Correct Answer : C

168/284
Exp: 6

THE ABCD2 SCORE

169/284
(Q.180) The given image is a feature of:

(a) Blunt trauma

(b) Posterior uveitis

(c) Anterior uveitis

(d) Intermediate uveitis

Your Response : a

Correct Answer : C

Exp: Anterior uveitis

Anterior uveitis/iridocyclitis
causing posterior synechiae
resulting in festooned pupil

170/284
(Q.181) A 60 year old male, chronic smoker presented with increasing shortness of breath.
Chest X ray shows the following features. What can be the most probable diagnosis?

(a) Left sided Lung collapse

(b) COPD

(c) Left sided Pleural Effusion

(d) Left sided Pneumothorax

Your Response : b

Correct Answer : B

Exp: COPD.

Chest Xray in the image


shows:

● Asymmetrical lower
zones

● Left darker than right

● Lung hyperexpansion.

Probable diagnosis is

● Chronic obstructive
pulmonary disease with a
large left lower zone lung
bulla.

171/284
(Q.182) Which reflex is being elicited in the patient?

(a) Hoffman reflex

(b) Wartenberg reflex

(c) Finger flexion reflex

(d) Supinator jerk

Your Response : b

Correct Answer : B

Exp: Wartenberg reflex

● The image shows the patient’s hand with fingers in flexion and thumb in abduction.
The examiner is pulling against resistance. This is called Wartenberg sign and is taken as
equivalent of Balbinski sign in case of amputation of both lower limbs.

● Under normal circumstances the thumb of the patient extends and terminal phalanx
may flex slightly.

● In UMNL, the thumb adducts and flexes strongly.

172/284
(Q.183) This clinical finding is a result of:

(a) Anterior uveitis

(b) Blunt trauma

(c) Perforating injury

(d) Sympathetic ophthalmia

Your Response :

Correct Answer : B

Exp: Blunt trauma

Image shows Vossius ring which is a circular deposit of pigment as a complete or


incomplete ring on the anterior lens capsule surface that was resulted from a blunt
trauma.

173/284
(Q.184) The medial wall of orbit does not include:

(a) Body of sphenoid

(b) Frontal process of maxilla

(c) The anterior and posterior ethmoidal canals

(d) Lesser wing of sphenoid

Your Response : d

Correct Answer : D

Exp: Lesser wing of sphenoid

Orbit

The orbit is a pear shaped cavity, the stalk of which is the optic canal.

- The roof consists of two bones : lesser wing of the sphenoid and the orbital plate of
the frontal

- The lateral wall also consists of two bones : greater wing of the sphenoid and
zygomatic.

- The floor consists of three bones : zygomatic, maxillary, and patatine.

- The medial wall consists of four bones : frontal process of maxilla, lacrimal,
ethmoid and body of sphenoid.

- The superior orbital fissure is a slit between the greater and lesser wings of the
sphenoid bone.

174/284
(Q.185) A 8 year male boy was brought to eye OPD with complain of red eye. On examination
patient had bilateral involvement, at interval of about 4 weeks, with circumciliary
congestion; ground glass haziness of cornea and poorly visible details of iris. Pupil was
sluggishly reactive. Systemic examination shows cervical lymphadenopathy and frontal
eminence prominent. Most likely clinical diagnosis is -

(a) Anterior uveitis

(b) Keratoconjunctivitis

(c) Interstitial keratitis

(d) Intermediate uveitis with keratitis

Your Response : c

Correct Answer : C

Exp: Interstitial keratitis

Child is suffering from interstitial keratitis; mostly of inherited syphilitic origin, which
typically affect age group of 5-15 years and

present bilaterally although asymmetrically, with circumciliary congestion and


ground glass appearance of cornea.

175/284
(Q.186) ‘Fortification spectra’ are feature of?

(a) Mature cataract

(b) Immature cataract

(c) Amaurosis Fugax

(d) Migraine

Your Response : d

Correct Answer : D

Exp: Migraine

a. In a typical attack, a small central disturbance in the field of vision marches toward
the periphery, leaving a transient scotoma in its wake.

b. The expanding border of migraine scotoma has a scintillating, dancing, or zig-zag


edge, resembling the bastions of a fortified city, hence the term fortification spectra.

c. Patients’ descriptions of fortification spectra vary widely and can be confused with
amaurosis fugax.

d. Migraine patterns usually last longer and are perceived in both eyes, whereas
amaurosis fugax is briefer and occurs in only one eye.

e. Migraine phenomena also remain visible in the dark or with the eyes closed

176/284
(Q.187) In visual pathway, neurons of first order are :

(a) Pigmented epithelium

(b) Rods & cones

(c) Bipolar cells

(d) Ganglion cells

Your Response :

Correct Answer : C

Exp: Bipolar cells

Bipolar cells are 1 st order neurons terminating on ganglionic cells. Ganglion cells axon
constitute optic nerve and are 2nd order neurons, running up to LGB. 3rd order neuron
arises in LGB to occipital cortex.

(Q.188) True regarding Amaurosis fugax ?

(a) Refers to a transient ischemic attack of the retina

(b) Patients describe a rapid fading of vision like a curtain descending, sometimes affecting
only a portion of the visual field

(c) The most common source is an atherosclerotic plaque in the carotid artery or aorta

(d) All of above

Your Response : d

Correct Answer : D

177/284
Exp: All of above

Amaurosis Fugax

This term refers to a transient ischemic attack of the retina . Because neural tissue has a
high rate of metabolism, interruption of blood flow to the retina for more than a few
seconds results in transient monocular blindness, a term used interchangeably with
amaurosis fugax. Patients describe a rapid fading of vision like a curtain descending,
sometimes affecting only a portion of the visual field. Amaurosis fugax usually results
from an embolus that becomes stuck within a retinal arteriole. If the embolus breaks up
or passes, flow is restored and vision returns quickly to normal without permanent
damage. With prolonged interruption of blood flow, the inner retina suffers infarction.
Ophthalmoscopy reveals zones of whitened, edematous retina following the distribution
of branch retinal arterioles. Complete occlusion of the central retinal artery produces
arrest of blood flow and a milky retina with a cherry-red fovea. Emboli are composed of
cholesterol (Hollenhorst plaque), calcium, or platelet-fibrin debris. The most common
source is an atherosclerotic plaque in the carotid artery or aorta, although emboli also
can arise from the heart, especially in patients with diseased valves, atrial fibrillation, or
wall motion abnormalities.

In rare instances, amaurosis fugax results from low central retinal artery perfusion
pressure in a patient with a critical stenosis of the ipsilateral carotid artery and poor
collateral flow via the circle of Willis. In this situation, amaurosis fugax develops when
there is a dip in systemic blood pressure or a slight worsening of the carotid stenosis.
Sometimes there is contralateral motor or sensory loss, indicating concomitant
hemispheric cerebral ischemia.

Retinal arterial occlusion also occurs rarely in association with retinal migraine, lupus
erythematosus, anticardiolipin antibodies (Fig. 28-6), anticoagulant deficiency states
(protein S, protein C, and antithrombin deficiency), pregnancy, IV drug abuse, blood
dyscrasias, dysproteinemias, and temporal arteritis.

Marked systemic hypertension causes sclerosis of retinal arterioles, splinter


hemorrhages, focal infarcts of the nerve fiber layer (cotton-wool spots), and leakage of
lipid and fluid (hard exudate) into the macula . In hypertensive crisis, sudden visual loss
can result from vasospasm of retinal arterioles and retinal ischemia. In addition, acute
hypertension may produce visual loss from ischemic swelling of the optic disc. Patients
with acute hypertensive retinopathy should be treated by lowering the blood pressure.
However, the blood pressure should not be reduced precipitously, because there is a
danger of optic disc infarction from sudden hypoperfusion.

Impending branch or central retinal vein occlusion can produce prolonged visual
obscurations that resemble those described by patients with amaurosis fugax. The veins
appear engorged and phlebitic, with numerous retinal hemorrhages. In some patients
venous blood flow recovers spontaneously, whereas others evolve a frank obstruction
with extensive retinal bleeding ("blood and thunder" appearance), infarction, and visual
loss. Venous occlusion of the retina is often idiopathic, but hypertension, diabetes, and
glaucoma are prominent risk factors. Polycythemia, thrombocythemia, or other factors
leading to an underlying hypercoagulable state should be corrected; aspirin treatment
may be beneficial.

178/284
(Q.189) Which of the following is associated with ipsilateral oculomotor palsy with contralateral
cerebellar ataxia?

(a) Nothnagel’s syndrome

(b) Benedikt’s syndrome

(c) Weber’s syndrome

(d) Claude’s syndrome

Your Response :

Correct Answer : A

Exp: Nothnagel’s syndrome

a. In Nothnagel’s syndrome, injury to the superior cerebellar peduncle causes


ipsilateral oculomotor palsy and contralateral cerebellar ataxia.

b. In Benedikt’s syndrome, injury to the red nucleus results in ipsilateral oculomotor


palsy and contralateral tremor, chorea, and athetosis.

c. Claude’s syndrome incorporates features of both the aforementioned syndromes, by


injury to both the red nucleus and the superior cerebellar peduncle.

d. In Weber’s syndrome, injury to the cerebral peduncle causes ipsilateral oculomotor


palsy with contralateral hemiparesis.

179/284
(Q.190) Name the cell marked as X in Pap smear.

(a) Superficial cells

(b) Intermediate cells

(c) Para-basal cells

(d) Basal cells

Your Response : d

Correct Answer : A

Exp: Superficial cells

Squamous epithelial cell found in normal combined fast smears

180/284
(Q.191) The earliest morphological evidence of ovulation on endometrial biopsy is:

(a) Pseudostratification

(b) Basal vacuolation

(c) Decrease in glycogen content

(d) Predecidual reaction

Your Response : c

Correct Answer : B

Exp: Basal vacuolation

● Endometrial biopsy was used in the past to find out whether the female has ovulated
or not. Nowadays USG follicular monitoring is being done

● Subnuclear basal vacuolation is characterized by glandular growth and presence of


vacuoles due to secretion of glycogen between nuclei and basement membrane. It is due
to the effect of progesterone. Basal vacuolization is the earliest evidence ot ovulation
(36-48 hours after ovulation) and persists until 21st day of the cycle.

● Pseudostratification of nuclei is characteristic of proliferation phase but persists until


active progesterone secretion begins. Hence, it is noted until 18th -19th day of the
menstrual cycle.

● Predecidual reaction is first evident on day 23 of the menstrual cycle.

181/284
(Q.192) Subfertility is defined as the failure to conceive within :

(a) 3 months of unprotected regular sexual intercourse

(b) 6 months of unprotected regular sexual intercourse

(c) 1 year of unprotected regular sexual intercourse

(d) 2 year of unprotected regular sexual intercourse

Your Response : c

Correct Answer : C

Exp: 1 year of unprotected regular sexual intercourse

• Subfertility is defined as the failure to conceive within 1 year of unprotected regular


sexual intercourse.

• For couples who have had no previous conception, the subfertility is defined as primary,
while couples who have had a previous conception and have then not conceived again
are defined as having secondary subfertility.

182/284
(Q.193) Treatment modality for Stage II endometrial carcinoma?

(a) Surgery only

(b) Radiotherapy

(c) Surgery followed by radiotherapy

(d) Debulking surgery

Your Response : a

Correct Answer : C

Exp: Surgery followed by radiotherapy

Endometrial carcinoma management depends on stage at the time of admission.

183/284
(Q.194) Major sources of androgen in females are all except:

(a) Adrenals

(b) Ovaries

(c) Peripheral conversion to androgen precursors in the liver, gastro- intestinal tract and
adipose tissue

(d) Corpus Luteum

Your Response : d

Correct Answer : D

Exp: Corpus Luteum

● In females 50% of testosterone comes from peripheral conversion of


androstenedione

● The remaining 50% is equally divided between adrenal gland and ovary

● While the products from corpus luteum are

o Progesterone

o Estrogen

o Relaxin

o Inhibin A

184/284
(Q.195) Semen variables less than the reference value for morphology are :

(a) Normo zoospermia

(b) Oligozoospermia

(c) Asthen zoospermia

(d) Teratozoospermia

Your Response : b

Correct Answer : D

Exp: Teratozoospermia

Nomenclature for some semen variables

• Normo zoospermia: normal ejaculate as defined by the reference value.

• Oligozoospermia: sperm concentration less than the reference value .

• Asthen zoospermia: less than the reference value for motility.

• Teratozoospermia: less than the reference value for morphology.

• Azoospermia: no spermatozoa in the ejaculate.

185/284
(Q.196) All are true about Krukenberg’s tumour except:

(a) It is always secondary

(b) Most common primary is stomach

(c) Usually bilaterality

(d) Signet ring cells are seen

Your Response : d

Correct Answer : A

Exp: It is always secondary

• Krukenberg tumor is generally a metastatic tumor to the ovary. But "krukenberg tumor
may be a primary tumor"

• The most common primary sites from where metastases to the ovaries occur are
gastrointestinal tract (pylorus, colon and rarely small intestine), gallbladder, pancreas,
breast and endometrial carcinoma.

• These are usually bilateral tumors which man taint shape of the ovary. Histologically
'signet ring' looking cells are characteristic of krukenberg tumor.

• In most patients with Krukenberg's tumors, the prognosis is poor. Median survival being
lew than a year Rarefy, no primary site can be identified and the Krukenberg's tumor may
be a primary tumor Also know: Metastatic tumors from the GI tract can be associated
with sex hormone (estrogen and androgen) production turn Patient may present with
postmenopausal bleeding.

(Q.197) All of the following are true about this contraceptive device except?

186/284
(a) It is reusable

(b) It should be removed


after 30 minutes of
intercourse

(c) A spermicide jelly should


be used along with it

(d) It does not protect against


STDs

Your Response : b

Correct Answer : B

Exp: It should be removed


after 30 minutes of
intercourse

The diaphragm should


remain in situ for 6 hours
after the intercourse.

It is popular reusable
method of birth control that
is used to prevent
pregnancy. Made of
silicone rubber, a
diaphragm is shaped like a
shallow cup and is inserted
by the woman into the
vagina to cover the cervix
before sexual intercourse.
A spermicide jelly is spread
around the rim of the
diaphragm before it is
inserted. After intercourse
the device is left in place
for 6 hours and then
removed, washed and
stored until next use.

Although the diaphragm


will act as a barrier,
preventing sperm from
entering womb through the
cervix - it is not particularly
effective as a mechanical
barrier alone - sperm can
still enter around the
edges. Rather it keeps
spermicide in place over
the cervix so that sperm
are killed before they can
travel any further. It should
be mentioned that unlike
condoms or female
Condoms, diaphragms do
not reduce your risk of
catching a sexually
transmitted infection (STI).
187/284
(Q.198) Latanoprost used topically in glaucoma primarily acts by:

(a) Decreasing aqueous humor formation

(b) Increasing uveo scleral outflow

(c) Releasing papillary block

(d) Increasing trabecular outflow

Your Response : a

Correct Answer : B

Exp: Increasing uveo scleral outflow

Latanoprost - (0.0005%) synthetic drug which is an ester analogue of prostaglandin F2


alpha, It is only drug which exclusively acts by increasing uveo scleral outflow and by
causing reduction in episcleral venous pressure

Side effects - conjunctival hyperaemia, foreign body sensation and increased


pigmentation of the iris.

188/284
(Q.199) A 37-year-old woman presents to the office for a consultation about family planning. She
is married for 11 years and she has never had any extra-marital relationships. They have
been tested negative for HIV infection. She hopes to become pregnant and to stop using
condoms. She had normal Pap smears for the past 18 years. The last Pap smear was 1 year
ago. When does she need to have her next Pap smear?

(a) During this visit

(b) In 6 months

(c) In 1 year

(d) In 2 years

Your Response : c

Correct Answer : D

Exp: In 2 years

This patient is at low risk of cervical cancer. She is in a monogamous relationship for 11
years and she had many negative Pap smears along the past 18 years. For these
reasons, it is appropriate to lengthen the interval for Pap smear tests to every 3 years.
So, her next Pap smear should be in 2 years as her last one was 1 year ago. The correct
answer is D

(Q.200) Gland homologous to prostate in females is?

(a) Gartner's gland

(b) Skene's gland

(c) Bartholin's gland

(d) Cowper's gland

Your Response : b

Correct Answer : B

Exp: Skene's gland

● Bartholin gland in females-homologous to bulbourethral gland/cowper gland in


males.

● Skenes gland in females-homologous to prostate gland in males.

189/284
(Q.201) Amount of liquor is maximum at:

(a) 32-34 weeks

(b) 36-38 weeks

(c) 34-36 weeks

(d) 38-40 weeks

Your Response : a

Correct Answer : A

Exp: 32-34 weeks

Amount of liquor is maximum at 32-34 weeks Amniotic fluid index (AFI) is a quantitative
estimate of amniotic fluid and an indicator of fetal well-being. It is a part of the biophysical
profile.

● AFI is the score (expressed in cm) given to the amount of amniotic fluid seen on
ultrasonography of a pregnant uterus. To determine the AFI, doctors may use a four-
quadrant technique, when the deepest, unobstructed, vertical length of each pocket of
fluid is measured in each quadrant and then added up to the others, or the so-called
"Single Deepest Pocket" technique.

● An AFI between 8-18 is considered normal. Median AFI level is approximately 14


from week 20 to week 35, when the amniotic fluid begins to reduce in preparation for
birth.

● An AFI < 5-6 is considered as oligohydramnios. The exact number can vary by
gestational age. The fifth percentile for gestational age is sometimes used as a cutoff
value.

● An AFI > 20-24 is considered as polyhydramnios

190/284
(Q.202) The following manoeuvre is called:

(a) Pinard

(b) Ritzen

(c) Loveset

(d) Mauriceau – Smellie- Veit

Your Response : d

Correct Answer : D

Exp: Mauriceau-Smellie-Veit

The image shows Mauriceau-Smellie-Veit menoeuvre used for delivery of after-coming


head in breech delivery.

191/284
(Q.203) A 32-year old diabetic woman delivered a large baby after difficult vaginal delivery. At
labor, the head emerged first and then the shoulders were stuck behind the bony pelvis.
The obstetrician had to use some tools and maneuvers to deliver the baby. The baby had a
good cry and red pink color but his right arm was not moving. It was hanging on his side
with medial rotation. Which of these nerve roots are most likely affected?

(a) C5 and C6

(b) C6 and C7

(c) C7 and C8

(d) C8 and T1

Your Response : c

Correct Answer : A

Exp: C5 and C6

Shoulder dystocia is a condition characterized by difficulty to deliver shoulder of the baby


after delivery of the head. There are some maneuvers used by the obstetrician to deliver
it safely. Some complications could happen during manual delivery of the shoulder.
Brachial plexus injury, fetal hypoxia and even fetal death are complications of these
maneuvers. This case presentation represents Erb’s palsy which caused by injury to C5
and C6. The correct answer is A

192/284
(Q.204) The following tube is best utilized for:

(a) Eye surgery

(b) Oral surgery

(c) Neurosurgery

(d) Thoracic surgery

Your Response : d

Correct Answer : B

Exp: Oral surgery

RAE preformed Endotracheal tube are designed to conveniently position the anaesthesia
circuit out of the surgical field for oral & maxillofacial procedures.

193/284
(Q.205) Labyrinthine artery is a branch of:

(a) Internal carotid artery

(b) Basilar artery

(c) Posterior cerebellar artery

(d) Anteroinferior cerebellar artery

Your Response : b

Correct Answer : D

Exp: Anteroinferior cerebellar artery

Labyrinthine artery is a branch of anteroinferior cerebellar artery but can


sometimes arise from basilar artery. • It supplies whole of the inner ear

194/284
(Q.206) True about multiple papillomatosis:

(a) HSV is causative agent

(b) Radiotherapy treatment of choice

(c) It is premalignant

(d) It is more common in 15 to 33 yrs

Your Response : c

Correct Answer : C

Exp: It is Premalignant

● It is caused by infection with human papilloma virus.(HPV) subtype 6 and 11 not by


Herpes simplex virus i.e.HSV is not the causative agent

● The primary treatment modality for respiratorypapillomatosis is surgery

● Juvenile papillomatosis due to subtype 11,16,18 canundergo malignant


transformations, though it is rare.

● Respiratory papillomatosisis m/c seen in childrenbetween the ages 2 to 5 years


although it can be seenin adults in third decade also.

195/284
(Q.207) Which of the following tests on maternal serum is most useful in distinguishing between
open neural tube defects and ventral wall defects in a fetus?

(a) Carinoembryogenic antigen

(b) Sphingomyelin

(c) Alpha-feto protein

(d) Pseudocholinesterase.

Your Response : b

Correct Answer : D

Exp: Pseudocholinesterase.

● Pseudocholinesterase measurement on maternal serum is most useful in


distinguishing between open neural tube defects and ventral wall defects in a fetus

(Q.208) The intra-abdominal pressure during laparoscopy should be set between:

(a) 5-8 mm Hg

(b) 10-15 mmHg

(c) 20-25 mmHg

(d) 30-35 mm Hg

Your Response : b

Correct Answer : B

Exp: 10-15 mmHg

● During laparoscopy, pneumoperitoneum is created with CO2 or nitrous oxide.

● CO2 is preferred because N2O can cause explosion in presence of volatile


anesthetic drugs.

● About 2 litres of gas is introduced at 10 mg Hg the intraabdominal pressure during


any laparoscopic surgery should be 10-15 mmHg.

196/284
(Q.209) A 17 year-old girl visits the clinic with her mother who is concerned that her daughter
had not menstruated yet and her breast had not been significantly developed. She also
notes that her daughter is color blind. The girl denies any sexual relationship or using any
recreational drugs. Examination reveals normally appearing girl with small non-developed
breasts. There is lack of pubic hair and her vagina and cervix are not fully developed. She
is 5f 5in tall and weighs 59 kg. Cardiac, chest and abdominal examinations are
unremarkable.

What is the most likely initial diagnosis?

(a) Androgen insensitivity disorder

(b) Early pregnancy

(c) Kallmann’s syndrome

(d) Turner's syndrome

Your Response : a

Correct Answer : C

Exp: Kallmann’s syndrome

This patient has primary amenorrhea, delayed secondary sexual development and color
blindness. All these features are characteristic of Kallmann’s syndrome. It is a hereditary
disorder of GnRH synthesis. Androgen sensitivity disorder is characterized by the
development of breasts and female external genitalia in an XY candidate. This patient
has primary amenorrhea and under-developed secondary sexual characteristics which
exclude pregnancy. Turner’s syndrome is characterized by amenorrhea and lack of
secondary sexual characteristics. It is excluded in this case because it causes
dysmorphic body shape and short stature which are not available in this case. The
correct answer is C

197/284
(Q.210) Which of the following is true about vasa previa except:

(a) Incidence is 1: 1500

(b) Mortality rate of 20% with undiagnosed case

(c) Associated with low lying placenta

(d) Cesarean section is indicated

Your Response :

Correct Answer : B

Exp: Mortality rate of 20% with undiagnosed case

● Vasa previa – It is a condition in which the fetal blood vessels unsupported by either
umbical cord or placental tissue, Overlies the internal os and is vulnerable to rupture
when supporting membrane rupture.

● Thus bleeding in case of vasa previa is of fetal origin and not maternal origin (unlike
placenta previa and abruptio)

● It is rare condition and occurs in 1 in 2000 - 3000 deliveries (i.e. option ais correct).

● Vasa previa should be suspected if any of the following condition exists

● Velamentous cord insertion

● Bilobed placenta

● Succenturiate lobed placenta

● Placenta previa /low lying placenta in second trimester (option 'c' is correct)

● Pregnancy resulting from IVF

● Multiple pregnancies

● Diagnosis of vasa previa - In all cases of antepartum and intrapartum hemorrhage,


the possibility of vasa previa should be kept in mind and blood should be tested for fetal
hemoglobin characterized by resistance to denaturation by alkaline reagent (Singer alkali

denaturation test/Apt test )

● Doppler examination can also reveal fetal blood vessels traversing below the
presenting part

● Management- In a diagnosed case of vasa previa elective cesarean section should


be done or emergency LSCS should Be done if it Is diagnosed intrapartum.

198/284
(Q.211) A 25 yr old heterosexual male from Chandigarh came with history of dysuria and noted
some pus like drainage in his underwear & at the top of his penis. He gave a history
of being sexually active with five or six partner in the past 6 months. His physical
examination showed yellow urethral discharge & tenderness at the tip of the penis. Gram
staining of discharge showing gram negative cocci, On the basis of gram staining what is
the probable cause of infection?

(a) Mycoplasma hominis

(b) Ureaplasmaurealyticum

(c) Gonococci

(d) Chlamydia trachomatis

Your Response : c

Correct Answer : C

Exp: Gonococci

Image description = Gram stain of a urethral exudate of a patient with gonorrhea. Nuclei
of many polymorphonuclear cells are seen. Intracellular gram-negative diplococci
(Neisseria gonorrhoeae) in one polymorphonuclear cell are marked by the arrow.

199/284
(Q.212) All statements are true regarding following except?

(a) It occurs usually on 14th day of menstrual cycle

(b) Epithelial cells have fluid filled vacuoles consist of mucin & glycogen between nucleus &
basement membrane.

(c) It is the earliest feature of ovulation.

(d) Best diagnosis of ovulation is done by endometrial biopsy

Your Response :

Correct Answer : C

Exp: It is the earliest feature of ovulation.

Option A- true.

Option B-this feature if cells is called as sub-nuclear vacuolation

Option C-Earliest feature is sub- nuclear vacuolation

Option D-true, biopsy is best always.

Sub nuclear vacuolation is a characteristic feature of the endometrium in the secretory


phase.

This appearance is a presumptive evidence of progesterone activity.

200/284
(Q.213) Treatment of choice for intraheptic cholestasis in pregnancy is :

(a) Cholestyramine

(b) Ursodiol ( ursodeoxycholic acid)

(c) Corticosteroids ( Dexamethasone )

(d) Antihistaminics

Your Response : b

Correct Answer : B

Exp: Ursodiol (ursodeoxycholic acid)

● Ursodeoxycholic acid is the treatment of choice for Intrahepatic Cholestasis of


Pregnancy.

● 'Treatment is symptomatic, with the bile salt ursodeoxycholic acid (10-15 mg/kg/day)
the treatment of choice, and better than SAdenosyl Methionine.

● Ursodeoxycholic acid relieves pruiritis, reduces bile salt levels in maternal serum
and may reduce the frequency offetal complications.

● Both bile salt sequestration withcholestyramine and dexamethasone have given


variable results

201/284
(Q.214) A 72-yrs-old female presents to the emergency room with generalized weakness. An
ECG was done, which is shown below. Correcting which one of the following electrolyte
disturbances will return the ECG to normal?

(a) Hyperkalemia

(b) Hyponatremia

(c) Hypocalcaemia

(d) Hypercalcemia

Your Response : b

Correct Answer : A

Exp: Hyperkalemia

Hyper kalemia is a life-threatening emergency that can be recognized by the peaked T


waves observed on an electrocardiogram. The peaked T waves are produced by an
accelerated repolarization of ventricular muscle. Potentially fatal hyperkalemia is treated
by administering insulin (along with glucose), which helps K+ transport into cells and
therefore lowers extracellular K+.

202/284
(Q.215) A 23 year old woman represent to her physician with ptosis, meiosis and anhidrosis in
the left eye. Which one of the following drugs is used to determine whether this lesion is
preganglionic or postganglionic?

(a) Atenolol

(b) Cocaine

(c) Epinephrine

(d) Phenylephrine

Your Response : c

Correct Answer : D

Exp: Phenylephrine

It's not Cocaine. cocaine is only to confirm the diagnosis of Horner's syndrome. Usually
Hydroxyamphetamine is used to differentiate between pre-ganglionic and post-ganglionic
Horner's syndrome. hydroxyamphetamine potentiates release of norepinephrine. In this
question, hydroxyamphetamine is not given among the options. The answer here is
R.PHENYLEPHRINE. the underlying concept is denervation hypersensitivity in this case

The answer is PHENYLEPHRINE. Phenylephrine acts by a different phenomenon -


DENERVATION HYPERSENSITIVTY

203/284
(Q.216) Which is not true about human papilloma virus is:

(a) Papova virus

(b) DNA virus

(c) Affects only skin in humans

(d) Causes virus induced carcinoma

Your Response : c

Correct Answer : C

Exp: Affects only skin in humans

Formerly the papillomaviruses and polyomaviruses were together grouped under


Papovaviridae family, but now they are separated as 2 different families
(Papillomaviridae and Polymaviridae).HPV are non-enveloped, icosahedral capsids and
contains a double-stranded circular DNA.Papillomaviruses are highly tropic for epithelial
cells of the skin and mucous membranes.HPV have oncogenic potential.

204/284
(Q.217) All of the following statements concerning transduction are true except:

(a) Transduction may be generalized

(b) Transduction may be specialized

(c) Temperate phages are the preferred vehicles for gene transfer

(d) The phages make slow replicas of their DNA

Your Response :

Correct Answer : D

Exp: The phages make slow replicas of their DNA

In fact, the capacity of phages to make rapid replicas of their DNA has made them
valuable in genetic engineering.

205/284
(Q.218) Which is not true about ZIKA virus:

(a) Belong to flavivirus

(b) RT PCR is useful in diagnosis

(c) Causes macrocephaly

(d) May presents with conjunctivitis

Your Response : c

Correct Answer : C

Exp: Causes macrocephaly

Zika virus disease can have symptoms including mild fever, skin rash, conjunctivitis,
muscle and joint pain, malaise or headache. These symptoms normally last for 2-7 days.
There is scientific consensus that Zika virus is a cause of microcephaly and Guillain-
Barré syndrome. Links to other neurological complications are also being investigated.
Infection with Zika virus may be suspected based on symptoms and recent history of
travel (e.g. residence in or travel to an area with active Zika virus transmission).

Laboratory diagnosis: The FDA has issued Emergency Use Authorization (EUA) for
several diagnostic tools for Zika virus, including the Trioplex Real-Time RT-PCR (rRT-
PCR) assay (should be performed on serum collected during the first two weeks after
symptom onset and on urine samples collected less than 14 days after symptom onset)
and the Zika MAC-ELISA (qualitative detection of Zika virus IgM Abs in serum or CSF).

Prevention and treatment:

● No vaccine exists to prevent Zika.

● Prevent Zika by avoiding mosquito bites.

● There is no specific medicine or vaccine for Zika virus.

206/284
(Q.219) Which of the following feature is used for identification of Cryptococcus neoformans?

(a) Oxidase +ve

(b) Dextran fermentation

(c) Hydrolyse urea

(d) Ability to grow at 42 º C

Your Response :

Correct Answer : C

Exp: Hydrolyse urea

● Cryptococcus hydrolyzes the urea and give test positive.

(Q.220) A 45 years female complains of lower abdominal pain and vaginal discharge. On
examination there is cervicitis along with a mucopurulent cervical discharge. The gram
smear of the discharge shows presence of abundant pus cells but no bacteria. The best
approach to isolates the possible causative agent would be-

(a) Culture on chocolate agar supplemented with Haemin

(b) Culture on McCoy cells

(c) Culture on a bilayer human blood agar

(d) Culture on vero cell lines

Your Response : d

Correct Answer : B

Exp: Culture on McCoy cells

Chlamydiae are usually grown on McCoy cell line and HeLa cell line.

Genital chlamydiasis is suspected if gram stained smear of urogenital exudates shows


significant no of neutrophils. > 4/oil immersion field in urethritis, > 30 in cervicitis in the
absence of gonococcal bacteria.

207/284
(Q.221) A 23-year-old man presents to the emergency room complaining of massive watery
diarrhea for 6 hours. He doesn’t have any vomiting or abdominal pain. Stool culture reveals
a curved; gram negative rod which secretes an enterotoxin contains subunit A and a
binding subunit B. What is the most likely reason for this patient’s massive diarrhea?

(a) Deficiency in the brush border enzymes

(b) Decreased production of cAMP

(c) Increased production of cAMP

(d) Destruction of the absorptive epithelium

Your Response : c

Correct Answer : C

Exp: Increased production of cAMP

This patient most likely presents with cholera. Cholera enterotoxin causes secretory
diarrhea, which means that excretion of water exceeds to a large extent its absorption.
The enterotoxin indirectly stimulates adenylate cyclase to produce cAMP, which results in
increased intracellular concentration of cyclic AMP which causes prolonged opening of
the chloride channels leading to massive excretion of water. The correct answer is C

208/284
(Q.222) Several villagers in karnataka known to eat pork during celebrations were reported to be
suffering from an outbreak of epileptiform seizures. One of the first things you should
investigate is:

(a) The prevalence of Ascaris infections in the population

(b) The prevalence of schistosomiasis in the population

(c) The presence of Trypanosoma brucei gambiense in the villagers

(d) The presence of Taenia solium in the pigs

Your Response : d

Correct Answer : D

Exp: The presence of Taenia solium in the pigs

● TAENIA SOLIUM - PORK TAPEWORM / Armed tape worm

● The adult worms of T. solium reside or inhabit the upper jejunum. Infection has
worldwide distribution.

● Embryonated hexacanth eggs passed with stool are ingested by pig and the embryo
is released.

● After a period of 2-3 months of development the encysted larval stage called
cysticerci occurs in the striated muscles of pig

● Humans become infected by eating pork containing larvae, cysticercus cellulosae.


When improperly cooked cysticercus infected meat is eaten by man, the scolex remains
undigested and attaches itself to the intestinal wall and chain of proglottids begin to grow
to adult worm.

● Diagnosis : Demonstration of eggs in stool specimen

● Treatment :Albendazole, Mebandazole, Niclosamide

● Prevention: Treatment of infected persons, Thorough cooking of pork and proper


processing & Proper disposal of human excreta (good hygiene/ sanitation).

209/284
(Q.223) A 32 year male patient with complaints of fever with chills, malaise and headache along
with diarrhea since last 5 days. Further history revealed campaigning with his friends
approximately 2 weeks back. A peripheral blood smear was done which showed the
following finding, then a card test (Rapid diagnostic test) for malaria was done, which
showed the following result. What could be the possible diagnosis?

(a) Babesiosis

(b) Dengue

(c) Malaria

(d) Kala- Azar

Your Response : b

Correct Answer : A

Exp: Babesiosis

Rapid diagnostic test comes negative


he4e which rule out malaria. If the
diagnosis of babesiosis is being
considered, manual (nonautomated)
review of blood smears should be
requested explicitly. In symptomatic
patients with acute infection, Babesia
parasites typically can be detected by
blood-smear examination, although
multiple smears may need to be
examined. Sometimes it can be
difficult to distinguish between
Babesia and malaria parasites and
even between parasites and artifacts
(such as stain or platelet debris).
Consider having a reference
laboratory confirm the diagnosis and
the species. In some settings,
molecular techniques can be useful for
detecting and differentiating among
Babesia species.

210/284
(Q.224) What is somatic hypermutation?

(a) Activated B cell rearranges its constant region genes of the light chain

(b) Activated B Cells undergo rearrangement of their hypervariable regions

(c) Vascular epithelium upregulate adhesion molecules at the site of infection.

(d) Activated B cell rearranges its constant region genes of the heavy chain

Your Response :

Correct Answer : B

Exp: Activated B Cells undergo rearrangement of their hypervariable regions

• Somatic hypermutation is when activated B Cells undergo rearrangement of their


hypervariable regions; then test themselves on antigen to see if they recognize it
better/worse/the same. If worse, they are deleted, if better they are selected, cloned and
used to secrete higher affinity antibodies.

211/284
(Q.225) The test shown in the figure is negative in-

(a) Staphylococcus

(b) Streptococcus

(c) Micrococcus

(d) Both A and B

Your Response : d

Correct Answer : B

Exp: Streptococcus.

● The image shows the catalase test. Catalase is an enzyme, produced by


microorganisms that live in oxygenated environments to neutralize toxic forms of oxygen
metabolites, H2O2. The catalase enzyme neutralizes the bactericidal effects of
hydrogen peroxide and protects them. Anaerobes generally lack the catalase enzyme.

● A small inoculum of bacterial isolate is mixed into hydrogen peroxide solution


(3%) and rapid formation of gas bubbles is looked for.

● In the Gram positive cocci, Staphylococcus and Micrococcus are both catalase
positive.

212/284
(Q.226) Identify the parasite?

(a) Hookworms

(b) Ascaris adult worms

(c) Schistosomes adult worms

(d) Microfilaria

Your Response : c

Correct Answer : C

Exp: Schistosomes adult worms

Image description = Schistosomes adult worms (thin female resides in gynecophoric


canal of thicker male).

213/284
(Q.227) Parasite which not infects through ingestion of aquatic vegetation:

(a) Fasciola hepatica

(b) Fasciolopsisbuski

(c) Paragonimuswestermani

(d) Watsoniuswatsoni

Your Response :

Correct Answer : C

Exp: Paragonimuswestermani

Life cycle: Trematodes complete their life cycle in 3 different hosts, one definitive host
(man) & two intermediate hosts.

● 1st intermediate host: fresh water snail or mollusk.

● 2nd intermediate host: either aquatic plant or fish or crab.

● Mode of transmission: by eating aquatic plants, fishes or crabs harboring infective


form (metacercariae) or by the penetration of free living cercariae (Schistosoma).

Except Paragonimuswestermani (crab or crab fish), all are infecting human by ingestion
of aquatic vegetation containing metacercariae.

214/284
(Q.228) All of the following organisms are known to survive intracellularly except -

(a) Neisseria meningitides

(b) Salmonella typhi

(c) Streptococcus pyogenes

(d) Legionella pneumophila

Your Response : d

Correct Answer : C

Exp: Streptococcus pyogenes

Streptococcus pyogenes grows extracellular and causes pyogenic infection. Intracellular


bacteria are Listeria monocytogenes, Legionella, Yersinia, Salmonella typhi, Shigella,
Neisseria, Mycobacterium, Francisellatularensis, Pneumococci.

215/284
(Q.229) Which of the following helminthic diseases is transmitted by direct skin penetration by
helminth larvae?

(a) Filariasis

(b) Onchocerciasis

(c) Loasis

(d) Schistosomiasis

Your Response : a

Correct Answer : D

Exp: Schistosomiasis

● Schistosomiasis transmitted by direct skin penetration by fork tailed cercariae.

● All the other options belong to tissue nematodes which are transmitted by arthropod
vectors. Apart from Schistosoma, Strongyloides stercoralis, Ancylostoma duodenale,
Nacator americanus are also transmitted by direct skin penetration.

216/284
(Q.230) Hair perforation test is positive in infection with:

(a) Trichophyton

(b) Microsporum

(c) Epidermophyton

(d) All of the above

Your Response : d

Correct Answer : A

Exp: Trichophyton

● Hair perforation test is done to distinguish between isolates of dermatophytes,


particularly trichophyton mentagrophytes and its variants.

217/284
(Q.231) Identify staining and microorganism?

(a) 10% KOH mount showing Candia albicans

(b) India Ink preparation showing Candida albicans

(c) 10% KOH mount showing Cryptococcus

(d) India Ink preparation showing Cryptococcus

Your Response : c

Correct Answer : D

Exp: India Ink preparation showing Cryptococcus

Image description =India ink (Negative staining) preparation of CSF showing


Cryptococcus neoformans with capsule. The capsule appears as a clear halo around the
yeast cells.

218/284
(Q.232) A 60 year old hypertensive patient on angiotensin II receptor antagonist is posted for
hernia repair surgery. The hypertensive drug should be:

(a) Stopped on the day of operation

(b) Discontinued 24 hours preoperatively

(c) Discontinued one week preoperatively

(d) Administered in increased dosage on the day of operation

Your Response : a

Correct Answer : A

Exp: Stopped on the day of operation

All hypertensives should be continued except ACE inhibitors and angiotensin receptor
antagonist which are omitted on the day of surgery, otherwise patient can develop
intraoperative hypotension.

219/284
(Q.233) A patient with history of myocardial ischemia was operated for cholecystectomy. In
post-operative period he developed severe respiratory distress. Pulmonary artery
catheterization was done which shows value of 28 mmHg. The likely diagnosis is:

(a) Cardiac asthma

(b) Pneumothorax due to rupture of pleura during surgery

(c) Haemothorax

(d) Bronchial asthma

Your Response :

Correct Answer : A

Exp: Cardiac asthma

Pulmonary capillary wedge pressure more than 25 mmHg indicates pulmonary edema
i.e., cardiac asthma.

220/284
(Q.234) A 25 year male to be operated for appendicectomy. During lumbar puncture there is
bloody tap. The next step should be:

(a) Defer the surgery

(b) Convert to GA

(c) Withdraw and reinsert the needle

(d) Give test dose of local anaesthetic

Your Response : b

Correct Answer : C

Exp: Withdraw and reinsert the needle

If there is bloody tap during spinal, needle should be withdrawn flushed with normal
saline and reinserted preferably in another space.

221/284
(Q.235) A 24-year-old man is carried into the emergency department by friends after a fight in
the street. During the fight, the patient was struck in the head with a knife, which became
firmly implanted in his skull. His blood pressure is 106/72 mm Hg, pulse is 118/min, and
respirations are 14/min. Pulse oximetry shows 93% on room air. His Glasgow Coma Scale
score is 8. He opens his eyes to pain, and his speech is incomprehensible. Compressing
the fingernail bed causes him to withdraw his hand. The estimated length of the knife is 14
cm, 3 cm of which have penetrated his skull. There is no other evidence of injury on his
body. Which of the following is the most important next step in management of this
patient?

(a) Irrigate the skull wound with normal saline solution

(b) Remove the knife from his skull under local anesthesia

(c) Transport the patient to radiology for CT scan of the head

(d) Intubate the patient

Your Response : c

Correct Answer : D

Exp: Intubate the patient

This patient is brought to the emergency department with a penetrating injury to his skull.
As is true for any trauma, maintenance of an adequate airway and hemodynamic stability
are the first steps in acute management. The Glasgow Coma Scale (GCS) score is useful
in assessing level of consciousness and depth of injury. It consists of 3 parameters,
scored against a standard scale: best eye (E) response (maximum of 4 points); best
verbal (V) response (maximum of 5 points); and best motor (M) response (maximum of 6
points). GCS scores range from 3-15. Intubation is recommended for a GCS score $8.
This patient opens his eyes to pain only. He makes incomprehensible sounds and
withdraws his hand in response to painful stimuli. His GCS score is E2 + V2 + M4 = 8.
His respiratory rate and oxygen saturation level suggest that he may be becoming
hypopneic and hypoxemic. He should be intubated. (Choice B) Local wound
management is not an emergency and can be addressed at a later stage. (Choice C)
Removal of the knife may lead to an increase in bleeding due to opening of the dural
venous sinuses. This should be attempted in the operating room. (Choice D) It is
important to ensure the patient's hemodynamic stability and complete a full initial
evaluation before any further radiographic imaging.

222/284
(Q.236) All are true about the image shown except:

(a) Anti-gravity drainage

(b) Cold abscess

(c) Transillumination test is negative

(d) Rubbery consistency of involved lymph nodes

Your Response : a

Correct Answer : D

Exp: Rubbery consistency of involved lymph nodes

The image shows anti-gravity drainage of a cold abscess being done. The upper deep
cervical lymph nodes are usually involved. On palpation matting of lymph nodes is
characteristic feature of TB, while the rubbery consistency of lymph nodes is seen in
Hodgkins lymphoma.

Cold abscess features are:

● No local rise of temperature

● No redness

● Transillumination test is negative

● On sternocleidomastoid contraction test, it becomes less prominent indicating it is


below the deep fascia.

223/284
(Q.237) Stapedial reflex is mediated by:

(a) V and VII nerves

(b) V and VIII nerves

(c) VII and VI nerves

(d) VII and VIII nerves

Your Response : b

Correct Answer : D

Exp: VII and VIII nerves

The acoustic reflex (also known as the stapedius reflex, middle-ear-muscles (MEM)
reflex, attenuation reflex, or auditory reflex) is an involuntary muscle contraction that
occurs in the middle ear in response to high-intensity sound stimuli or when the person
starts to vocalize.

224/284
(Q.238) Study the given image, reassurance is sufficient in how many patients:

(a) 90%

(b) 70%

(c) 60%

(d) 10%

Your Response :

Correct Answer : A

Exp: 90%

Laryngomalacia is the most common congenital anomaly of larynx

Reassurance is sufficient in 90% of the cases

225/284
(Q.239) Identify the parasitic structure?

(a) Unfertilized egg of Ascaris

(b) Mature cyst of Entamoeba histolytica

(c) Trophozoite of Entamoeba histolytica

(d) Cystic form of Balamuthia mandrillaris in Wet mount

Your Response : c

Correct Answer : B

Exp: Mature cyst of Entamoeba histolytica

Image description =Mature Cyst (quadrinucleated) of E. histolytica in an unstained wet


mount preparation with saline.

226/284
(Q.240) Immunoconglutinins (IK) are a type of:

(a) IgG antibodies

(b) IgA antibodies

(c) IgM antibodies

(d) IgE antibodies

Your Response : b

Correct Answer : B

Exp: IgA antibodies

Immunoconglutinins are IgA antibodies. They are found in saliva and jejunal secretions.

(Q.241) During positioning of patient who has undergone craniotomy it is suddenly found that
pO2 = 85% and end tidal carbon dioxide becomes zero. The possible diagnosis is:

(a) Endotracheal tube in esophagus

(b) Blockage of endotracheal tube by secretions

(c) Venous air embolism

(d) Tracheomalacia

Your Response :

Correct Answer : A

Exp: Endotracheal tube in esophagus

End tidal carbon dioxide is the surest sign of correct intubation. During positioning there
is chance of accidental extubation or displacement of tube.

End tidal carbon dioxide may decrease in venous air embolism, even becomes zero if
embolus is large enough to block the whole pulmonary circulation but this will occur later
on when the dura is opened, not during positioning.

227/284
(Q.242) A patient in ICU is being cannulated through radial artery for direct arterial pressure
monitoring. After some time the patient developed paleness of involved hand. The next
best step in management is:

(a) Local application of thrombophob gel

(b) IV local anaesthesia through the same line

(c) Stellate ganglion block

(d) Remove the needle and massage the hand

Your Response : d

Correct Answer : C

Exp: Stellate ganglion block

Paleness of hand indicates that radial artery has gone in spasm and moreover collaterals
(ulnar artery) is also not adequately functioning, so to relieve spasm IV local anaesthetic
will also be beneficial but most definitive management is stellate ganglion block.

(Q.243) A 25 years male is undergoing incision and drainage of abscess under general
anaesthesia with spontaneous respiration. The most efficient anaesthetic circuit is:

(a) Mapleson A

(b) Mapleson B

(c) Mapleson C

(d) Mapleson D

Your Response : d

Correct Answer : A

Exp: Mapleson A

Mapleson A (Magill) is the circuit of choice for spontaneously breathing patient.

228/284
(Q.244) A young scooterist met an accident with crush injury of hand, foot and splenic rupture
and is to be operated by laparotomy. He had taken food 1 hour before. The technique most
preferred is:

(a) Rapid sequence intubation with suxamethonium

(b) Rapid sequence intubation with rocuronium

(c) Rapid sequence intubation with mivacurium

(d) Conventional general anaesthesia

Your Response : b

Correct Answer : B

Exp: Rapid sequence intubation with rocuronium

The patient is full stomach so rapid sequence intubation with Sellik’smanoeuvre has to
be accomplished. Since this patient has crush injury so succinylcholine cannot be use d.
Rocuronium almost having the same onset of action is an excellent alternative to
succinylcholine for rapid sequence intubation.

229/284
(Q.245) A 60-year-old man was brought in with a history of high grade fever with chills and
rigors for the previous two days. He also had a mild chest pain and productive cough. The
sputum was submitted for investigation. X-ray chest showed consolidation in the right
lower lobe. The direct smear from sputum showing this picture, On the basis of gram
staining what is the probable cause of infection?

(a) Streptococcus pyogenes

(b) Streptococcus
pneumoniae

(c) N. meningitidis

(d) Kleb. pneumoniae

Your Response :

Correct Answer : B

Exp: Streptococcus
pneumoniae

Streptococcus
pneumoniae in sputum
are seen as lancet-
shaped gram-positive
diplococcic surrounded by
clear halo (Capsule).
Degenerating nuclei of
polymorphonuclear cells
are the large darker
irregular red shapes.
Mucus and amorphous
debris are present in the
background.

230/284
(Q.246) Treatment of Ca larynx in stage. T1, MONO is

(a) Radiotherapy

(b) Surgery Total laryngectomy

(c) Laser therapy

(d) Micro laryngoscopic surgery

Your Response :

Correct Answer : D

Exp: Microlaryngoscopic surgery

Treatment of Ca Larynx

Stage Site Treatment

Tl All site External beam radio therapy/Microlaryngeal surgery

T2 Glottic and subglottic lesion Radiotherapy

Supraglottic lesionSupraglottic laryngectomy

T3andT4 All sites Total laryngectomy with neck dissection for clinically positive nodes
and post-operative radiotherapy if nodes are not palpable.

231/284
(Q.247) A surgeon tells a medical student to tap the side of the face of a patient who just had
thyroid surgery. The surgeon is most worried about damage to which of the following
vessels?

(a) Common carotid artery

(b) Superior Thyroid artery

(c) Internal jugular artery

(d) Inferior thyroid artery

Your Response : b

Correct Answer : D

Exp: Inferior thyroid artery

The surgeon has asked the medical student to test for tetany, which can occur if the
blood supply to the parathyroid glands (through the superior and inferior thyroid arteries)
is disrupted during thyroid surgery. Specifically, the medical student is being asked to tap
with his fingers the muscles of mastication, notably the masseter, which because of its
strength is a sensitive indicator of tetany. Tetany will be seen as an abnormally strong
jerk or contraction related to the hypocalcemia that can develop if secretion of
parathyroid hormone is inadequate.

232/284
(Q.248) Free water clearance of 1.3 ml/min signifies?

(a) No secretion of vassopressin

(b) Urine is hypotonic to plasma

(c) Urine is hypertonic to plasma

(d) Urine is isosmotic to plasma

Your Response :

Correct Answer : B

Exp: Urine is hypotonic to plasma

Here free water clearance is positive, means urine is hypotonic to plasma. Ideally ADH
secretion should not be there. But remember ADH secretion will not be zero (low basal
level will be present).

See the attached graph. (clearly written, with low plasma ADH level, hypotonic urine is
excreted)..

The relationship between urine osmolality and plasma arginine vasopressin (AVP) levels.
The dashed horizontal line reports the normal plasma osmolality (285 mOsm/kg H2O),
with low plasma AVP levels, urine hypo-osmotic to plasma is excreted, and with high
plasma AVP levels, hyperosmotic urine is excreted, Maximally concentrated urine (1,200
mOsm/kg H2O) is produced when the plasma AVP level is about 5pg/mL .

233/284
(Q.249) If a cat apneustic center is destroyed along with cutting of vagi. Which of the following
statement is correct regarding the breathing pattern seen in cat?

(a) Prolonged inspiratory spasm

(b) Prolonged expiratory spasm

(c) Slow nad shallow respiration

(d) Animal will die

Your Response : d

Correct Answer : A

Exp: Prolonged inspiratory spasm

Prolonged Inspiratory Spasm, as Pneumotaxic center acts through apneustic center and
vagal inhibition is lost.

234/284
(Q.250) A 35-year-old man complains of rapid onset of midepigastric pain with radiation to the
back after eating a large meal. Physical exam shows low grade fever, epigastric
tenderness, and decreased bowel sounds. Abdominal CT scan shows localized dilation of
the upper duodenum and a small collection of fluid in the left pleural cavity. On physical
examination following is seen Diagnosis is ?

(a) Haemorrhagic disease

(b) Acute pancreatitis

(c) Ulcerative colitis

(d) Crohns disease

Your Response : c

Correct Answer : B

Exp: Acute pancreatitis

● Image given above shows Cullen's sign

● Life threatening disease Caused by leakage of pancreatic enzymes that auto digest
the pancreas

● More common in males and in a younger age group

● 70% related to gallstone disease alcohol

● Etiologies include (PANCREATITIS)

● Posterior perforation of peptic ulcer

● Alcohol

● Neoplasm

● Cholelithiasis

● Cholecystectomy
235/284
● Increased calcium

● Biliary tract obstruction

● Renal disease

● ERCP

● Anorexia

● Trauma

● Infections

● Toxins/drugs (thiazides, AZT, protease inhibitors)

● Incineration

● Stings (scorpion)

● Hypertriglyceridemia

Symptoms

● Severe epigastric pain

● Described as steady and boring

● Radiates to the back

● Relieved by leaning forward

● Nausea

● Vomiting

● Weakness

● Low fever (70-85%)

● Shock (20-40%), due to loss of fluid in peripancreatic third space.

Physical examination

● Abdominal tenderness without guarding or rebound

● Diminished bowel sounds from a localized ileus

● Hemorrhagic pancreatitis indicated by Grey Turner’s sign (purple discoloration of the


flank)

● Cullen’s sign (periumbilical purple discoloration)

● Jaundice is rare.

Evaluation

Labs:

Elevated amylase: elevated in 95% of acute attacks during the first 12-24hrs

236/284
(Q.251) A 45-year-old builder fell from scaffolding to the ground this evening. He landed on his
left side and initially had pain in his left lower chest. He has now developed severe
abdominal pain. He is becoming increasingly tachycardic and is hypotensive. His abdomen
is tender with guarding and he is tender over his left 10th and 11th ribs. Investigation of
choice would be:

(a) Diagnostic peritoneal tap or lavage.

(b) CT abdomen ± contrast.

(c) Erect chest X-ray.

(d) Supine abdominal X-ray.

Your Response :

Correct Answer : A

Exp: Diagnostic peritoneal tap or lavage.

● Lower chest injuries may be associated with rupture of liver or spleen.

● Splenic rupture may present immediately or after a delay of hours, even days.

● The signs are peritonism and shock that is out of proportion to the degree of
observed blood loss or apparent trauma.

● The patients may be tender locally, in the left upper quardrant, or generally.

● Ultrasound or CT scan will show the splenic rupture but if the patient is shocked and
there is likely to be any delay in arranging imaging (e.g. outside of working hours),
diagnostic peritoneal lavage will confirm the presence of intra-peritoneal haemorrhage.

● Lapartomy need not wait for imaging.

(Q.252) Body’s metabolic response to trauma in plotted on the graph as shown below. Identify A
& B?

237/284
(a) A = Ebb phase, B = Slow Phase

(b) A = Ebb phase, B = Flow Phase

(c) A = Reverse phase, B = Flow Phase

(d) A = Flow Phase, B = Ebb Phase

Your Response : b

Correct Answer : B

Exp: A = Ebb phase, B = Flow Phase

● In 1930, Sir David Cuthbertson divided the metabolic response to injury in humans
into ‘ebb’ and ‘flow’ phases. The ebb phase begins at the time of injury and lasts for
approximately 24-48 hours. It may be attenuated by proper resuscitation, but not
completely abolished.

● The ebb phase is characterized by hypovolaemia, decreased basal metabolic rate,


reduced cardiac output, hypothermia and lactic acidosis.The predominant hormones
regulating the ebb phase are catecholamines, cortisol system.

● The magnitude of this neuroendocrine response depends on the degree of blood


loss and the stimulation of somatic afferent nerves at the site of injury. The main
physiological role of the ebb phase is to conserve both circulating volume and energy
stores for recovery and repair.

● Following resuscitation, the ebb phase evolves into a hypermetabolic flow phase,
which corresponds to the SIRS. This phase involves the mobilization of body energy
stores for recovery and repair, and the subsequent replacement of lost or damaged
tissue.

● It is characterized by tissue oedema (from vasodilatation and increased capillary


leakage), increased basal metabolic rate (hypermetabolism), increased cardiac output,
raised body temperature. leucocytosis, increased oxygen consumption and increased
gluconeogenesis.

● The flow phase may be subdivided into an initial catabolic phase, lasting
approximately 3-10 days, followed by an anabolic phase, which may last for weeks if
extensive recovery and repair are required following serious injury.

● During the catabolic phase, the increased production of counter-regulatory


hormones (including catecholamines, cortisol, insulin and glucagons) and inflammatory
cytokines (e.g. IL-1, IL-6 and TNF) results in significant fat and protein mobilization,
leading to significant weight loss and increased urinary nitrogen excretion.

238/284
(Q.253) Which is the best fluid for resuscitation during shock state?

(a) Crystalloids

(b) Colloids

(c) Plasma substitutes

(d) 5% dextrose

Your Response : a

Correct Answer : A

Exp: Crystalloids

● “Fluid resuscitation is a major adjunct to physically controlling hemorrhage in


patients with shock.

● The ideal type of fluid to be used continues to be debated; however, crystalloids


continue to be the mainstay of fluid choice”

● Several studies have demonstrated increased risk of death in bleeding trauma


patients treated with colloid compared to patients treated with crystalloid. In patients with
severe hemorrhage, restoration of intravascular volume should be achieved with blood
products.

(Q.254) 2.36/m presented with dyspepsia and abdominal distension. Esophagoscopy was done
and a biopsy was taken from the lesion in the esophagus. Which of the following is not true
regarding the condition?

239/284
(a) Squamous epithelium are replacing Columnar cells

(b) Mucus secreting cells may be seen

(c) May progress to dysplasia

(d) Risk of malignancy is increased

Your Response :

Correct Answer : A

Exp: Squamous epithelium are replacing Columnar cells

Barrets esophagus in which the columnar cells replace the squamous cells. Mucin
secreting cells may be seen. The condition can progress to dysplasia and thereof to
malignancy

(Q.255) A 5 year old boy presented with history of intermittent episodes of blood P/R along with
pain abdomen localized to the umbilical region. On probing it was found that there was
history on an episode of intussusception in the past. The following radiological
investigation was performed. Identify the investigation and the probable diagnosis.

240/284
(a) Selective mesenteric arteriography; Meckels diverticulum

(b) Selective mesenteric arteriography; Angiodysplasia

(c) Tc99mpertechnetatescan; Meckels diverticulum

(d) Tc99m sulphur colloid scan; Angiodysplasia

Your Response : c

Correct Answer : C

241/284
Exp: Tc99mpertechnetatescan; Meckels diverticulum

● The question is giving a history of bleeding P/R, pain abdomen around umbilicus
and intussusception which is pointing towards a diagnosis of Meckels diverticulum.

● Although angiodysplasia may present with bleeding P/R but pain abdomen and
intussusception are rare.

● The given Image is of a nuclear scan showing normal background uptake and
increased uptake in stomach (yellow arrow)suggesting that the radiotracer is binding to
gastric mucosa. One such tracer is Tc99m pertechnetate.

● Also there is a spot of increased uptake in the right side of the umbilicus suggestive
of ectopic gastric mucosa (red arrow).

● Therefore the given image is of a Tc99m pertechnetate scan showing Meckels


Diverticulum.

242/284
(Q.256) A 17-year-old man is stabbed in the axilla. He is noted to have loss of function of the
ulnar nerve. If nonoperative therapy is used for treatment of this nerve injury, at what rate
is the nerve expected to regenerate?

(a) 1 mm/week

(b) 1 mm/month

(c) 1 mm/day

(d) 10 mm/month

Your Response :

Correct Answer : C

Exp: 1 mm/day

● When a nerve axon is disrupted, such as occurs with axonotmesis or neurotmesis,


the distal nerve degenerates to the motor endplate and the proximal nerve degenerates
back to the nodes of Ranvier.

● If scar tissue does not prohibit renewal, nerves will regenerate at an average of 1
mm/d.

243/284
(Q.257) A 24-year-old man falls on the ground when is struck in the right temple by a baseball.
While being driven to the hospital, he lapses into coma. He is unresponsive with the dilated
right pupil when he reaches the emergency department. The most important step in initial
management is :

(a) Craniotomy

(b) CT scan of the head

(c) X-ray of the skull and cervical spine

(d) Doppler ultrasound examination of the neck.

Your Response : b

Correct Answer : B

Exp: CT scan of the head

For any patient with head injury an emergency CT scan is indicated before surgical
intervention. Guidelines for CT in Head injury:

● GCS < 13 at any point of time

● GCS 13 or 14 at 2 hours

● Presence of focal neurologic deficit

● Suspected skull fracture

● Seizures

● More than one episode to vomiting

● Urgent CT scan is needed if none of the above but:

● Age > 65

● Coagulopathy (e.g. patient is taking anticoagulants)

● Antegrade amnesia > 30 min (CT with in 8 hours)

● Dangerous mechanism of injury.

244/284
(Q.258) Internal nasal valve is being shown in the given image and choose the incorrect
statement:

(a) Is the widest portion of nostril

(b) Is responsible for sinus ventilation

(c) Is responsible for parabolic nasal flow

(d) Upper edge of lower lateral cartilage is important boundary.

Your Response :

Correct Answer : A

Exp: Is the widest portion of nostril

● Internal nasal valve:

● Is the narrowest portion of nostril.

● Is responsible for sinus ventilation.

● Is responsible for parabolic nasal flow.

245/284
(Q.259) The Superior and side view of the larynx is shown below. Identify the Abductor of the
vocal cords:

(a) A

(b) B

(c) C

(d) D

Your Response : c

Correct Answer : A

Exp: Posterior cricoarytenoid muscle

● The image shows the only abductor of the larynx which is posterior cricoarytenoid
muscle.

● This happens to be the posterior – most muscle.

● Posterior cricoarytenoid muscles abduct and externally rotate the arytenoids


cartilages, resulting in abducted vocal folds.

● All other muscles are adductors of the vocal cords.

246/284
(Q.260) As an incidental finding during an upper abdominal CT scan, a 3-cm mass in the adrenal
gland is noted. The appropriate next step in analysis and management of this finding
would be:

(a) Observation

(b) CT-guided needle biopsy

(c) Excision of the mass

(d) Measurement of urine catecholamine excretion

Your Response : d

Correct Answer : A

Exp: Observation

● With the increasing use of CT and MRI scans for other purposes, small
“incidentalomas” of the adrenal gland are becoming a frequent finding.

● In the absence of any clinical signs or symptoms of endocrine dysfunction, most


experts now recommend observation and a search for evidence of endocrine dysfunction
for lesions less than 5 cm in diameter.

247/284
(Q.261) Anal fissure is best diagnosed by:

(a) Proctoscopy

(b) Digital rectal examination

(c) Sigmoidoscopy

(d) Visual examination and typical history

Your Response : c

Correct Answer : D

Exp: Visual examination and typical history

Anal fissure is best diagnosed by Visual examination and typical history Visual
examination and typical history

● Anal fissure is longitudinal split in anoderm of distal anal canal. Usually extending
from anal verge to dentate line. It is most commonly caused by hard stool (constipation).
Diarrhea can also cause it.

● MC site is midline posteriorly. Anterior anal fissure are commonly seen in females.

● Since the area of split involved is sensate, the characteristic symptom is severe pain
associate with defecation. In between evacuation pain subsides.

● Chronic fissure is characterized by hypertrophied anal papilla internally and sentinel


tag externally.

● Most patients are young adults. Men and women are affected equally.

● Besides pain, other symptoms may be per rectal bleed which is fresh and small in
quantity.

● Anal fissure are best diagnosed by visual examination revealing a ulcer in typical 6o
clock position (midline posterior). Any other examination especially digital rectal
examination should not be done as its a painful condition.

● A typical position warranting further examination requires examination under GA


(general anesthesia).

248/284
(Q.262) 72-year-old female with a history of diabetes, asthma, and COPD is hospitalized for left
leg cellulitis. She receives oral (PO) antibiotic therapy with clindamycin and begins to
improve with significant reduction in the erythema and the area affected by the cellulitis.
However, on day 4 of her hospitalization she complains to the team that she is having a
large number of watery, foul-smelling stools. Her team, is highly suspicion of a C. difficile
infection, and orders a toxin PCR. However, the PCR is found to be negative. What is the
most appropriate next step in the management of this patient's diarrhea?

(a) Limited sigmoidoscopy

(b) PO metronidazole

(c) PO vancomycin

(d) No further treatment

Your Response : c

Correct Answer : A

Exp: Limited sigmoidoscopy

C. difficile is a gram positive bacterial infection causing pseudomembranous colitis,


which often presents with recurrent, watery, foul smelling diarrhea. It is often caused by
recent antibiotic use, especially clindamycin or ampicillin. C. difficile has a wide range of
severity from mild diarrhea to severe-life threatening diarrhea and colitis. Diagnosis of C.
difficile is made by identification of the C. difficile toxin by ELISA or PCR. In cases where
C. diff is highly suspected (recent antibiotic use, classic presentation) but a toxin assay is
negative, a limited sigmoidoscopy or colonoscopy should be performed to confirm
diagnosis by noting pseudomembranous colitis

249/284
(Q.263) Of the following transmissible agents that may give rise to focal liver infections, which
may cause an anaphylactic response if the contents are spilled in the event of resection or
rupture ?

(a) Entamoeba histolytica

(b) Staphylococcus aureus

(c) Clonorchis sinensis

(d) Echinococcus granulosus

Your Response :

Correct Answer : D

Exp: Echinococcus granulosus

● Pyogenic abscesses, such as that with Staphylococcus or Proteus may cause febrile
reactions and can lead to septicemia, but not hypersensitivity reactions.

● Clonorchis sinensis is an Oriental liver fluke that takes up residence in the biliary
tree, and is a source of inflammatory response, but not anaphylaxis. Amoeba are
protozoans that in some zymogen classes not only give rise to colonic inflammation but
invade the portal

blood stream and can set up hepatic abscess, which when drained does not give
anaphylaxis.

● This is not the case with hydatid cyst caused by Echinococcus granulosus. Not only
can rupture of the cyst result in implantation and dissemination of daughter cysts,
Echinococcus can cause a lethal hypersensitivity reaction as well if spilled into the
peritoneum.

250/284
(Q.264) A colonoscopic view from a 36 year old patient is shown below. He presents with
unexplained rectal bleeding ,abdominal pain and diarrhoea since many months. which of
the following gene is associated with this cancer predisposing condition

(a) APC

(b) ATM

(c) WT1

(d) P16INK4

Your Response : c

Correct Answer : A

Exp: APC

Image given above is familial adenomatous polyposis

Familial adenomatous polyposis is dominantly inherited colon cancer syndrome due to


germline mutation in adenomatosis polyposis coli(APC)

251/284
(Q.265) Chronic Urethral Obstruction due to benign prostatic hypertrophy can lead to the
following change in the kidney parenchyma:

(a) Hyperplasia

(b) Hypertrophy

(c) Atrophy

(d) Dysplasia

Your Response : b

Correct Answer : C

Exp: Atrophy

● “Chronic urethral obstruction due to BPH leads to hydronephrosis.

● In chronic cases, the kidney shows cortical tubular atrophy with marked interstitial
fibrosis.

● Progressive blunting of the calyces occurs and these eventually become cupped.”

(Q.266) Which of the following is not true about LeFort fracture?

(a) Le Fort fracture doesn’t involves pterygoid plates

(b) Maxilla and face can dissociate from cranium in LeFort III

(c) A person can have more than 2 LeFort fractures at same time

(d) Fracture running across the apices of teeth is LeFort I

Your Response : b

Correct Answer : A

252/284
Exp: Le Fort fracture doesn’t involves pterygoid plates

Le Fort fractures have been described into three types depending upon where the
fracture line runs. By definition all types must include pterygoid plates to complete a Le
Fort fracture.

Fracture line running of Le Fort fracture:

Type I- PA → MS (I) → PP

Type II- NF → IOR → MS (L) → PP

Type III- NF → MF → O → ZF → PP

PA- pyriform aperture

MS (I)- maxillary sinus, inferior wall

MS (L) - maxillary sinus, lateral wall

PP- pterygoid plates

NF- nasofrontal suture

MF- maxilla-frontal suture

ZF- zygomaticofrontal suture

Type I fracture runs just above the teeth apices- separating alveolus and teeth from face

Type II runs in a pyramidal shape separating maxilla from face a/k/a pyramidal fracture

Type III separates whole of face from cranium- a/k/a craniofacial dysjunction

Type I- floating palate/alveolus

Type II- floating maxilla

Type III- floating face

253/284
(Q.267) Which of the following is primarily a disease of nipple areola complex?

(a) Duct papilloma

(b) Paget’s disease

(c) Periductal mastitis

(d) Fibroadenoma

Your Response : a

Correct Answer : B

Exp: Paget’s disease

● Paget's (PAJ-its) disease of the breast is a rare form of breast cancer. Paget's
disease of the breast starts on the nipple and extends to the dark circle of skin (areola)
around the nipple. Paget's disease of the breast isn't related to Paget's disease of the
bone, a metabolic bone disease.

● Paget's disease of the breast occurs most often in women older than age 50. Most
women with Paget's disease of the breast have underlying ductal breast cancer, either in
situ — meaning in its original place — or, less commonly, invasive breast cancer.

● Only in rare cases is Paget's disease of the breast confined to the nipple itself.

254/284
(Q.268) A person underwent routine medical examination for recruitment. He was found to have
nodule on PR examination. A biopsy revealed carcinoid tumor. Which of the following is
likely to present with carcinoid syndrome

(a) Metastasis to LNs

(b) Size >2cm

(c) Liver metastasis

(d) Size <2cm

Your Response : c

Correct Answer : C

Exp: Liver metastasis

Classical symptoms of carcinoid syndrome includes

Cutaneous flushing (80%);

Diarrhea (76%);

Hepatomegaly (71%);

Cardiac lesions, most commonly right-sided heart valvular disease (41% to 70%);
Asthma (25%)

These symptoms are due to secretion of vasoactive peptides into systemic circulation
(which include serotonin, 5-HTP (a precursor of serotonin synthesis),
histamine,dopamine, kallikrein, substance P, prostaglandin, and neuropeptide K).

All these peptide undergo first pass metabolism in the liver. So, for carcinoid syndrome to
occur these peptides must escape liver into systemic circulation. And since all GIT drains
through liver, a carcinoid syndrome must be extra-abdominal or must have a liver
metastasis so that secreted vasoactive peptide do not undergo first pass metabolism in
liver and cause symptoms of carcinoid syndrome.

255/284
(Q.269) Goldmans index is used to calculate which of the following in Preoperative
assessment?

(a) Pulmonary risk

(b) Cardiac risk

(c) Metabolic risk

(d) Hepatic risk

Your Response : b

Correct Answer : B

Exp: Cardiac risk

Goldmans Index is used for assessing cardiac risk in a non-cardiac surgery patient. Its
components are as follows-

● Jugular venous distention - 11 points

● Recent MI within 6 months - 10 points

● Premature ventricular contractions (≥5 per min) - 7 points

● Rhythm other than sinus rhythm - 7 points

● Age >70 - 5 points

● Emergency surgery - 4 points

● Aortic valve stenosis - 3 points

● Medical condition within the chest or abdomen- 3 points

● Surgery within the chest or abdomen - 3 points

(Q.270) Which among the following are true about the wound shown in the picture below ?

256/284
(a) Breadth is maximum.

(b) Length is maximum.

(c) Depth is maximum.

(d) It has wound of entry and exit.

Your Response : a

Correct Answer : C

257/284
Exp: Depth is maximum.

The wound in the picture above


representsstab wound.

STAB WOUNDS

● Produced when a pointed


weapon or object or instrument is
forcibly thrust into the body

● Length

● May be same as the


breadth of the blade of weapon or

● Slightly smaller because of


elasticity of skin or

● Larger because of the lateral


movement of the victim or
assailant

● Breadth – may not


approximate with each other
because of gaping

● Depth – greatest among the


3 dimensions

● No relationship with the


length of the weapon

● Margins – regular and clean


cut

● Shape – depends on the type


of weapon and site of injury

● Single edged weapon –


wedge shaped or triangular. Blunt
end of the wound may have small
splits in the skin at each end of
the corner – fishtailing

● Double edged weapon –


spindle shaped or elliptical

● Direction

● Indicated by the track of


wound

● Helps to determine the


relative position of the victim and
assailant at the material time

258/284
(Q.271) At autopsy the cyanide poisoning case will show the following feature except:

(a) The eyes are bright, glistening and prominent with dilated pupils.

(b) The jaws are firmly closed

(c) Characteristic bitter lemon smell

(d) The colour of the cheeks and postmortem staining may be cherry-red

Your Response : c

Correct Answer : C

Exp: Characteristic bitter lemon smell

Post-Mortem Appearances Of Cyandine Poisoning

● The colour of the cheeks and postmortem staining may be cherry-red in about half
the cases, because oxygen remains in the cells as oxyhaemoglobin, and due to the
formation of cyanmethaemoglobin.

● The odour of bitter almond may be noticed on opening the body.

● The eyes may be bright, glistening and prominent with dilated pupils.

● The jaws are firmly closed and there is froth at the mouth

● The mucosa of the stomach may be eroded and blackened due to the formation of
alkaline haematin.

259/284
(Q.272) Which of the following mechanochemical enzymes can be found on the surfaces of
cellular organelles where it mediates movement toward the plusend of microtubules?

(a) Myosin (myosin II)

(b) Minimyosin (myosin I)

(c) Dynein

(d) Kinesin

Your Response : c

Correct Answer : d

Exp: Kinesin

Myosin II: powers muscle contraction.

• Myosin-V, one of 18 known classes of myosin motors, is involved in fast axonal/dendritic


transport, the anterograde and retrograde movement of vesicular cargo in neurons (main
myosin motor).

• Myosin-V is absolutely essential for transport of vesicles in actin-rich cortical regions of


neurons, i.e. dendritic spines and axon termini.

• Other types of myosin that are also involved in neuronal transport are classes I and VI.

• Only Myosin1b has been shown to associate with organelles transport (not Ia and Ic)

• Kinesin is plus end directed motor but dynein is negative end directed motor.

260/284
(Q.273) A 73 year-old man presents with progressive dyspnea on exertion over the past one
year. He reports a dry cough but no wheezes, sputum production, fevers or
hemoptysis. He is a life-long non-smoker and worked as a lawyer until retiring 3 years ago.
His pulmonary function testing is as follows. What is the most probable diagnosis?

(a) Bronchitis.

(b) Emphysema.

(c) Idiopathic Pulmonary Fibrosis.

(d) Asthma.

Your Response : b

Correct Answer : C

261/284
Exp: Idiopathic Pulmonary Fibrosis.

● This patient has a reduced FEV1 and FVC with a preserved FEV1/FVC ratio.

● The total lung capacity is reduced and the patient, therefore, has a restrictive defect.

● The flow-volume loop also has the characteristic appearance of a restrictive process
– tall, narrow and a short expiratory phase.

● Based on the fact that his TLC is below 50% predicted, this would be classified as a
“severe” restrictive defect.

● His DLCO is also markedly reduced indicating he has a reduced alveolarcapillary


interface for gas exchange and suggesting that the cause of his restrictive process lies
within the lung parenchyma.

● Idiopathic pulmonary fibrosis might be a cause in this case .

(Q.274) Tapping the patella tendon elicits a reflex contraction of the quadriceps muscle. During
the contraction of the quadriceps muscle,

(a) The Ib afferents from the Golgi tendon organ increase their rate of firing

(b) The la afferents from the muscle spindle increase their rate of firing

(c) The alpha motoneurons innervating the extrafusal muscle fibers decrease their rate of
firing

(d) The gamma motoneurons innervating the intrafusal muscle fibers increase their rate
firing

Your Response : a

Correct Answer : A

Exp: The Ib afferents from the Golgi tendon organ increase their rate of firing

Read the question, it's asking during contraction of quadriceps muscle? During
contraction tension in the muscle tendon will be high that leads to 1b activation via Golgi
tendon organ.

If the question is : during tapping of tendon? Then answer will be B (1a via muscle
spindle).

262/284
(Q.275) Identify the condition based on the given CT image?

(a) Ulcerative colitis

(b) Intestinal ischemia

(c) Crohn’s disease

(d) Typhoid

Your Response :

Correct Answer : C

Exp: Crohn’s disease

The given image shows hypervascular mesentery suggestive of activeCrohn’s


disease. This sign is also known as Comb sign.

263/284
(Q.276) A young male with a thyroid nodule was diagnosed for papillary carcinoma. After
diagnostic evaluation and pre-anesthesia clearance he was planned for total
thyroidectomy. During thyroid surgery, initial skin flap is to be raised in which plane

(a) Subcutaneous plane

(b) Subfascial plane

(c) Subplatysmal plane

(d) Below the strap muscles

Your Response : c

Correct Answer : C

Exp: Subplatysmal plane

Major surgical steps of thyroid surgery-

● Position of patient- supine with neck extended (with a help of a sand bag below the
shoulders)- also called Roses position. Table is elevated at head end in 15 degree
reverse trendelenberg.

● Area prepared- painted with antiseptic lotion is from chin to nipples.

● Incision made is around 2 cm above the sterna notch in a horizontal manner. Length
of incision to be kept in between the sternocleidomastoid muscles.

● Incision is carried till the platysma is reached and then platysma also cut. Just below
the platysma is a relatively avascular plane and this is the plane in which both the inferior
and superior skin flaps are raised.

● Then the flaps are retracted with help of thyroid retractors, most popular in use is
Jolls thyroid retractor.

● After elevating the flaps the first major vein encountered is Anterior jugular vein. This
vein may be preserved or sacrified.

● The strap muscles encountered are - sternohyoid (superficial to) and sternothyroid
(just deeper). Both are retracted to finally cut investing fascia and expose the thyroid
gland.

● Middle thyroid vein is the first vascular structure that is ligated to initiate mobilization
of thyroid gland. Thyoid gland separation begins at lower pole.

● Ligation of arteries- both arteries, superior thyroid and inferior thyroid is to be ligated
as close to gland as possible. Inferior thyroid artery should not be ligated in a bunch but
only those branches that seem to be entering the thyroid gland. (Inferior thyroid artery
supplies both the superior and inferior parathyroid gland and branches to these glands
should be preserved).

● Finally, thyroid is elevated off the trachea by dissecting Berrys ligaments.

264/284
(Q.277) In primary hyperparathyroidism, which one of the following is the most common
finding?

(a) Solitary adenoma

(b) Double adenoma

(c) Parathyroid hyperplasia

(d) Carcinoma

Your Response :

Correct Answer : A

Exp: Solitary adenoma

● Primary hyperparathyroidism result from enlargement of single gland or parathyroid


adenoma in approximately 80% of cases multiple adenoma or hyperplasia in 15 – 20% of
cases and parathyroid carcinoma in 1% cases.

● Harrison writes, a single abnormal gland is the cause in 80% of cases.

(Q.278) Stimulation of chemoreceptors causes?

(a) Hypovolemia

(b) Hypervolemia

(c) Tachycardia

(d) Bradycardia

Your Response :

Correct Answer : D

Exp: Bradycardia

Bradycardia>>> tachycardia. Peripheral chemoreceptor stimulation has two effects on


medullary vagal centre (CIC/CVC). The principal effect (direct effect) is to stimulate CIC
and thereby Bradycardia. Second (indirect) effect is mediated by respiratory system
mediates. Chemoreceptor stimulates respiratory centre which inhibits CIC. This causes
tachycardia.

So, better answer is the direct effect: Bradycardia.

265/284
(Q.279) In which condition, the following type of ECG is seen?

(a) Delayed conduction through SA node.

(b) Delayed conduction through bundle of his.

(c) Delayed conduction through purkinje fibers.

(d) Delayed conduction through AV node.

Your Response : d

Correct Answer : D

Exp: Delayed conduction through AV node

Prolonged PR interval is seen in the ECG.

PR interval

● It is the time from the onset of the P wave to the start of the QRS complex.

● It reflects conduction through the AV node.

● The normal PR interval is between 120 – 200 ms duration (three to five small
squares).

● If the PR interval is > 200 ms, first degree heart block is said to be present.

● PR interval < 120 ms suggests pre-excitation (the presence of an accessory


pathway between the atria and ventricles) or AV nodal (junctional) rhythm.

Prolonged PR interval

● Delayed conduction through the AV node

● May occur in isolation or co-exist with other blocks (e.g., second-degree AV block,
trifascicular block)

266/284
(Q.280) Wernicke’s encephalopathy involves which part of CNS:

(a) Mamillary body

(b) Thalamus

(c) Amygdala

(d) Arcuate fasciculus

Your Response : b

Correct Answer : A

Exp: Mamillary body

Most common Mammillary body>> Thalamus.

Hippocampus not involved (no loss of neurone number, only small volume loss). The
anterior thalamus receives direct input from the hippocampus via the fornix and indirect
input from the hippocampus via the mammillary bodies and the mamillothalamic tract.
Disruption to these connections is at the heart of anterograde amnesia.

KORSAKOFF SYNDROME

Korsakoff syndrome becomes apparent in up to 80% of patients who survive Wernicke


encephalopathy. It is more likely to follow Wernicke encephalopathy in the setting of
alcoholism than in nutritional deficiency alone, thereby implying a synergistic mechanism
that may be due to repeated episodes of alcohol withdrawal with associated glutamate
neurotoxicity, compounded by lack of thiamine. The primary pathologic findings occur in
the limbic system, especially the mamillary bodies, amygdale, and dorsomedial and
anterior thalamus. Cortical involvement may be related to alcohol neurotoxicity rather
than thiamine deficiency.

267/284
(Q.281) Major neurotransmitter in afferents in nucleus tractus solitaries to regulate
cardiovascular system-

(a) Serotoxin

(b) Glutamate

(c) Glycine

(d) Norpinephrine

Your Response : d

Correct Answer : B

Exp: Glutamate

There is a general consensus that glutamate is the neurotransmitter released at the


terminals of baroreceptor cardiopulmonary and chemoreceptor afferents in the nTS.

268/284
(Q.282) Common posterior fossa brain tumors in children include all, except:

(a) Medulloblastoma

(b) Ependymoma

(c) Cerebellar astrocytoma

(d) Oligodendroglioma

Your Response :

Correct Answer : D

Exp: Oligodendroglioma

Among infra-tentorial (posterior fossa tumors), cerebellar astrocytoma is the most


common while medulloblastoma is the second most common brain tumor. Since majority
of ependymoma arise from the fourth ventricle lining, they are also included in the
posterior fossa tumors (probably the third most common). Oligodendrogliomas are rare.

269/284
(Q.283) A 5 year old child with Systemic-onset JIA (SOJIA) is found to have Leukopenia,
thrombocytopenia & raised ferritin levels in serum. What should be suspected?

(a) Development of ALL

(b) Development of MAS

(c) Clinical relapse of SOJIA

(d) Etanercept toxicity

Your Response :

Correct Answer : B

Exp: Development of MAS

● Juvenile Idiopathic Arthritis (JIA) is of many varieties, including Polyarticular,


Pauciarticular or Systemic onset.

● Macrophage activation syndrome (MAS) is a rare but potentially fatal complication of


systemic-onset juvenile idiopathic arthritis (SOJIA). It is also known as secondary
hemophagocytic syndrome.

● MAS manifests as anemia, leukopenia, thrombocytopenia with high fevers,


lymphadenopathy & hepatosplenomegaly. ESR typically falls due to
hypofibrinogenemia& serum ferritin is raised. Clinical bleeding may also occur.

● Confirmation is by BM biopsy showing evidence of hemophagocytosis.

● Emergency treatment with methylprednisolone, cyclosporine or anakinra may be


needed.

270/284
(Q.284) Parents bring their newborn daughter to you for consultation about diagnosis and
management. Their first two children, a boy and a girl, have a complete form of albinism
with pink irides, blond hair, and pale skin. Which of the following represents your correct
advice concerning the newborn child?

(a) A 1/8 risk for albinism and skin cancer from DNA deletions

(b) A 1/8 risk for albinism and skin cancer from DNA cross-linkage

(c) A 1/4 risk for albinism and skin cancer from DNA deletions

(d) A 1/4 risk for albinism and skin cancer from DNA cross-linkage

Your Response :

Correct Answer : D

Exp: A 1/4 risk for albinism and skin cancer from DNA cross-linkage

● Normal parents having two affected children, male and female, is suggestive of
autosomal recessive inheritance.

● This interpretation fits with the usual inheritance of oculocutaneous albinism,


implying a 1/4 risk for a newborn in whom signs and symptoms of albinism are not yet
evident. The defect in melanin synthesis in albinism decreases the amount of this
protective pigment in skin and increases the exposure of DNA in skin cells to sunlight.

● Ultraviolet rays from sunlight cause DNA cross-linkage between at least two bases in
the same or opposite strands of DNA. Crosslinking occurs through the formation of
thymine-thymine dimers.

● The DNA cross-links cause higher rates of mutation and skin cancer in albinism,
mandating the wearing of protective clothing, sunglasses, and sunscreens by affected
individuals. DNA deletions and point mutations are less common than DNA cross-links
after sunlight exposure.

(Q.285) Identify the disorder seen in the X-ray below:

271/284
(a) Laryngomalacia

(b) Recurrent laryngeal papillomatosis

(c) Epiglottitis

(d) Croup

Your Response : c

Correct Answer : D

Exp: Croup

This is a typical “steeple sign” seen in children with croup.

272/284
(Q.286) A child in the third grade has problems with spelling and reading. She appears very
quiet and confused in class. Her mind seems to wander whenever the teacher tells a story
or explains something complicated. She is skilled in art and so far has performed well in
arithmetic. Which of the following diagnostic procedures is most likely to yield useful
findings in this child?

(a) An intelligence test

(b) A language evaluation

(c) A psychiatric assessment to rule out depression

(d) A neurologic examination

Your Response : b

Correct Answer : B

Exp: A language evaluation

● The question offers a classic example of a language problem, particularly in a child


with partial understanding.

● Many children or parents do not admit to this problem.

273/284
(Q.287) Common site of regional enteritis in children:

(a) Duodenum-jejunum

(b) Ileum-caecum

(c) Colon-rectum

(d) Rectum

Your Response :

Correct Answer : B

Exp: Ileum-caecum

Due to patchy nature of intestinal involvement of Crohn’s disease, it is also referred to as


regional enteritis. Ileocecal portion of the intestine is the most commonly involved portion
in Crohn’s disese; while colorectal portion is the most commonly involved part of the
intestine in Ulcerative colitis.

274/284
(Q.288) Which Vitamin causes Toxicity?

(a) Vitamin B1

(b) Vitamin B12

(c) Vitamin B3

(d) Vitamin C

Your Response : a

Correct Answer : B

Exp: Vitamin B3 –Niacin

Niacin Toxicity:

Nicotinic acid has been used to


treat hyperlipidemia (in dose of 1-
6 g/d). This dose causes
dilatation of capillaries →
Flushing, Rash Excessive
sweating, blurred vision, impaired
glucose tolerance. Intake of both
nicotinic acid & nicotinamide in
excess of 500 gm/d causes Liver
damage.

275/284
(Q.289) Which of the following statements about nucleotide metabolism is NOT CORRECT?

(a) An early step in purine biosynthesis is formation of PRPP (Phospho Ribosyl


PyroPhosphate)

(b) IMP(Inosine Monophosphate is precursor of both CTP & TMP

(c) Orotic acid is an intermediate in pyrimidine nucleotide biosynthesis

(d) Ribonucleotide Reductase converts ribonucleoside diphosphated to the corresponding


deoxy-ribonucleoside diphosphates.

Your Response : d

Correct Answer : B

Exp: IMP(Inosine Monophosphate is precursor of both CTP & TMP

IMP (Inosine Monophosphate) is precursor of both AMP & GMP (Not CTP & TMP)

An early step in purine biosynthesis is formation of PRPP(Phospho Ribosyl PyroPhate).


Orotic acid is an intermediate in pyrimidine nucleotide biosynthesis Ribonucleotide
Reductase coverts ribonucleoside diphosphates.

(Q.290) Which of the following is true regarding mitochondrial DNA?

(a) One set from each parents

(b) It has 3 x 109 base pairs

(c) It has more mutations than nuclear DNA

(d) It codes for less than 20% of oxidative phosphorylation proteins.

Your Response :

Correct Answer : C

Exp: It has more mutations than nuclear DNA

Mitochondrial DNA has no introns. So it has more chances of mutations. Nuclear DNA
has introns, which prevents mutations.

276/284
(Q.291) A normal anion gap acidosis is most likely to be due to:

(a) DKA

(b) Renal tubular acidosis

(c) Methanol poisoning

(d) Uremia

Your Response : b

Correct Answer : B

Exp: Renal tubular acidosis

Renal tubular acidosis with renal bicarbonate loss and diarrhea-induced stool losses of
bicarbonate are the common causes of a normal anion gap acidosis.

(Q.292) A child died soon after birth. On examination there was hepatosplenomegaly and edema
all over body. Most probable diagnosis is ; -

(a) β-thalassemia

(b) α- Thalassemia

(c) Hereditary spherocytosis

(d) ABO incompatibility /sickle cell anemia.

Your Response : b

Correct Answer : B

Exp: α- Thalassemia

● Out of the given option, α -thalassemia may presents in so severs form.

● “ Homozygous state for α - haplotype is most severe form of disease.

● Babies are born either with hydrops fetalis secondary to congestive -eart failure or
die in utero.’

● Hydropic infants have very little useful hemoglobin.

● Affected infants are still born between 30-40 weeks or die shortly after birth.

(Q.293) Graph shown below is the titration curve of a biochemical compoun d. Which of the
following statement is true?

277/284
(a) The maximum buffering capacity of the compound is represented by points A and B

(b) The points A and B represent the range of ionisation of the amine and carboxyl group

(c) The compound has three ionisable side chains

(d) The compound has one ionisable group

Your Response :

Correct Answer : B

Exp: The points A and B represent the range of ionisation of the amine and carboxyl
group

● On titrating a weak polyprotic multiple equivalence points will occur

● Midpoint: When the number of moles of strong base added equals half of the moles
of weak acid already present; thus, the midpoint is half of the equivalence point

o At midpoint, pH = pKa

o Buffering capacity is maximum

● The curve starts at a higher pH than titration curve of a strong base

● There is a steep climb in pH before the first midpoint

● Gradual increase of pH until past the midpoint.

● Right before the equivalence point there is a sharp increase in pH

● pH steadies itself around the midpoint because the solutions at this point in the
curve are buffer solutions, which means that adding small increments of a strong base
will only barely change the ph

● Increase in pH near the equivalence point

● Number of equivalence points determine the number of ionisable groups; So here


toe ionisable group are there

278/284
(Q.294) Deficiency causing manifestation shown in Photograph:

(a) Vitamin Thiamine

(b) Vitamin Riboflavin

(c) Vitamin Niacin

(d) Vitamin Ascorbic acid

Your Response : d

Correct Answer : B

Exp: Vitamin Riboflavin

Symptoms and signs of vitamin


riboflavin deficiency include sore
throat , lesions of the lips,
cheiliosis and mucosa of the
mouth, glossitis, conjunctivitis,
seborrheic dermatitis, impaired
protein synthesis and
normochromic normocytic
anemia.

(Q.295) A 9 month-old boy is brought to the clinic by his mother who is concerned that the boy
developed swelling in his right scrotum. On examination, there is a scrotal mass that
disappears when pushing it back to the external inguinal ring. When the boy cries, the
mass is felt again protruding out of the external inguinal ring. What is the most likely
etiology of this condition?

(a) Defect in the inguinal canal

(b) Defect in the pelvic floor

(c) Patent processus vaginalis

(d) Undescended testis

Your Response : c

Correct Answer : C

Exp: Patent processus vaginalis

The most important cause of inguinal hernia in infants is patent processus vaginalis
which fails to be obliterated after descending of testes. Persistent lumen of tunica
vaginalis causes hydrocele, not hernias.

279/284
(Q.296) A floppy infant was brought with difficulty in breathing. CXR was performe d. Family
history was positive for death of sibling at 1 year of age. Which of the following enzyme
deficiency will lead to this presentation?

(a) Muscle phosphorylase deficiency

(b) Acid maltase deficiency

(c) Glucose-6- phosphatase deficiency

(d) Arylsulfatase B deficiency

Your Response :

Correct Answer : B

Exp: Acid maltase deficiency

● The image shows a floppy infant suggestive of muscle weakness. Th CXR shows
presence of increased Cardiothoracic ratio. This indicates cardiac muscle involvement
well.

● Due to positive family history indicating genetic basis of disease and clinical findings
given, the diagnosis of Pompe’s disease is the first diagnosis.

280/284
(Q.297) Statement incorrect about development is ?

(a) Pincer grasp at 3 months.

(b) Sitting at 6 months.

(c) Social smile at 3 months.

(d) 2 year old can use pleurals.

Your Response : a

Correct Answer : A

Exp: Pincer grasp at 3 months

● Grasp:

● Bidextrous – 5 months

● Palmar grasp – 7 months

● Pincer grasp -9 months

● Thus, pincer grasp development during 8th month of life.

281/284
(Q.298) Main action of Ivabradine is?

(a) ACEI

(b) Bradycardia

(c) Diuretic

(d) Calcium channel blocker

Your Response : a

Correct Answer : B

Exp: Bradycardia

It acts by reducing HR via specific Inhibition of the funny channels. It is used for
symptomatic treatment of chronic stable angina pectoris.

282/284
(Q.299) Gallavardin sign is seen in?

(a) AS

(b) AR

(c) MVP

(d) HOCM

Your Response : a

Correct Answer : A

Exp: AS

Ejection systolic murmur heard at apex, left sternal edge and aortic area which radiates
to the carotids (Gallavardin Phenomenon) is seen in AS.

283/284
(Q.300) Best Initial treatment for MODY is low dose of ________?

(a) Metformin

(b) Insulin

(c) Acarbose

(d) Sulphonylurea

Your Response : d

Correct Answer : D

Exp: Sulphonylurea

me can be maintained on Sulfonylurea for many decades and glycemic control is often
better than that achieved on Insulin. Initiates dose can be 1/4th the Normal starting dose.

284/284

Das könnte Ihnen auch gefallen